[obm-l] Re: [obm-l] Teoria dos números, trigonometria e racionalidade

2022-12-11 Por tôpico Anderson Torres
Em dom., 11 de dez. de 2022 às 10:32, Anderson Torres
 escreveu:
>
> Em sáb., 10 de dez. de 2022 às 22:08, marcone augusto araújo borges
>  escreveu:
> >
> > Seja p um número primo tal que p = = 3 (mod4) e @ um ângulo tal que tan@ é 
> > racional. Prove que tan((p+1)@) também é racional com numerador múltiplo de 
> > p
> > Desde já agradeço por algum esclarecimento ou solução.
>
> Bem, o que eu consigo pensar é em algo desse tipo.
>
> Sabemos que tan(m+n) = (tan(m) + tan(n))/(1-tan(m)* tan(n))
>
> Escrevamos tan(nX)=p(n)/q(n), onde p e q são polinômios em t=tan(X).
> Temos então a seguinte recorrência:
>
> p(1)=t; p(n+1)=p(n)+tq(n)
> q(1)=1; q(n+1)=-tp(n)+q(n)
>
> Jogando aqui e ali, temos
>
> p(1)=t; p(2)=2t; p(n+2)=2p(n+1)-(t^2+1)p(n)
> q(1)=1; q(2)=1-t^2; q(n+2)=2q(n+1)-(t^2+1)q(n)
>
> De cara, se nota que p sempre será múltiplo de p, e que q sempre deixa
> resto 1 módulo t, o que já dá uma pista do que procurar...
> Decerto, vai aparecer alguma coisa do tipo x^2+1, e com isso se usa o
> fato de p ser primo da forma 4k-1...
>

Acho que dá para melhorar. Suponha tan(nX)=A(n)/B(n). Assim,

A(n+1) =  B*A(n) + A*B(n)
B(n+1) = -A*A(n) + B*B(n)

E portanto

A(n+2) = 2B*A(n+1) - (A^2+B^2)*A(n), A(1)=A, A(2)=2AB
B(n+2) = 2B*B(n+1) - (A^2+B^2)*B(n), B(1)=B, B(2)=B^2-A^2

A ideia então seria demonstrar que A(p+1) é múltiplo de p para p primo
da forma 4k-1, e B(p+1) não é múltiplo de p para p primo da forma
4k-1.

Dessa forma, ao menos em princípio seria possível verificar a segunda
premissa, pois a primeira é óbvia.

> >
> > --
> > Esta mensagem foi verificada pelo sistema de antivírus e
> > acredita-se estar livre de perigo.

-- 
Esta mensagem foi verificada pelo sistema de antiv�rus e
 acredita-se estar livre de perigo.


=
Instru��es para entrar na lista, sair da lista e usar a lista em
http://www.mat.puc-rio.br/~obmlistas/obm-l.html
=


[obm-l] Re: [obm-l] Teoria dos números, trigonometria e racionalidade

2022-12-11 Por tôpico Anderson Torres
Em sáb., 10 de dez. de 2022 às 22:08, marcone augusto araújo borges
 escreveu:
>
> Seja p um número primo tal que p = = 3 (mod4) e @ um ângulo tal que tan@ é 
> racional. Prove que tan((p+1)@) também é racional com numerador múltiplo de p
> Desde já agradeço por algum esclarecimento ou solução.

Bem, o que eu consigo pensar é em algo desse tipo.

Sabemos que tan(m+n) = (tan(m) + tan(n))/(1-tan(m)* tan(n))

Escrevamos tan(nX)=p(n)/q(n), onde p e q são polinômios em t=tan(X).
Temos então a seguinte recorrência:

p(1)=t; p(n+1)=p(n)+tq(n)
q(1)=1; q(n+1)=-tp(n)+q(n)

Jogando aqui e ali, temos

p(1)=t; p(2)=2t; p(n+2)=2p(n+1)-(t^2+1)p(n)
q(1)=1; q(2)=1-t^2; q(n+2)=2q(n+1)-(t^2+1)q(n)

De cara, se nota que p sempre será múltiplo de p, e que q sempre deixa
resto 1 módulo t, o que já dá uma pista do que procurar...
Decerto, vai aparecer alguma coisa do tipo x^2+1, e com isso se usa o
fato de p ser primo da forma 4k-1...

>
> --
> Esta mensagem foi verificada pelo sistema de antivírus e
> acredita-se estar livre de perigo.

-- 
Esta mensagem foi verificada pelo sistema de antiv�rus e
 acredita-se estar livre de perigo.


=
Instru��es para entrar na lista, sair da lista e usar a lista em
http://www.mat.puc-rio.br/~obmlistas/obm-l.html
=


[obm-l] Re: [obm-l] Re: [obm-l] Re: [obm-l] Teoria dos Números

2020-10-26 Por tôpico Otávio Araújo
De nada mano.

Em seg, 26 de out de 2020 09:40, joao pedro b menezes <
joaopedrobmene...@gmail.com> escreveu:

> Muito obrigado pela ajuda! Entendi o exercício agora.
>
> Em dom, 25 de out de 2020 às 19:59, Otávio Araújo <
> otavio17.ara...@gmail.com> escreveu:
>
>> Vc resolve essa questão mostrando q p=n^2+n+1.  Se n=1 acabou. Se n>1,Já
>> que p divide n^3-1 e é primo, temos que p divide n-1 ou n^2+n+1. Não
>> podemos ter p dividindo n-1 pois n divide p-1 -> n<= p-1 n-1> Portanto p divide n^2+n+1. Faca n^2+n+1 = kp, k inteiro positivo. Temos que
>> kp=n^2+n+1 é congruente a 1 módulo n. Do enunciado temos p congruente a 1
>> módulo n,  mas p é congruente a 1 módulo n e é diferente de 1(pois é primo)
>> -> p>= n+1 e  k será congruente a 1 módulo n também. Suponha que k>1,
>> k>1 implica k>= n+1 daí kp>=(n+1)^2 > n^2+n+1, contradição. Portanto
>> k=1 e p=n^2+n+1.
>>
>> Em dom, 25 de out de 2020 17:37, joao pedro b menezes <
>> joaopedrobmene...@gmail.com> escreveu:
>>
>>> Olá, boa tarde.
>>> Estou com dúvida nesse exercício:
>>> " Sejam n um inteiro positivo maior que 1 e p um primo positivo tal que
>>> n divide p − 1 e p divide n 3 − 1. Mostre que 4p − 3 ´e um quadrado
>>> perfeito."
>>> Já agradeço pela ajuda e pelo tempo!
>>>
>>>


[obm-l] Re: [obm-l] Re: [obm-l] Teoria dos Números

2020-10-26 Por tôpico joao pedro b menezes
Muito obrigado pela ajuda! Entendi o exercício agora.

Em dom, 25 de out de 2020 às 19:59, Otávio Araújo 
escreveu:

> Vc resolve essa questão mostrando q p=n^2+n+1.  Se n=1 acabou. Se n>1,Já
> que p divide n^3-1 e é primo, temos que p divide n-1 ou n^2+n+1. Não
> podemos ter p dividindo n-1 pois n divide p-1 -> n<= p-1 n-1 Portanto p divide n^2+n+1. Faca n^2+n+1 = kp, k inteiro positivo. Temos que
> kp=n^2+n+1 é congruente a 1 módulo n. Do enunciado temos p congruente a 1
> módulo n,  mas p é congruente a 1 módulo n e é diferente de 1(pois é primo)
> -> p>= n+1 e  k será congruente a 1 módulo n também. Suponha que k>1,
> k>1 implica k>= n+1 daí kp>=(n+1)^2 > n^2+n+1, contradição. Portanto  k=1
> e p=n^2+n+1.
>
> Em dom, 25 de out de 2020 17:37, joao pedro b menezes <
> joaopedrobmene...@gmail.com> escreveu:
>
>> Olá, boa tarde.
>> Estou com dúvida nesse exercício:
>> " Sejam n um inteiro positivo maior que 1 e p um primo positivo tal que n
>> divide p − 1 e p divide n 3 − 1. Mostre que 4p − 3 ´e um quadrado perfeito."
>> Já agradeço pela ajuda e pelo tempo!
>>
>>


[obm-l] Re: [obm-l] Teoria dos Números

2020-10-25 Por tôpico Otávio Araújo
Vc resolve essa questão mostrando q p=n^2+n+1.  Se n=1 acabou. Se n>1,Já
que p divide n^3-1 e é primo, temos que p divide n-1 ou n^2+n+1. Não
podemos ter p dividindo n-1 pois n divide p-1 -> n<= p-1 n-1 p>= n+1 e  k será congruente a 1 módulo n também. Suponha que k>1,  k>1
implica k>= n+1 daí kp>=(n+1)^2 > n^2+n+1, contradição. Portanto  k=1 e
p=n^2+n+1.

Em dom, 25 de out de 2020 17:37, joao pedro b menezes <
joaopedrobmene...@gmail.com> escreveu:

> Olá, boa tarde.
> Estou com dúvida nesse exercício:
> " Sejam n um inteiro positivo maior que 1 e p um primo positivo tal que n
> divide p − 1 e p divide n 3 − 1. Mostre que 4p − 3 ´e um quadrado perfeito."
> Já agradeço pela ajuda e pelo tempo!
>
>


[obm-l] Re: [obm-l] Teoria dos números

2020-04-30 Por tôpico Anderson Torres
Em qua., 29 de abr. de 2020 às 10:33, Anderson Torres
 escreveu:
>
> Em qui., 23 de abr. de 2020 às 06:31, Jeferson Almir
>  escreveu:
> >
> > Amigos, peço ajuda nessa questão.
> >
> >  Sejam a e b inteiros positivos >=2 tal que (a^n)-1|(b^n)-1 pra todos os 
> > inteiros positivos n,mostrar que b é potencia inteira de a.
> >
>
> Ajuda? Esse problema é bem dificinho.
>
> A ideia é, por absurdo, supor que exista K tal que a^K < b < a^(K+1),
> verificar que x_n = (b^n-1) / (a^n-1) é sempre inteiro e a partir dela
> gerar uma sequência de inteiros que converge para 0.

Vou ser um pouco mais explícito.

Seja X(1,N) = (b^N-1) / (a^N-1). Pense que a é uma constante aqui.

Note que X(1,N) tem um comportamento parecido com (b/a)^N. Vamos então
considerar a sequência

X(2,N) = b*X(1,N) - a*X(1,N+1)

Veja que X(2,N) = ((a-1)*b^(n+1) + (b-1)*a^(n+1) +(a-b))/((a^n-1)(a^(n+1)-1)).

Esta bagaça não necessariamente converge, mas note que ela tem um
comportamento parecido com  (b/a^2)^N, o que é muito mais lento que
(b/a)^N.

Então, teremos essa pilha de funções:

X(1,N) = ...
X(2,N) = b*X(1,N) - a*X(1,N+1)
X(3,N) = b*X(2,N) - a*X(2,N+1)
X(4,N) = b*X(3,N) - a*X(3,N+1)

Tua tarefa será demonstrar que em algum momento uma dessas funções
X(k,N) converge para 0. Tendo feito isso, a demonstração será fácil.

>
> > --
> > Esta mensagem foi verificada pelo sistema de antivírus e
> > acredita-se estar livre de perigo.

-- 
Esta mensagem foi verificada pelo sistema de antiv�rus e
 acredita-se estar livre de perigo.


=
Instru��es para entrar na lista, sair da lista e usar a lista em
http://www.mat.puc-rio.br/~obmlistas/obm-l.html
=


[obm-l] Re: [obm-l] Teoria dos números

2020-04-29 Por tôpico Anderson Torres
Em qui., 23 de abr. de 2020 às 06:31, Jeferson Almir
 escreveu:
>
> Amigos, peço ajuda nessa questão.
>
>  Sejam a e b inteiros positivos >=2 tal que (a^n)-1|(b^n)-1 pra todos os 
> inteiros positivos n,mostrar que b é potencia inteira de a.
>

Ajuda? Esse problema é bem dificinho.

A ideia é, por absurdo, supor que exista K tal que a^K < b < a^(K+1),
verificar que x_n = (b^n-1) / (a^n-1) é sempre inteiro e a partir dela
gerar uma sequência de inteiros que converge para 0.

> --
> Esta mensagem foi verificada pelo sistema de antivírus e
> acredita-se estar livre de perigo.

-- 
Esta mensagem foi verificada pelo sistema de antiv�rus e
 acredita-se estar livre de perigo.


=
Instru��es para entrar na lista, sair da lista e usar a lista em
http://www.mat.puc-rio.br/~obmlistas/obm-l.html
=


[obm-l] Re: [obm-l] Re: [obm-l] teoria dos números

2020-04-11 Por tôpico Israel Meireles Chrisostomo
muito obrigado, professor  Ralph

Em sáb., 11 de abr. de 2020 às 13:18, Ralph Costa Teixeira <
ralp...@gmail.com> escreveu:

> Tome por exemplo
> a=1
> b=xy
> c=y
>
> Mais genericamente
> a=k
> b=kxy
> c=ky
> servem para k≠0 complexo qualquer.
>
> On Sat, Apr 11, 2020, 11:17 Israel Meireles Chrisostomo <
> israelmchrisost...@gmail.com> wrote:
>
>> Como posso provar a seguinte afirmação "Sejam x,y,z números complexos
>> tais que xyz=1, mostre que existem a,b,c complexos tais que
>> b/c=x,c/a=y,a/b=z"
>>
>> --
>> Israel Meireles Chrisostomo
>>
>> --
>> Esta mensagem foi verificada pelo sistema de antivírus e
>> acredita-se estar livre de perigo.
>
>
> --
> Esta mensagem foi verificada pelo sistema de antivírus e
> acredita-se estar livre de perigo.



-- 
Israel Meireles Chrisostomo

-- 
Esta mensagem foi verificada pelo sistema de antiv�rus e
 acredita-se estar livre de perigo.



[obm-l] Re: [obm-l] teoria dos números

2020-04-11 Por tôpico Ralph Costa Teixeira
Tome por exemplo
a=1
b=xy
c=y

Mais genericamente
a=k
b=kxy
c=ky
servem para k≠0 complexo qualquer.

On Sat, Apr 11, 2020, 11:17 Israel Meireles Chrisostomo <
israelmchrisost...@gmail.com> wrote:

> Como posso provar a seguinte afirmação "Sejam x,y,z números complexos tais
> que xyz=1, mostre que existem a,b,c complexos tais que b/c=x,c/a=y,a/b=z"
>
>
> --
> Israel Meireles Chrisostomo
>
> --
> Esta mensagem foi verificada pelo sistema de antivírus e
> acredita-se estar livre de perigo.

-- 
Esta mensagem foi verificada pelo sistema de antiv�rus e
 acredita-se estar livre de perigo.



[obm-l] Re: [obm-l] Re: [obm-l] Re: [obm-l] Re: [obm-l] Teoria dos números

2020-03-22 Por tôpico Israel Meireles Chrisostomo
O meu sonho tmbm é esse kk

Em dom., 22 de mar. de 2020 às 13:22, Israel Meireles Chrisostomo <
israelmchrisost...@gmail.com> escreveu:

> vc é engenheiro?
>
> Em dom., 22 de mar. de 2020 às 13:19, Israel Meireles Chrisostomo <
> israelmchrisost...@gmail.com> escreveu:
>
>> mas vc possui algum graduação ?
>>
>> Em dom., 22 de mar. de 2020 às 13:00, Pedro José 
>> escreveu:
>>
>>> Boa tarde!
>>> Perfeita a sua correção.
>>> Quanto ao questionamento, nem tenho formação em matemática, meu sonho é
>>> cursar no IMPA ao me aposentar. Sou pitaqueiro. Ouço um assunto que não
>>> conheço, tento aprendê-lo. Na verdade, gosto de matemática. Talvez seja ela
>>> o "Mundo das ideias", o mundo ideal, a qual Platão se referiu.
>>> Saudações,
>>> PJMS
>>>
>>> Em dom, 22 de mar de 2020 12:25, Israel Meireles Chrisostomo <
>>> israelmchrisost...@gmail.com> escreveu:
>>>
 Acho q tem uma ´pequena correção no seguinte passo "4k+1. Pegando os
 fatores (4n-1)^2 e (4n+1)^2, teremos que 2^6 |p(n) para qualquer n=4k+1."O
 correto seria "Para n=4k+1.Pegando os fatores (n-1)^2 e (n+1)^2"

 Em dom., 22 de mar. de 2020 às 10:14, Israel Meireles Chrisostomo <
 israelmchrisost...@gmail.com> escreveu:

> Primeiramente obrigado pela solução.Mas Pedro, tenho uma pergunta :   o
> sr. é professor de Matemática?
>
> Em dom., 22 de mar. de 2020 às 01:34, Pedro José 
> escreveu:
>
>> Bom dia!
>> Dei uma mancada.
>> O expoente de 3 é 3 e não 2.
>> Retornando às classes mod 3.
>> Ao último fator é côngruo à (n-1)*n
>> Para n=3k aparece outro fator e 3^3|p(n), n=3k.
>> n=3k+1, tenho (n-1)^2 e (n-1), 3^3|p(n), n=3k+1
>> n=3k+2, tenho(n-2)^2 é (n+1)^2, 3^3|p(n), n=3k+2,
>> Logo 3^3|p(n) para todo n inteiro.
>> D>=2^6*3^3*5. Mas D<=2^6*3^3*5, então D=8640
>> Desculpem-me pelo erro.
>> Saudações,
>> PJMS.
>>
>>
>>
>> Em sáb, 21 de mar de 2020 13:20, Pedro José 
>> escreveu:
>>
>>> Boa tarde!
>>> Nem carece método numérico.
>>> Para n=1 ou n=0 ou n=2 temos que qualquer inteiro divide o polinômio
>>> p(n)=(n-2)^2*(n-1)^2*n^2*(n+1)^2*(4n^2-4n-9)
>>>
>>> p(3)=8640
>>> p(4)=561600 então (p(3),p(4))=8640=2^6*3^3*5.
>>> Seja D o maior inteiro que divide p(n) para todo n inteiro, D<=8640
>>> Vamos pegar as classes de equivalência mod 4. Seja k um inteiro.
>>> Para 4k temos que n^2= 16k^2 e (n-2) é par logo (n-2)^2= 4s^2 com s
>>> inteiro. Logo 2^6 divide p(n) para qualquer n =4k.
>>> 4k+1. Pegando os fatores (4n-1)^2 e (4n+1)^2, teremos que 2^6 |p(n)
>>> para qualquer n=4k+1.
>>> 4k+2. Pegando (n-2)^2 e n^2, teremos que 2^6|p(n) para qualquer
>>> n=4k+2
>>> 4k+3, pegando os mesmos fatores de 4k+1, 2^6|p(n) para n=4k+3.
>>> Portanto 2^6|p(n) para qualquer inteiro
>>> Agora classes de equivalência mod 3
>>> 3k, pegando n^2, 3^2|p(n) para n=3k
>>> 3k+1, pegando (n-1)^2; 3^2| p(n), n=3k+1
>>> 3k+2, pegando (n-2)^2, 3^2| p(n), n=3k+2
>>> Daí 3^2|p(n) para qualquer n inteiro.
>>> Classes de equivalência mod 5.
>>> 5k, n^2 , 5 |p(n), n =5
>>> 5k +1, (n-1)^2, 5|p(n), n=5k+1
>>> 5k+2, (n-2)^2, 5|p(n), n=5k+2
>>> 5k+3, (4n^2-4n-9)=(100k^2-100k+15)
>>> 5|p(n), n=5k+3
>>> 5k+4, (n+1)^2, 5|p(n) , n=5k+4.
>>> Então 5|p(n) para todo inteiro
>>> D>=2^6*3^2×*5
>>> Mas D<=2^6*3^2*5, logo D=8640
>>>
>>> Saudações,
>>> PJMS
>>>
>>> Em sáb, 21 de mar de 2020 04:39, Pedro José 
>>> escreveu:
>>>
 Bom dia!
 Falta de novo, em seu questionamento, informar que n é inteiro ou
 natural e colocar a condição para qualquer valor de n. Chamando o 
 polinômio
 de p(n)
 Para n=0, 1 ou 2, qualquer inteiro divide.
 Faria mdc(p(3),p(4))= A1
 Se der "pequeno", com poucos fatores primos e expoentes pequenos.
 Paro em A1, se não.
 (p(5),A1)=A2 uso o mesmo critério de parar
 (p(6),A2)=A3 até parar em:
 Ai=(p(i+3),A(i-1)).
 Aí faço o polinômio módfi^xi, onde fi é um fator primo de Aí e xi
 seu expoente. verifico se para cada resíduon= 1, 2...fi^n-1 se P(n)=0 
 mod
 fi^si
 Se falhar diminuto xi em 1 e repito o teste para todos resíduos de
 fi^(xi-1)-1 até um dado xki em que todos os p(resíduos) foram 
 equivalente a
 zero módulo fi^xki ou quando fizer para o expoente 1  e não zerar para
 todos resíduos de fi, quando o fator será descartado.
 Depois repito para cada fator primo f e seu respectivo expoente.
 Ao final D = Produtório de cada fator fi elevado ao expoente xki
 que zerou p(n) mod fi^xki para todos os resíduos, descartando os fí em 
 que
 xji chegou a 1 e não atendeu ou considerando nesse caso xki=0.

 Mas resolveria por método numérico.
 Depois poste sua solução.

 

[obm-l] Re: [obm-l] Re: [obm-l] Re: [obm-l] Re: [obm-l] Teoria dos números

2020-03-22 Por tôpico Israel Meireles Chrisostomo
vc é engenheiro?

Em dom., 22 de mar. de 2020 às 13:19, Israel Meireles Chrisostomo <
israelmchrisost...@gmail.com> escreveu:

> mas vc possui algum graduação ?
>
> Em dom., 22 de mar. de 2020 às 13:00, Pedro José 
> escreveu:
>
>> Boa tarde!
>> Perfeita a sua correção.
>> Quanto ao questionamento, nem tenho formação em matemática, meu sonho é
>> cursar no IMPA ao me aposentar. Sou pitaqueiro. Ouço um assunto que não
>> conheço, tento aprendê-lo. Na verdade, gosto de matemática. Talvez seja ela
>> o "Mundo das ideias", o mundo ideal, a qual Platão se referiu.
>> Saudações,
>> PJMS
>>
>> Em dom, 22 de mar de 2020 12:25, Israel Meireles Chrisostomo <
>> israelmchrisost...@gmail.com> escreveu:
>>
>>> Acho q tem uma ´pequena correção no seguinte passo "4k+1. Pegando os
>>> fatores (4n-1)^2 e (4n+1)^2, teremos que 2^6 |p(n) para qualquer n=4k+1."O
>>> correto seria "Para n=4k+1.Pegando os fatores (n-1)^2 e (n+1)^2"
>>>
>>> Em dom., 22 de mar. de 2020 às 10:14, Israel Meireles Chrisostomo <
>>> israelmchrisost...@gmail.com> escreveu:
>>>
 Primeiramente obrigado pela solução.Mas Pedro, tenho uma pergunta :   o
 sr. é professor de Matemática?

 Em dom., 22 de mar. de 2020 às 01:34, Pedro José 
 escreveu:

> Bom dia!
> Dei uma mancada.
> O expoente de 3 é 3 e não 2.
> Retornando às classes mod 3.
> Ao último fator é côngruo à (n-1)*n
> Para n=3k aparece outro fator e 3^3|p(n), n=3k.
> n=3k+1, tenho (n-1)^2 e (n-1), 3^3|p(n), n=3k+1
> n=3k+2, tenho(n-2)^2 é (n+1)^2, 3^3|p(n), n=3k+2,
> Logo 3^3|p(n) para todo n inteiro.
> D>=2^6*3^3*5. Mas D<=2^6*3^3*5, então D=8640
> Desculpem-me pelo erro.
> Saudações,
> PJMS.
>
>
>
> Em sáb, 21 de mar de 2020 13:20, Pedro José 
> escreveu:
>
>> Boa tarde!
>> Nem carece método numérico.
>> Para n=1 ou n=0 ou n=2 temos que qualquer inteiro divide o polinômio
>> p(n)=(n-2)^2*(n-1)^2*n^2*(n+1)^2*(4n^2-4n-9)
>>
>> p(3)=8640
>> p(4)=561600 então (p(3),p(4))=8640=2^6*3^3*5.
>> Seja D o maior inteiro que divide p(n) para todo n inteiro, D<=8640
>> Vamos pegar as classes de equivalência mod 4. Seja k um inteiro.
>> Para 4k temos que n^2= 16k^2 e (n-2) é par logo (n-2)^2= 4s^2 com s
>> inteiro. Logo 2^6 divide p(n) para qualquer n =4k.
>> 4k+1. Pegando os fatores (4n-1)^2 e (4n+1)^2, teremos que 2^6 |p(n)
>> para qualquer n=4k+1.
>> 4k+2. Pegando (n-2)^2 e n^2, teremos que 2^6|p(n) para qualquer n=4k+2
>> 4k+3, pegando os mesmos fatores de 4k+1, 2^6|p(n) para n=4k+3.
>> Portanto 2^6|p(n) para qualquer inteiro
>> Agora classes de equivalência mod 3
>> 3k, pegando n^2, 3^2|p(n) para n=3k
>> 3k+1, pegando (n-1)^2; 3^2| p(n), n=3k+1
>> 3k+2, pegando (n-2)^2, 3^2| p(n), n=3k+2
>> Daí 3^2|p(n) para qualquer n inteiro.
>> Classes de equivalência mod 5.
>> 5k, n^2 , 5 |p(n), n =5
>> 5k +1, (n-1)^2, 5|p(n), n=5k+1
>> 5k+2, (n-2)^2, 5|p(n), n=5k+2
>> 5k+3, (4n^2-4n-9)=(100k^2-100k+15)
>> 5|p(n), n=5k+3
>> 5k+4, (n+1)^2, 5|p(n) , n=5k+4.
>> Então 5|p(n) para todo inteiro
>> D>=2^6*3^2×*5
>> Mas D<=2^6*3^2*5, logo D=8640
>>
>> Saudações,
>> PJMS
>>
>> Em sáb, 21 de mar de 2020 04:39, Pedro José 
>> escreveu:
>>
>>> Bom dia!
>>> Falta de novo, em seu questionamento, informar que n é inteiro ou
>>> natural e colocar a condição para qualquer valor de n. Chamando o 
>>> polinômio
>>> de p(n)
>>> Para n=0, 1 ou 2, qualquer inteiro divide.
>>> Faria mdc(p(3),p(4))= A1
>>> Se der "pequeno", com poucos fatores primos e expoentes pequenos.
>>> Paro em A1, se não.
>>> (p(5),A1)=A2 uso o mesmo critério de parar
>>> (p(6),A2)=A3 até parar em:
>>> Ai=(p(i+3),A(i-1)).
>>> Aí faço o polinômio módfi^xi, onde fi é um fator primo de Aí e xi
>>> seu expoente. verifico se para cada resíduon= 1, 2...fi^n-1 se P(n)=0 
>>> mod
>>> fi^si
>>> Se falhar diminuto xi em 1 e repito o teste para todos resíduos de
>>> fi^(xi-1)-1 até um dado xki em que todos os p(resíduos) foram 
>>> equivalente a
>>> zero módulo fi^xki ou quando fizer para o expoente 1  e não zerar para
>>> todos resíduos de fi, quando o fator será descartado.
>>> Depois repito para cada fator primo f e seu respectivo expoente.
>>> Ao final D = Produtório de cada fator fi elevado ao expoente xki que
>>> zerou p(n) mod fi^xki para todos os resíduos, descartando os fí em que 
>>> xji
>>> chegou a 1 e não atendeu ou considerando nesse caso xki=0.
>>>
>>> Mas resolveria por método numérico.
>>> Depois poste sua solução.
>>>
>>> Saudações,
>>> PJMS.
>>>
>>>
>>>
>>>
>>> Em sex, 20 de mar de 2020 12:42, Israel Meireles Chrisostomo <
>>> israelmchrisost...@gmail.com> escreveu:
>>>
 Qual o maior inteiro que divide (n - 2)^2 (n - 1)^2 n^2 (n + 1)^2
 (4 n^2 

[obm-l] Re: [obm-l] Re: [obm-l] Re: [obm-l] Re: [obm-l] Teoria dos números

2020-03-22 Por tôpico Israel Meireles Chrisostomo
mas vc possui algum graduação ?

Em dom., 22 de mar. de 2020 às 13:00, Pedro José 
escreveu:

> Boa tarde!
> Perfeita a sua correção.
> Quanto ao questionamento, nem tenho formação em matemática, meu sonho é
> cursar no IMPA ao me aposentar. Sou pitaqueiro. Ouço um assunto que não
> conheço, tento aprendê-lo. Na verdade, gosto de matemática. Talvez seja ela
> o "Mundo das ideias", o mundo ideal, a qual Platão se referiu.
> Saudações,
> PJMS
>
> Em dom, 22 de mar de 2020 12:25, Israel Meireles Chrisostomo <
> israelmchrisost...@gmail.com> escreveu:
>
>> Acho q tem uma ´pequena correção no seguinte passo "4k+1. Pegando os
>> fatores (4n-1)^2 e (4n+1)^2, teremos que 2^6 |p(n) para qualquer n=4k+1."O
>> correto seria "Para n=4k+1.Pegando os fatores (n-1)^2 e (n+1)^2"
>>
>> Em dom., 22 de mar. de 2020 às 10:14, Israel Meireles Chrisostomo <
>> israelmchrisost...@gmail.com> escreveu:
>>
>>> Primeiramente obrigado pela solução.Mas Pedro, tenho uma pergunta :   o
>>> sr. é professor de Matemática?
>>>
>>> Em dom., 22 de mar. de 2020 às 01:34, Pedro José 
>>> escreveu:
>>>
 Bom dia!
 Dei uma mancada.
 O expoente de 3 é 3 e não 2.
 Retornando às classes mod 3.
 Ao último fator é côngruo à (n-1)*n
 Para n=3k aparece outro fator e 3^3|p(n), n=3k.
 n=3k+1, tenho (n-1)^2 e (n-1), 3^3|p(n), n=3k+1
 n=3k+2, tenho(n-2)^2 é (n+1)^2, 3^3|p(n), n=3k+2,
 Logo 3^3|p(n) para todo n inteiro.
 D>=2^6*3^3*5. Mas D<=2^6*3^3*5, então D=8640
 Desculpem-me pelo erro.
 Saudações,
 PJMS.



 Em sáb, 21 de mar de 2020 13:20, Pedro José 
 escreveu:

> Boa tarde!
> Nem carece método numérico.
> Para n=1 ou n=0 ou n=2 temos que qualquer inteiro divide o polinômio
> p(n)=(n-2)^2*(n-1)^2*n^2*(n+1)^2*(4n^2-4n-9)
>
> p(3)=8640
> p(4)=561600 então (p(3),p(4))=8640=2^6*3^3*5.
> Seja D o maior inteiro que divide p(n) para todo n inteiro, D<=8640
> Vamos pegar as classes de equivalência mod 4. Seja k um inteiro.
> Para 4k temos que n^2= 16k^2 e (n-2) é par logo (n-2)^2= 4s^2 com s
> inteiro. Logo 2^6 divide p(n) para qualquer n =4k.
> 4k+1. Pegando os fatores (4n-1)^2 e (4n+1)^2, teremos que 2^6 |p(n)
> para qualquer n=4k+1.
> 4k+2. Pegando (n-2)^2 e n^2, teremos que 2^6|p(n) para qualquer n=4k+2
> 4k+3, pegando os mesmos fatores de 4k+1, 2^6|p(n) para n=4k+3.
> Portanto 2^6|p(n) para qualquer inteiro
> Agora classes de equivalência mod 3
> 3k, pegando n^2, 3^2|p(n) para n=3k
> 3k+1, pegando (n-1)^2; 3^2| p(n), n=3k+1
> 3k+2, pegando (n-2)^2, 3^2| p(n), n=3k+2
> Daí 3^2|p(n) para qualquer n inteiro.
> Classes de equivalência mod 5.
> 5k, n^2 , 5 |p(n), n =5
> 5k +1, (n-1)^2, 5|p(n), n=5k+1
> 5k+2, (n-2)^2, 5|p(n), n=5k+2
> 5k+3, (4n^2-4n-9)=(100k^2-100k+15)
> 5|p(n), n=5k+3
> 5k+4, (n+1)^2, 5|p(n) , n=5k+4.
> Então 5|p(n) para todo inteiro
> D>=2^6*3^2×*5
> Mas D<=2^6*3^2*5, logo D=8640
>
> Saudações,
> PJMS
>
> Em sáb, 21 de mar de 2020 04:39, Pedro José 
> escreveu:
>
>> Bom dia!
>> Falta de novo, em seu questionamento, informar que n é inteiro ou
>> natural e colocar a condição para qualquer valor de n. Chamando o 
>> polinômio
>> de p(n)
>> Para n=0, 1 ou 2, qualquer inteiro divide.
>> Faria mdc(p(3),p(4))= A1
>> Se der "pequeno", com poucos fatores primos e expoentes pequenos.
>> Paro em A1, se não.
>> (p(5),A1)=A2 uso o mesmo critério de parar
>> (p(6),A2)=A3 até parar em:
>> Ai=(p(i+3),A(i-1)).
>> Aí faço o polinômio módfi^xi, onde fi é um fator primo de Aí e xi seu
>> expoente. verifico se para cada resíduon= 1, 2...fi^n-1 se P(n)=0 mod 
>> fi^si
>> Se falhar diminuto xi em 1 e repito o teste para todos resíduos de
>> fi^(xi-1)-1 até um dado xki em que todos os p(resíduos) foram 
>> equivalente a
>> zero módulo fi^xki ou quando fizer para o expoente 1  e não zerar para
>> todos resíduos de fi, quando o fator será descartado.
>> Depois repito para cada fator primo f e seu respectivo expoente.
>> Ao final D = Produtório de cada fator fi elevado ao expoente xki que
>> zerou p(n) mod fi^xki para todos os resíduos, descartando os fí em que 
>> xji
>> chegou a 1 e não atendeu ou considerando nesse caso xki=0.
>>
>> Mas resolveria por método numérico.
>> Depois poste sua solução.
>>
>> Saudações,
>> PJMS.
>>
>>
>>
>>
>> Em sex, 20 de mar de 2020 12:42, Israel Meireles Chrisostomo <
>> israelmchrisost...@gmail.com> escreveu:
>>
>>> Qual o maior inteiro que divide (n - 2)^2 (n - 1)^2 n^2 (n + 1)^2 (4
>>> n^2 - 4 n - 9))?
>>> Eu sei resolver esse problema com meu algoritmo, porém gostaria de
>>> saber como os colegas o resolveriam.
>>> --
>>> Israel Meireles Chrisostomo
>>>
>>> --
>>> Esta mensagem foi verificada pelo sistema de antivírus e

[obm-l] Re: [obm-l] Re: [obm-l] Re: [obm-l] Teoria dos números

2020-03-22 Por tôpico Pedro José
Boa tarde!
Perfeita a sua correção.
Quanto ao questionamento, nem tenho formação em matemática, meu sonho é
cursar no IMPA ao me aposentar. Sou pitaqueiro. Ouço um assunto que não
conheço, tento aprendê-lo. Na verdade, gosto de matemática. Talvez seja ela
o "Mundo das ideias", o mundo ideal, a qual Platão se referiu.
Saudações,
PJMS

Em dom, 22 de mar de 2020 12:25, Israel Meireles Chrisostomo <
israelmchrisost...@gmail.com> escreveu:

> Acho q tem uma ´pequena correção no seguinte passo "4k+1. Pegando os
> fatores (4n-1)^2 e (4n+1)^2, teremos que 2^6 |p(n) para qualquer n=4k+1."O
> correto seria "Para n=4k+1.Pegando os fatores (n-1)^2 e (n+1)^2"
>
> Em dom., 22 de mar. de 2020 às 10:14, Israel Meireles Chrisostomo <
> israelmchrisost...@gmail.com> escreveu:
>
>> Primeiramente obrigado pela solução.Mas Pedro, tenho uma pergunta :   o
>> sr. é professor de Matemática?
>>
>> Em dom., 22 de mar. de 2020 às 01:34, Pedro José 
>> escreveu:
>>
>>> Bom dia!
>>> Dei uma mancada.
>>> O expoente de 3 é 3 e não 2.
>>> Retornando às classes mod 3.
>>> Ao último fator é côngruo à (n-1)*n
>>> Para n=3k aparece outro fator e 3^3|p(n), n=3k.
>>> n=3k+1, tenho (n-1)^2 e (n-1), 3^3|p(n), n=3k+1
>>> n=3k+2, tenho(n-2)^2 é (n+1)^2, 3^3|p(n), n=3k+2,
>>> Logo 3^3|p(n) para todo n inteiro.
>>> D>=2^6*3^3*5. Mas D<=2^6*3^3*5, então D=8640
>>> Desculpem-me pelo erro.
>>> Saudações,
>>> PJMS.
>>>
>>>
>>>
>>> Em sáb, 21 de mar de 2020 13:20, Pedro José 
>>> escreveu:
>>>
 Boa tarde!
 Nem carece método numérico.
 Para n=1 ou n=0 ou n=2 temos que qualquer inteiro divide o polinômio
 p(n)=(n-2)^2*(n-1)^2*n^2*(n+1)^2*(4n^2-4n-9)

 p(3)=8640
 p(4)=561600 então (p(3),p(4))=8640=2^6*3^3*5.
 Seja D o maior inteiro que divide p(n) para todo n inteiro, D<=8640
 Vamos pegar as classes de equivalência mod 4. Seja k um inteiro.
 Para 4k temos que n^2= 16k^2 e (n-2) é par logo (n-2)^2= 4s^2 com s
 inteiro. Logo 2^6 divide p(n) para qualquer n =4k.
 4k+1. Pegando os fatores (4n-1)^2 e (4n+1)^2, teremos que 2^6 |p(n)
 para qualquer n=4k+1.
 4k+2. Pegando (n-2)^2 e n^2, teremos que 2^6|p(n) para qualquer n=4k+2
 4k+3, pegando os mesmos fatores de 4k+1, 2^6|p(n) para n=4k+3.
 Portanto 2^6|p(n) para qualquer inteiro
 Agora classes de equivalência mod 3
 3k, pegando n^2, 3^2|p(n) para n=3k
 3k+1, pegando (n-1)^2; 3^2| p(n), n=3k+1
 3k+2, pegando (n-2)^2, 3^2| p(n), n=3k+2
 Daí 3^2|p(n) para qualquer n inteiro.
 Classes de equivalência mod 5.
 5k, n^2 , 5 |p(n), n =5
 5k +1, (n-1)^2, 5|p(n), n=5k+1
 5k+2, (n-2)^2, 5|p(n), n=5k+2
 5k+3, (4n^2-4n-9)=(100k^2-100k+15)
 5|p(n), n=5k+3
 5k+4, (n+1)^2, 5|p(n) , n=5k+4.
 Então 5|p(n) para todo inteiro
 D>=2^6*3^2×*5
 Mas D<=2^6*3^2*5, logo D=8640

 Saudações,
 PJMS

 Em sáb, 21 de mar de 2020 04:39, Pedro José 
 escreveu:

> Bom dia!
> Falta de novo, em seu questionamento, informar que n é inteiro ou
> natural e colocar a condição para qualquer valor de n. Chamando o 
> polinômio
> de p(n)
> Para n=0, 1 ou 2, qualquer inteiro divide.
> Faria mdc(p(3),p(4))= A1
> Se der "pequeno", com poucos fatores primos e expoentes pequenos. Paro
> em A1, se não.
> (p(5),A1)=A2 uso o mesmo critério de parar
> (p(6),A2)=A3 até parar em:
> Ai=(p(i+3),A(i-1)).
> Aí faço o polinômio módfi^xi, onde fi é um fator primo de Aí e xi seu
> expoente. verifico se para cada resíduon= 1, 2...fi^n-1 se P(n)=0 mod 
> fi^si
> Se falhar diminuto xi em 1 e repito o teste para todos resíduos de
> fi^(xi-1)-1 até um dado xki em que todos os p(resíduos) foram equivalente 
> a
> zero módulo fi^xki ou quando fizer para o expoente 1  e não zerar para
> todos resíduos de fi, quando o fator será descartado.
> Depois repito para cada fator primo f e seu respectivo expoente.
> Ao final D = Produtório de cada fator fi elevado ao expoente xki que
> zerou p(n) mod fi^xki para todos os resíduos, descartando os fí em que xji
> chegou a 1 e não atendeu ou considerando nesse caso xki=0.
>
> Mas resolveria por método numérico.
> Depois poste sua solução.
>
> Saudações,
> PJMS.
>
>
>
>
> Em sex, 20 de mar de 2020 12:42, Israel Meireles Chrisostomo <
> israelmchrisost...@gmail.com> escreveu:
>
>> Qual o maior inteiro que divide (n - 2)^2 (n - 1)^2 n^2 (n + 1)^2 (4
>> n^2 - 4 n - 9))?
>> Eu sei resolver esse problema com meu algoritmo, porém gostaria de
>> saber como os colegas o resolveriam.
>> --
>> Israel Meireles Chrisostomo
>>
>> --
>> Esta mensagem foi verificada pelo sistema de antivírus e
>> acredita-se estar livre de perigo.
>
>
>>> --
>>> Esta mensagem foi verificada pelo sistema de antivírus e
>>> acredita-se estar livre de perigo.
>>
>>
>>
>> --
>> Israel Meireles Chrisostomo
>>
>
>
> --
> Israel Meireles 

[obm-l] Re: [obm-l] Re: [obm-l] Teoria dos números

2020-03-22 Por tôpico Israel Meireles Chrisostomo
Acho q tem uma ´pequena correção no seguinte passo "4k+1. Pegando os
fatores (4n-1)^2 e (4n+1)^2, teremos que 2^6 |p(n) para qualquer n=4k+1."O
correto seria "Para n=4k+1.Pegando os fatores (n-1)^2 e (n+1)^2"

Em dom., 22 de mar. de 2020 às 10:14, Israel Meireles Chrisostomo <
israelmchrisost...@gmail.com> escreveu:

> Primeiramente obrigado pela solução.Mas Pedro, tenho uma pergunta :   o
> sr. é professor de Matemática?
>
> Em dom., 22 de mar. de 2020 às 01:34, Pedro José 
> escreveu:
>
>> Bom dia!
>> Dei uma mancada.
>> O expoente de 3 é 3 e não 2.
>> Retornando às classes mod 3.
>> Ao último fator é côngruo à (n-1)*n
>> Para n=3k aparece outro fator e 3^3|p(n), n=3k.
>> n=3k+1, tenho (n-1)^2 e (n-1), 3^3|p(n), n=3k+1
>> n=3k+2, tenho(n-2)^2 é (n+1)^2, 3^3|p(n), n=3k+2,
>> Logo 3^3|p(n) para todo n inteiro.
>> D>=2^6*3^3*5. Mas D<=2^6*3^3*5, então D=8640
>> Desculpem-me pelo erro.
>> Saudações,
>> PJMS.
>>
>>
>>
>> Em sáb, 21 de mar de 2020 13:20, Pedro José 
>> escreveu:
>>
>>> Boa tarde!
>>> Nem carece método numérico.
>>> Para n=1 ou n=0 ou n=2 temos que qualquer inteiro divide o polinômio
>>> p(n)=(n-2)^2*(n-1)^2*n^2*(n+1)^2*(4n^2-4n-9)
>>>
>>> p(3)=8640
>>> p(4)=561600 então (p(3),p(4))=8640=2^6*3^3*5.
>>> Seja D o maior inteiro que divide p(n) para todo n inteiro, D<=8640
>>> Vamos pegar as classes de equivalência mod 4. Seja k um inteiro.
>>> Para 4k temos que n^2= 16k^2 e (n-2) é par logo (n-2)^2= 4s^2 com s
>>> inteiro. Logo 2^6 divide p(n) para qualquer n =4k.
>>> 4k+1. Pegando os fatores (4n-1)^2 e (4n+1)^2, teremos que 2^6 |p(n) para
>>> qualquer n=4k+1.
>>> 4k+2. Pegando (n-2)^2 e n^2, teremos que 2^6|p(n) para qualquer n=4k+2
>>> 4k+3, pegando os mesmos fatores de 4k+1, 2^6|p(n) para n=4k+3.
>>> Portanto 2^6|p(n) para qualquer inteiro
>>> Agora classes de equivalência mod 3
>>> 3k, pegando n^2, 3^2|p(n) para n=3k
>>> 3k+1, pegando (n-1)^2; 3^2| p(n), n=3k+1
>>> 3k+2, pegando (n-2)^2, 3^2| p(n), n=3k+2
>>> Daí 3^2|p(n) para qualquer n inteiro.
>>> Classes de equivalência mod 5.
>>> 5k, n^2 , 5 |p(n), n =5
>>> 5k +1, (n-1)^2, 5|p(n), n=5k+1
>>> 5k+2, (n-2)^2, 5|p(n), n=5k+2
>>> 5k+3, (4n^2-4n-9)=(100k^2-100k+15)
>>> 5|p(n), n=5k+3
>>> 5k+4, (n+1)^2, 5|p(n) , n=5k+4.
>>> Então 5|p(n) para todo inteiro
>>> D>=2^6*3^2×*5
>>> Mas D<=2^6*3^2*5, logo D=8640
>>>
>>> Saudações,
>>> PJMS
>>>
>>> Em sáb, 21 de mar de 2020 04:39, Pedro José 
>>> escreveu:
>>>
 Bom dia!
 Falta de novo, em seu questionamento, informar que n é inteiro ou
 natural e colocar a condição para qualquer valor de n. Chamando o polinômio
 de p(n)
 Para n=0, 1 ou 2, qualquer inteiro divide.
 Faria mdc(p(3),p(4))= A1
 Se der "pequeno", com poucos fatores primos e expoentes pequenos. Paro
 em A1, se não.
 (p(5),A1)=A2 uso o mesmo critério de parar
 (p(6),A2)=A3 até parar em:
 Ai=(p(i+3),A(i-1)).
 Aí faço o polinômio módfi^xi, onde fi é um fator primo de Aí e xi seu
 expoente. verifico se para cada resíduon= 1, 2...fi^n-1 se P(n)=0 mod fi^si
 Se falhar diminuto xi em 1 e repito o teste para todos resíduos de
 fi^(xi-1)-1 até um dado xki em que todos os p(resíduos) foram equivalente a
 zero módulo fi^xki ou quando fizer para o expoente 1  e não zerar para
 todos resíduos de fi, quando o fator será descartado.
 Depois repito para cada fator primo f e seu respectivo expoente.
 Ao final D = Produtório de cada fator fi elevado ao expoente xki que
 zerou p(n) mod fi^xki para todos os resíduos, descartando os fí em que xji
 chegou a 1 e não atendeu ou considerando nesse caso xki=0.

 Mas resolveria por método numérico.
 Depois poste sua solução.

 Saudações,
 PJMS.




 Em sex, 20 de mar de 2020 12:42, Israel Meireles Chrisostomo <
 israelmchrisost...@gmail.com> escreveu:

> Qual o maior inteiro que divide (n - 2)^2 (n - 1)^2 n^2 (n + 1)^2 (4
> n^2 - 4 n - 9))?
> Eu sei resolver esse problema com meu algoritmo, porém gostaria de
> saber como os colegas o resolveriam.
> --
> Israel Meireles Chrisostomo
>
> --
> Esta mensagem foi verificada pelo sistema de antivírus e
> acredita-se estar livre de perigo.


>> --
>> Esta mensagem foi verificada pelo sistema de antivírus e
>> acredita-se estar livre de perigo.
>
>
>
> --
> Israel Meireles Chrisostomo
>


-- 
Israel Meireles Chrisostomo

-- 
Esta mensagem foi verificada pelo sistema de antiv�rus e
 acredita-se estar livre de perigo.



[obm-l] Re: [obm-l] Re: [obm-l] Teoria dos números

2020-03-22 Por tôpico Israel Meireles Chrisostomo
Primeiramente obrigado pela solução.Mas Pedro, tenho uma pergunta :   o sr.
é professor de Matemática?

Em dom., 22 de mar. de 2020 às 01:34, Pedro José 
escreveu:

> Bom dia!
> Dei uma mancada.
> O expoente de 3 é 3 e não 2.
> Retornando às classes mod 3.
> Ao último fator é côngruo à (n-1)*n
> Para n=3k aparece outro fator e 3^3|p(n), n=3k.
> n=3k+1, tenho (n-1)^2 e (n-1), 3^3|p(n), n=3k+1
> n=3k+2, tenho(n-2)^2 é (n+1)^2, 3^3|p(n), n=3k+2,
> Logo 3^3|p(n) para todo n inteiro.
> D>=2^6*3^3*5. Mas D<=2^6*3^3*5, então D=8640
> Desculpem-me pelo erro.
> Saudações,
> PJMS.
>
>
>
> Em sáb, 21 de mar de 2020 13:20, Pedro José 
> escreveu:
>
>> Boa tarde!
>> Nem carece método numérico.
>> Para n=1 ou n=0 ou n=2 temos que qualquer inteiro divide o polinômio
>> p(n)=(n-2)^2*(n-1)^2*n^2*(n+1)^2*(4n^2-4n-9)
>>
>> p(3)=8640
>> p(4)=561600 então (p(3),p(4))=8640=2^6*3^3*5.
>> Seja D o maior inteiro que divide p(n) para todo n inteiro, D<=8640
>> Vamos pegar as classes de equivalência mod 4. Seja k um inteiro.
>> Para 4k temos que n^2= 16k^2 e (n-2) é par logo (n-2)^2= 4s^2 com s
>> inteiro. Logo 2^6 divide p(n) para qualquer n =4k.
>> 4k+1. Pegando os fatores (4n-1)^2 e (4n+1)^2, teremos que 2^6 |p(n) para
>> qualquer n=4k+1.
>> 4k+2. Pegando (n-2)^2 e n^2, teremos que 2^6|p(n) para qualquer n=4k+2
>> 4k+3, pegando os mesmos fatores de 4k+1, 2^6|p(n) para n=4k+3.
>> Portanto 2^6|p(n) para qualquer inteiro
>> Agora classes de equivalência mod 3
>> 3k, pegando n^2, 3^2|p(n) para n=3k
>> 3k+1, pegando (n-1)^2; 3^2| p(n), n=3k+1
>> 3k+2, pegando (n-2)^2, 3^2| p(n), n=3k+2
>> Daí 3^2|p(n) para qualquer n inteiro.
>> Classes de equivalência mod 5.
>> 5k, n^2 , 5 |p(n), n =5
>> 5k +1, (n-1)^2, 5|p(n), n=5k+1
>> 5k+2, (n-2)^2, 5|p(n), n=5k+2
>> 5k+3, (4n^2-4n-9)=(100k^2-100k+15)
>> 5|p(n), n=5k+3
>> 5k+4, (n+1)^2, 5|p(n) , n=5k+4.
>> Então 5|p(n) para todo inteiro
>> D>=2^6*3^2×*5
>> Mas D<=2^6*3^2*5, logo D=8640
>>
>> Saudações,
>> PJMS
>>
>> Em sáb, 21 de mar de 2020 04:39, Pedro José 
>> escreveu:
>>
>>> Bom dia!
>>> Falta de novo, em seu questionamento, informar que n é inteiro ou
>>> natural e colocar a condição para qualquer valor de n. Chamando o polinômio
>>> de p(n)
>>> Para n=0, 1 ou 2, qualquer inteiro divide.
>>> Faria mdc(p(3),p(4))= A1
>>> Se der "pequeno", com poucos fatores primos e expoentes pequenos. Paro
>>> em A1, se não.
>>> (p(5),A1)=A2 uso o mesmo critério de parar
>>> (p(6),A2)=A3 até parar em:
>>> Ai=(p(i+3),A(i-1)).
>>> Aí faço o polinômio módfi^xi, onde fi é um fator primo de Aí e xi seu
>>> expoente. verifico se para cada resíduon= 1, 2...fi^n-1 se P(n)=0 mod fi^si
>>> Se falhar diminuto xi em 1 e repito o teste para todos resíduos de
>>> fi^(xi-1)-1 até um dado xki em que todos os p(resíduos) foram equivalente a
>>> zero módulo fi^xki ou quando fizer para o expoente 1  e não zerar para
>>> todos resíduos de fi, quando o fator será descartado.
>>> Depois repito para cada fator primo f e seu respectivo expoente.
>>> Ao final D = Produtório de cada fator fi elevado ao expoente xki que
>>> zerou p(n) mod fi^xki para todos os resíduos, descartando os fí em que xji
>>> chegou a 1 e não atendeu ou considerando nesse caso xki=0.
>>>
>>> Mas resolveria por método numérico.
>>> Depois poste sua solução.
>>>
>>> Saudações,
>>> PJMS.
>>>
>>>
>>>
>>>
>>> Em sex, 20 de mar de 2020 12:42, Israel Meireles Chrisostomo <
>>> israelmchrisost...@gmail.com> escreveu:
>>>
 Qual o maior inteiro que divide (n - 2)^2 (n - 1)^2 n^2 (n + 1)^2 (4
 n^2 - 4 n - 9))?
 Eu sei resolver esse problema com meu algoritmo, porém gostaria de
 saber como os colegas o resolveriam.
 --
 Israel Meireles Chrisostomo

 --
 Esta mensagem foi verificada pelo sistema de antivírus e
 acredita-se estar livre de perigo.
>>>
>>>
> --
> Esta mensagem foi verificada pelo sistema de antivírus e
> acredita-se estar livre de perigo.



-- 
Israel Meireles Chrisostomo

-- 
Esta mensagem foi verificada pelo sistema de antiv�rus e
 acredita-se estar livre de perigo.



[obm-l] Re: [obm-l] Teoria dos números

2020-03-21 Por tôpico Pedro José
Bom dia!
Dei uma mancada.
O expoente de 3 é 3 e não 2.
Retornando às classes mod 3.
Ao último fator é côngruo à (n-1)*n
Para n=3k aparece outro fator e 3^3|p(n), n=3k.
n=3k+1, tenho (n-1)^2 e (n-1), 3^3|p(n), n=3k+1
n=3k+2, tenho(n-2)^2 é (n+1)^2, 3^3|p(n), n=3k+2,
Logo 3^3|p(n) para todo n inteiro.
D>=2^6*3^3*5. Mas D<=2^6*3^3*5, então D=8640
Desculpem-me pelo erro.
Saudações,
PJMS.



Em sáb, 21 de mar de 2020 13:20, Pedro José  escreveu:

> Boa tarde!
> Nem carece método numérico.
> Para n=1 ou n=0 ou n=2 temos que qualquer inteiro divide o polinômio
> p(n)=(n-2)^2*(n-1)^2*n^2*(n+1)^2*(4n^2-4n-9)
>
> p(3)=8640
> p(4)=561600 então (p(3),p(4))=8640=2^6*3^3*5.
> Seja D o maior inteiro que divide p(n) para todo n inteiro, D<=8640
> Vamos pegar as classes de equivalência mod 4. Seja k um inteiro.
> Para 4k temos que n^2= 16k^2 e (n-2) é par logo (n-2)^2= 4s^2 com s
> inteiro. Logo 2^6 divide p(n) para qualquer n =4k.
> 4k+1. Pegando os fatores (4n-1)^2 e (4n+1)^2, teremos que 2^6 |p(n) para
> qualquer n=4k+1.
> 4k+2. Pegando (n-2)^2 e n^2, teremos que 2^6|p(n) para qualquer n=4k+2
> 4k+3, pegando os mesmos fatores de 4k+1, 2^6|p(n) para n=4k+3.
> Portanto 2^6|p(n) para qualquer inteiro
> Agora classes de equivalência mod 3
> 3k, pegando n^2, 3^2|p(n) para n=3k
> 3k+1, pegando (n-1)^2; 3^2| p(n), n=3k+1
> 3k+2, pegando (n-2)^2, 3^2| p(n), n=3k+2
> Daí 3^2|p(n) para qualquer n inteiro.
> Classes de equivalência mod 5.
> 5k, n^2 , 5 |p(n), n =5
> 5k +1, (n-1)^2, 5|p(n), n=5k+1
> 5k+2, (n-2)^2, 5|p(n), n=5k+2
> 5k+3, (4n^2-4n-9)=(100k^2-100k+15)
> 5|p(n), n=5k+3
> 5k+4, (n+1)^2, 5|p(n) , n=5k+4.
> Então 5|p(n) para todo inteiro
> D>=2^6*3^2×*5
> Mas D<=2^6*3^2*5, logo D=8640
>
> Saudações,
> PJMS
>
> Em sáb, 21 de mar de 2020 04:39, Pedro José 
> escreveu:
>
>> Bom dia!
>> Falta de novo, em seu questionamento, informar que n é inteiro ou natural
>> e colocar a condição para qualquer valor de n. Chamando o polinômio de p(n)
>> Para n=0, 1 ou 2, qualquer inteiro divide.
>> Faria mdc(p(3),p(4))= A1
>> Se der "pequeno", com poucos fatores primos e expoentes pequenos. Paro em
>> A1, se não.
>> (p(5),A1)=A2 uso o mesmo critério de parar
>> (p(6),A2)=A3 até parar em:
>> Ai=(p(i+3),A(i-1)).
>> Aí faço o polinômio módfi^xi, onde fi é um fator primo de Aí e xi seu
>> expoente. verifico se para cada resíduon= 1, 2...fi^n-1 se P(n)=0 mod fi^si
>> Se falhar diminuto xi em 1 e repito o teste para todos resíduos de
>> fi^(xi-1)-1 até um dado xki em que todos os p(resíduos) foram equivalente a
>> zero módulo fi^xki ou quando fizer para o expoente 1  e não zerar para
>> todos resíduos de fi, quando o fator será descartado.
>> Depois repito para cada fator primo f e seu respectivo expoente.
>> Ao final D = Produtório de cada fator fi elevado ao expoente xki que
>> zerou p(n) mod fi^xki para todos os resíduos, descartando os fí em que xji
>> chegou a 1 e não atendeu ou considerando nesse caso xki=0.
>>
>> Mas resolveria por método numérico.
>> Depois poste sua solução.
>>
>> Saudações,
>> PJMS.
>>
>>
>>
>>
>> Em sex, 20 de mar de 2020 12:42, Israel Meireles Chrisostomo <
>> israelmchrisost...@gmail.com> escreveu:
>>
>>> Qual o maior inteiro que divide (n - 2)^2 (n - 1)^2 n^2 (n + 1)^2 (4 n^2
>>> - 4 n - 9))?
>>> Eu sei resolver esse problema com meu algoritmo, porém gostaria de saber
>>> como os colegas o resolveriam.
>>> --
>>> Israel Meireles Chrisostomo
>>>
>>> --
>>> Esta mensagem foi verificada pelo sistema de antivírus e
>>> acredita-se estar livre de perigo.
>>
>>

-- 
Esta mensagem foi verificada pelo sistema de antiv�rus e
 acredita-se estar livre de perigo.



[obm-l] Re: [obm-l] Teoria dos números

2020-03-21 Por tôpico Pedro José
Boa tarde!
Nem carece método numérico.
Para n=1 ou n=0 ou n=2 temos que qualquer inteiro divide o polinômio
p(n)=(n-2)^2*(n-1)^2*n^2*(n+1)^2*(4n^2-4n-9)

p(3)=8640
p(4)=561600 então (p(3),p(4))=8640=2^6*3^3*5.
Seja D o maior inteiro que divide p(n) para todo n inteiro, D<=8640
Vamos pegar as classes de equivalência mod 4. Seja k um inteiro.
Para 4k temos que n^2= 16k^2 e (n-2) é par logo (n-2)^2= 4s^2 com s
inteiro. Logo 2^6 divide p(n) para qualquer n =4k.
4k+1. Pegando os fatores (4n-1)^2 e (4n+1)^2, teremos que 2^6 |p(n) para
qualquer n=4k+1.
4k+2. Pegando (n-2)^2 e n^2, teremos que 2^6|p(n) para qualquer n=4k+2
4k+3, pegando os mesmos fatores de 4k+1, 2^6|p(n) para n=4k+3.
Portanto 2^6|p(n) para qualquer inteiro
Agora classes de equivalência mod 3
3k, pegando n^2, 3^2|p(n) para n=3k
3k+1, pegando (n-1)^2; 3^2| p(n), n=3k+1
3k+2, pegando (n-2)^2, 3^2| p(n), n=3k+2
Daí 3^2|p(n) para qualquer n inteiro.
Classes de equivalência mod 5.
5k, n^2 , 5 |p(n), n =5
5k +1, (n-1)^2, 5|p(n), n=5k+1
5k+2, (n-2)^2, 5|p(n), n=5k+2
5k+3, (4n^2-4n-9)=(100k^2-100k+15)
5|p(n), n=5k+3
5k+4, (n+1)^2, 5|p(n) , n=5k+4.
Então 5|p(n) para todo inteiro
D>=2^6*3^2×*5
Mas D<=2^6*3^2*5, logo D=8640

Saudações,
PJMS

Em sáb, 21 de mar de 2020 04:39, Pedro José  escreveu:

> Bom dia!
> Falta de novo, em seu questionamento, informar que n é inteiro ou natural
> e colocar a condição para qualquer valor de n. Chamando o polinômio de p(n)
> Para n=0, 1 ou 2, qualquer inteiro divide.
> Faria mdc(p(3),p(4))= A1
> Se der "pequeno", com poucos fatores primos e expoentes pequenos. Paro em
> A1, se não.
> (p(5),A1)=A2 uso o mesmo critério de parar
> (p(6),A2)=A3 até parar em:
> Ai=(p(i+3),A(i-1)).
> Aí faço o polinômio módfi^xi, onde fi é um fator primo de Aí e xi seu
> expoente. verifico se para cada resíduon= 1, 2...fi^n-1 se P(n)=0 mod fi^si
> Se falhar diminuto xi em 1 e repito o teste para todos resíduos de
> fi^(xi-1)-1 até um dado xki em que todos os p(resíduos) foram equivalente a
> zero módulo fi^xki ou quando fizer para o expoente 1  e não zerar para
> todos resíduos de fi, quando o fator será descartado.
> Depois repito para cada fator primo f e seu respectivo expoente.
> Ao final D = Produtório de cada fator fi elevado ao expoente xki que zerou
> p(n) mod fi^xki para todos os resíduos, descartando os fí em que xji chegou
> a 1 e não atendeu ou considerando nesse caso xki=0.
>
> Mas resolveria por método numérico.
> Depois poste sua solução.
>
> Saudações,
> PJMS.
>
>
>
>
> Em sex, 20 de mar de 2020 12:42, Israel Meireles Chrisostomo <
> israelmchrisost...@gmail.com> escreveu:
>
>> Qual o maior inteiro que divide (n - 2)^2 (n - 1)^2 n^2 (n + 1)^2 (4 n^2
>> - 4 n - 9))?
>> Eu sei resolver esse problema com meu algoritmo, porém gostaria de saber
>> como os colegas o resolveriam.
>> --
>> Israel Meireles Chrisostomo
>>
>> --
>> Esta mensagem foi verificada pelo sistema de antivírus e
>> acredita-se estar livre de perigo.
>
>

-- 
Esta mensagem foi verificada pelo sistema de antiv�rus e
 acredita-se estar livre de perigo.



[obm-l] Re: [obm-l] Teoria dos números

2020-03-21 Por tôpico Pedro José
Bom dia!
Falta de novo, em seu questionamento, informar que n é inteiro ou natural e
colocar a condição para qualquer valor de n. Chamando o polinômio de p(n)
Para n=0, 1 ou 2, qualquer inteiro divide.
Faria mdc(p(3),p(4))= A1
Se der "pequeno", com poucos fatores primos e expoentes pequenos. Paro em
A1, se não.
(p(5),A1)=A2 uso o mesmo critério de parar
(p(6),A2)=A3 até parar em:
Ai=(p(i+3),A(i-1)).
Aí faço o polinômio módfi^xi, onde fi é um fator primo de Aí e xi seu
expoente. verifico se para cada resíduon= 1, 2...fi^n-1 se P(n)=0 mod fi^si
Se falhar diminuto xi em 1 e repito o teste para todos resíduos de
fi^(xi-1)-1 até um dado xki em que todos os p(resíduos) foram equivalente a
zero módulo fi^xki ou quando fizer para o expoente 1  e não zerar para
todos resíduos de fi, quando o fator será descartado.
Depois repito para cada fator primo f e seu respectivo expoente.
Ao final D = Produtório de cada fator fi elevado ao expoente xki que zerou
p(n) mod fi^xki para todos os resíduos, descartando os fí em que xji chegou
a 1 e não atendeu ou considerando nesse caso xki=0.

Mas resolveria por método numérico.
Depois poste sua solução.

Saudações,
PJMS.




Em sex, 20 de mar de 2020 12:42, Israel Meireles Chrisostomo <
israelmchrisost...@gmail.com> escreveu:

> Qual o maior inteiro que divide (n - 2)^2 (n - 1)^2 n^2 (n + 1)^2 (4 n^2 -
> 4 n - 9))?
> Eu sei resolver esse problema com meu algoritmo, porém gostaria de saber
> como os colegas o resolveriam.
> --
> Israel Meireles Chrisostomo
>
> --
> Esta mensagem foi verificada pelo sistema de antivírus e
> acredita-se estar livre de perigo.

-- 
Esta mensagem foi verificada pelo sistema de antiv�rus e
 acredita-se estar livre de perigo.



[obm-l] Re: [obm-l] Re: [obm-l] Re: [obm-l] Re: [obm-l] Re: [obm-l] Re: [obm-l] Teoria dos números

2020-03-17 Por tôpico Pedro José
Boa noite!
Aí, como dizia minha falecida vó, são outros quinhentos.
Como nas propostas anteriores n era natural. Vamos seguir nessa linha, se
não for reformule o problema.
Seja f(n)=   n (427 - 90n - 70n^2 + 45n^3 + 18n^4)
f(0)=0 qualquer natural divide, portanto, é indiferente.
f(1)= 330
f(2)= 1230
É fácil verificar que mdc(330,1230)=30 então D<=30, onde D é o máximo
inteiro que divide f(n) para todo n natural.
f(n) = 7n +5n^4 + 8 n^5 mod 10.
f(0)=0 mod10
f(1)= 20 = 0 mod10
f(2)= 350= 0 mod10
f(3)= 2370 = 0 mod10
f(4)= 9500 = 0 mod10
f(5)= 28160 = 0 mod10
f(6)=68730 = 0 mod10
f(7)=146510 = 0 mod10
f(8)=282680 = 0 mod10
f(9)=505260 = 0 mod10
logo 10 | f(n) para qualquer n natural.

f(n) = n -n^3 mod 3
f(0) = 0 mod 3
f(1) = 0 mod 3
f(2)= -6 = 0 mod 3
logo 3| f(n) para todo n natural
então D = 30.

Saudações,
PJMS



Em ter., 17 de mar. de 2020 às 11:57, Israel Meireles Chrisostomo <
israelmchrisost...@gmail.com> escreveu:

> Sim é isso q eu quis dizer
>
> Em ter, 17 de mar de 2020 11:12, Carlos Gustavo Tamm de Araujo Moreira <
> g...@impa.br> escreveu:
>
>> Acho que a pergunta deve ser qual é o maior inteiro positivo que divide
>> essa expressão para todo valor de n ao mesmo tempo.
>>
>> On Tue, Mar 17, 2020 at 6:58 AM Pedro José  wrote:
>>
>>> Bom dia!
>>> Se você considerar a expressão n(427-90n-70n^2+45n^3+18n^4)
>>> D=|n(427-90n-70n^2+45n^3+18n^4)|
>>> Por exemplo, n=1
>>> D=330.
>>> Agora se liberar n para variar D tende a oo.
>>>
>>> Se n for raiz da expressão, também tende a oi, pois qualquer inteiro
>>> divide 0.
>>>
>>>
>>> Em seg, 16 de mar de 2020 22:16, Israel Meireles Chrisostomo <
>>> israelmchrisost...@gmail.com> escreveu:
>>>
 não entendi

 Em seg., 16 de mar. de 2020 às 22:01, Pedro José 
 escreveu:

> Para um dado n é o módulo do valor da expressão.
>
> Em seg, 16 de mar de 2020 21:49, Pedro José 
> escreveu:
>
>> Boa noite!
>> O módulo dessa expressão tende a oo. Não existe máximo.
>> Saudações,
>> PJMS
>>
>> Em seg, 16 de mar de 2020 20:36, Israel Meireles Chrisostomo <
>> israelmchrisost...@gmail.com> escreveu:
>>
>>> Qual é o maior inteiro que divide  n (427 - 90n - 70n^2 + 45n^3 +
>>> 18n^4)?
>>>
>>> --
>>> Israel Meireles Chrisostomo
>>>
>>> --
>>> Esta mensagem foi verificada pelo sistema de antivírus e
>>> acredita-se estar livre de perigo.
>>
>>
> --
> Esta mensagem foi verificada pelo sistema de antivírus e
> acredita-se estar livre de perigo.



 --
 Israel Meireles Chrisostomo

 --
 Esta mensagem foi verificada pelo sistema de antivírus e
 acredita-se estar livre de perigo.
>>>
>>>
>>> --
>>> Esta mensagem foi verificada pelo sistema de antivírus e
>>> acredita-se estar livre de perigo.
>>
>>
>> --
>> Esta mensagem foi verificada pelo sistema de antivírus e
>> acredita-se estar livre de perigo.
>
>
> --
> Esta mensagem foi verificada pelo sistema de antivírus e
> acredita-se estar livre de perigo.

-- 
Esta mensagem foi verificada pelo sistema de antiv�rus e
 acredita-se estar livre de perigo.



[obm-l] Re: [obm-l] Re: [obm-l] Re: [obm-l] Re: [obm-l] Re: [obm-l] Teoria dos números

2020-03-17 Por tôpico Israel Meireles Chrisostomo
Sim é isso q eu quis dizer

Em ter, 17 de mar de 2020 11:12, Carlos Gustavo Tamm de Araujo Moreira <
g...@impa.br> escreveu:

> Acho que a pergunta deve ser qual é o maior inteiro positivo que divide
> essa expressão para todo valor de n ao mesmo tempo.
>
> On Tue, Mar 17, 2020 at 6:58 AM Pedro José  wrote:
>
>> Bom dia!
>> Se você considerar a expressão n(427-90n-70n^2+45n^3+18n^4)
>> D=|n(427-90n-70n^2+45n^3+18n^4)|
>> Por exemplo, n=1
>> D=330.
>> Agora se liberar n para variar D tende a oo.
>>
>> Se n for raiz da expressão, também tende a oi, pois qualquer inteiro
>> divide 0.
>>
>>
>> Em seg, 16 de mar de 2020 22:16, Israel Meireles Chrisostomo <
>> israelmchrisost...@gmail.com> escreveu:
>>
>>> não entendi
>>>
>>> Em seg., 16 de mar. de 2020 às 22:01, Pedro José 
>>> escreveu:
>>>
 Para um dado n é o módulo do valor da expressão.

 Em seg, 16 de mar de 2020 21:49, Pedro José 
 escreveu:

> Boa noite!
> O módulo dessa expressão tende a oo. Não existe máximo.
> Saudações,
> PJMS
>
> Em seg, 16 de mar de 2020 20:36, Israel Meireles Chrisostomo <
> israelmchrisost...@gmail.com> escreveu:
>
>> Qual é o maior inteiro que divide  n (427 - 90n - 70n^2 + 45n^3 +
>> 18n^4)?
>>
>> --
>> Israel Meireles Chrisostomo
>>
>> --
>> Esta mensagem foi verificada pelo sistema de antivírus e
>> acredita-se estar livre de perigo.
>
>
 --
 Esta mensagem foi verificada pelo sistema de antivírus e
 acredita-se estar livre de perigo.
>>>
>>>
>>>
>>> --
>>> Israel Meireles Chrisostomo
>>>
>>> --
>>> Esta mensagem foi verificada pelo sistema de antivírus e
>>> acredita-se estar livre de perigo.
>>
>>
>> --
>> Esta mensagem foi verificada pelo sistema de antivírus e
>> acredita-se estar livre de perigo.
>
>
> --
> Esta mensagem foi verificada pelo sistema de antivírus e
> acredita-se estar livre de perigo.

-- 
Esta mensagem foi verificada pelo sistema de antiv�rus e
 acredita-se estar livre de perigo.



[obm-l] Re: [obm-l] Re: [obm-l] Re: [obm-l] Re: [obm-l] Teoria dos números

2020-03-17 Por tôpico Carlos Gustavo Tamm de Araujo Moreira
Acho que a pergunta deve ser qual é o maior inteiro positivo que divide
essa expressão para todo valor de n ao mesmo tempo.

On Tue, Mar 17, 2020 at 6:58 AM Pedro José  wrote:

> Bom dia!
> Se você considerar a expressão n(427-90n-70n^2+45n^3+18n^4)
> D=|n(427-90n-70n^2+45n^3+18n^4)|
> Por exemplo, n=1
> D=330.
> Agora se liberar n para variar D tende a oo.
>
> Se n for raiz da expressão, também tende a oi, pois qualquer inteiro
> divide 0.
>
>
> Em seg, 16 de mar de 2020 22:16, Israel Meireles Chrisostomo <
> israelmchrisost...@gmail.com> escreveu:
>
>> não entendi
>>
>> Em seg., 16 de mar. de 2020 às 22:01, Pedro José 
>> escreveu:
>>
>>> Para um dado n é o módulo do valor da expressão.
>>>
>>> Em seg, 16 de mar de 2020 21:49, Pedro José 
>>> escreveu:
>>>
 Boa noite!
 O módulo dessa expressão tende a oo. Não existe máximo.
 Saudações,
 PJMS

 Em seg, 16 de mar de 2020 20:36, Israel Meireles Chrisostomo <
 israelmchrisost...@gmail.com> escreveu:

> Qual é o maior inteiro que divide  n (427 - 90n - 70n^2 + 45n^3 +
> 18n^4)?
>
> --
> Israel Meireles Chrisostomo
>
> --
> Esta mensagem foi verificada pelo sistema de antivírus e
> acredita-se estar livre de perigo.


>>> --
>>> Esta mensagem foi verificada pelo sistema de antivírus e
>>> acredita-se estar livre de perigo.
>>
>>
>>
>> --
>> Israel Meireles Chrisostomo
>>
>> --
>> Esta mensagem foi verificada pelo sistema de antivírus e
>> acredita-se estar livre de perigo.
>
>
> --
> Esta mensagem foi verificada pelo sistema de antivírus e
> acredita-se estar livre de perigo.

-- 
Esta mensagem foi verificada pelo sistema de antiv�rus e
 acredita-se estar livre de perigo.



[obm-l] Re: [obm-l] Re: [obm-l] Re: [obm-l] Teoria dos números

2020-03-17 Por tôpico Pedro José
Bom dia!
Se você considerar a expressão n(427-90n-70n^2+45n^3+18n^4)
D=|n(427-90n-70n^2+45n^3+18n^4)|
Por exemplo, n=1
D=330.
Agora se liberar n para variar D tende a oo.

Se n for raiz da expressão, também tende a oi, pois qualquer inteiro divide
0.


Em seg, 16 de mar de 2020 22:16, Israel Meireles Chrisostomo <
israelmchrisost...@gmail.com> escreveu:

> não entendi
>
> Em seg., 16 de mar. de 2020 às 22:01, Pedro José 
> escreveu:
>
>> Para um dado n é o módulo do valor da expressão.
>>
>> Em seg, 16 de mar de 2020 21:49, Pedro José 
>> escreveu:
>>
>>> Boa noite!
>>> O módulo dessa expressão tende a oo. Não existe máximo.
>>> Saudações,
>>> PJMS
>>>
>>> Em seg, 16 de mar de 2020 20:36, Israel Meireles Chrisostomo <
>>> israelmchrisost...@gmail.com> escreveu:
>>>
 Qual é o maior inteiro que divide  n (427 - 90n - 70n^2 + 45n^3 +
 18n^4)?

 --
 Israel Meireles Chrisostomo

 --
 Esta mensagem foi verificada pelo sistema de antivírus e
 acredita-se estar livre de perigo.
>>>
>>>
>> --
>> Esta mensagem foi verificada pelo sistema de antivírus e
>> acredita-se estar livre de perigo.
>
>
>
> --
> Israel Meireles Chrisostomo
>
> --
> Esta mensagem foi verificada pelo sistema de antivírus e
> acredita-se estar livre de perigo.

-- 
Esta mensagem foi verificada pelo sistema de antiv�rus e
 acredita-se estar livre de perigo.



[obm-l] Re: [obm-l] Re: [obm-l] Teoria dos números

2020-03-16 Por tôpico Israel Meireles Chrisostomo
não entendi

Em seg., 16 de mar. de 2020 às 22:01, Pedro José 
escreveu:

> Para um dado n é o módulo do valor da expressão.
>
> Em seg, 16 de mar de 2020 21:49, Pedro José 
> escreveu:
>
>> Boa noite!
>> O módulo dessa expressão tende a oo. Não existe máximo.
>> Saudações,
>> PJMS
>>
>> Em seg, 16 de mar de 2020 20:36, Israel Meireles Chrisostomo <
>> israelmchrisost...@gmail.com> escreveu:
>>
>>> Qual é o maior inteiro que divide  n (427 - 90n - 70n^2 + 45n^3 +
>>> 18n^4)?
>>>
>>> --
>>> Israel Meireles Chrisostomo
>>>
>>> --
>>> Esta mensagem foi verificada pelo sistema de antivírus e
>>> acredita-se estar livre de perigo.
>>
>>
> --
> Esta mensagem foi verificada pelo sistema de antivírus e
> acredita-se estar livre de perigo.



-- 
Israel Meireles Chrisostomo

-- 
Esta mensagem foi verificada pelo sistema de antiv�rus e
 acredita-se estar livre de perigo.



[obm-l] Re: [obm-l] Teoria dos números

2020-03-16 Por tôpico Pedro José
Boa noite!
O módulo dessa expressão tende a oo. Não existe máximo.
Saudações,
PJMS

Em seg, 16 de mar de 2020 20:36, Israel Meireles Chrisostomo <
israelmchrisost...@gmail.com> escreveu:

> Qual é o maior inteiro que divide  n (427 - 90n - 70n^2 + 45n^3 + 18n^4)?
>
> --
> Israel Meireles Chrisostomo
>
> --
> Esta mensagem foi verificada pelo sistema de antivírus e
> acredita-se estar livre de perigo.

-- 
Esta mensagem foi verificada pelo sistema de antiv�rus e
 acredita-se estar livre de perigo.



[obm-l] Re: [obm-l] Teoria dos números

2020-03-16 Por tôpico Pedro José
Para um dado n é o módulo do valor da expressão.

Em seg, 16 de mar de 2020 21:49, Pedro José  escreveu:

> Boa noite!
> O módulo dessa expressão tende a oo. Não existe máximo.
> Saudações,
> PJMS
>
> Em seg, 16 de mar de 2020 20:36, Israel Meireles Chrisostomo <
> israelmchrisost...@gmail.com> escreveu:
>
>> Qual é o maior inteiro que divide  n (427 - 90n - 70n^2 + 45n^3 + 18n^4)?
>>
>> --
>> Israel Meireles Chrisostomo
>>
>> --
>> Esta mensagem foi verificada pelo sistema de antivírus e
>> acredita-se estar livre de perigo.
>
>

-- 
Esta mensagem foi verificada pelo sistema de antiv�rus e
 acredita-se estar livre de perigo.



[obm-l] Re: [obm-l] Re: [obm-l] Teoria dos números

2020-03-16 Por tôpico Israel Meireles Chrisostomo
muito obrigado

Em seg., 16 de mar. de 2020 às 13:29, Pedro José 
escreveu:

> Boa tarde!
>
> Difícil generalizar. Mas consegui dois valores que não zeram a expressão
> (soluções triviais), a duras penas, n=32 e n=43.
> Vou continuar pensando no assunto.
>
> Saudações,
> PJMS
>
>
> Em dom., 15 de mar. de 2020 às 18:48, Pedro José 
> escreveu:
>
>> Boa tarde!
>> Faltou um contraexemplo.
>> n=5
>> 3^2*4^2*5^2*6^2*71 não é múltiplo de 11 nem de 37.
>>
>> Saudações,
>> PJMS
>>
>> Em sáb, 14 de mar de 2020 19:47, Pedro José 
>> escreveu:
>>
>>> Boa noite!
>>> Creio que a pergunta correta seria, para que valores de n natural...
>>> 8140=2^2*5*11*37. Então a solução só se dará para um subconjunto dos
>>> naturais diferente de|N.
>>>
>>> Saudações,
>>> PJMS
>>>
>>>
>>> Em sex, 13 de mar de 2020 20:05, Israel Meireles Chrisostomo <
>>> israelmchrisost...@gmail.com> escreveu:
>>>
   Dado n natural verifique se a expressão
  (n − 2)² (n − 1)²n² (n + 1)² (4n²− 4n − 9)/8140 é um número inteiro

 --
 Israel Meireles Chrisostomo

 --
 Esta mensagem foi verificada pelo sistema de antivírus e
 acredita-se estar livre de perigo.
>>>
>>>
> --
> Esta mensagem foi verificada pelo sistema de antivírus e
> acredita-se estar livre de perigo.



-- 
Israel Meireles Chrisostomo

-- 
Esta mensagem foi verificada pelo sistema de antiv�rus e
 acredita-se estar livre de perigo.



[obm-l] Re: [obm-l] Teoria dos números

2020-03-16 Por tôpico Pedro José
Boa tarde!

Difícil generalizar. Mas consegui dois valores que não zeram a expressão
(soluções triviais), a duras penas, n=32 e n=43.
Vou continuar pensando no assunto.

Saudações,
PJMS


Em dom., 15 de mar. de 2020 às 18:48, Pedro José 
escreveu:

> Boa tarde!
> Faltou um contraexemplo.
> n=5
> 3^2*4^2*5^2*6^2*71 não é múltiplo de 11 nem de 37.
>
> Saudações,
> PJMS
>
> Em sáb, 14 de mar de 2020 19:47, Pedro José 
> escreveu:
>
>> Boa noite!
>> Creio que a pergunta correta seria, para que valores de n natural...
>> 8140=2^2*5*11*37. Então a solução só se dará para um subconjunto dos
>> naturais diferente de|N.
>>
>> Saudações,
>> PJMS
>>
>>
>> Em sex, 13 de mar de 2020 20:05, Israel Meireles Chrisostomo <
>> israelmchrisost...@gmail.com> escreveu:
>>
>>>   Dado n natural verifique se a expressão
>>>  (n − 2)² (n − 1)²n² (n + 1)² (4n²− 4n − 9)/8140 é um número inteiro
>>>
>>> --
>>> Israel Meireles Chrisostomo
>>>
>>> --
>>> Esta mensagem foi verificada pelo sistema de antivírus e
>>> acredita-se estar livre de perigo.
>>
>>

-- 
Esta mensagem foi verificada pelo sistema de antiv�rus e
 acredita-se estar livre de perigo.



[obm-l] Re: [obm-l] Teoria dos números

2020-03-15 Por tôpico Pedro José
Boa tarde!
Faltou um contraexemplo.
n=5
3^2*4^2*5^2*6^2*71 não é múltiplo de 11 nem de 37.

Saudações,
PJMS

Em sáb, 14 de mar de 2020 19:47, Pedro José  escreveu:

> Boa noite!
> Creio que a pergunta correta seria, para que valores de n natural...
> 8140=2^2*5*11*37. Então a solução só se dará para um subconjunto dos
> naturais diferente de|N.
>
> Saudações,
> PJMS
>
>
> Em sex, 13 de mar de 2020 20:05, Israel Meireles Chrisostomo <
> israelmchrisost...@gmail.com> escreveu:
>
>>   Dado n natural verifique se a expressão
>>  (n − 2)² (n − 1)²n² (n + 1)² (4n²− 4n − 9)/8140 é um número inteiro
>>
>> --
>> Israel Meireles Chrisostomo
>>
>> --
>> Esta mensagem foi verificada pelo sistema de antivírus e
>> acredita-se estar livre de perigo.
>
>

-- 
Esta mensagem foi verificada pelo sistema de antiv�rus e
 acredita-se estar livre de perigo.



[obm-l] Re: [obm-l] Teoria dos números

2020-03-14 Por tôpico Pedro José
Boa noite!
Creio que a pergunta correta seria, para que valores de n natural...
8140=2^2*5*11*37. Então a solução só se dará para um subconjunto dos
naturais diferente de|N.

Saudações,
PJMS


Em sex, 13 de mar de 2020 20:05, Israel Meireles Chrisostomo <
israelmchrisost...@gmail.com> escreveu:

>   Dado n natural verifique se a expressão
>  (n − 2)² (n − 1)²n² (n + 1)² (4n²− 4n − 9)/8140 é um número inteiro
>
> --
> Israel Meireles Chrisostomo
>
> --
> Esta mensagem foi verificada pelo sistema de antivírus e
> acredita-se estar livre de perigo.

-- 
Esta mensagem foi verificada pelo sistema de antiv�rus e
 acredita-se estar livre de perigo.



[obm-l] Re: [obm-l] Re: [obm-l] Teoria dos números

2019-12-13 Por tôpico Claudio Buffara
Em tese, nada impede...  a == b (mod m) <==> (a - b)/m é inteiro.
Por exemplo, em trigonometria trabalha-se muito com congruência mod 2*pi.
sen x = sen y  e  cos x = cos y <==> x == y (mod 2*pi)

On Fri, Dec 13, 2019 at 3:54 PM Esdras Muniz 
wrote:

> Existe congruência com números que não são inteiros?
>
> Em sex, 13 de dez de 2019 11:57, Prof. Douglas Oliveira <
> profdouglaso.del...@gmail.com> escreveu:
>
>> Olá caros amigos,
>> preciso de uma ajuda pra criar uma fórmula que seja congruente (módulo p)
>> ao somatório
>> S_a=sum{(a^k)/k}, com k de 1 a p-1, sendo p primo ímpar.
>>
>> Saudações
>> Douglas Oliveira
>>
>> --
>> Esta mensagem foi verificada pelo sistema de antivírus e
>> acredita-se estar livre de perigo.
>
>
> --
> Esta mensagem foi verificada pelo sistema de antivírus e
> acredita-se estar livre de perigo.

-- 
Esta mensagem foi verificada pelo sistema de antiv�rus e
 acredita-se estar livre de perigo.



[obm-l] Re: [obm-l] Teoria dos números

2019-12-13 Por tôpico Esdras Muniz
Existe congruência com números que não são inteiros?

Em sex, 13 de dez de 2019 11:57, Prof. Douglas Oliveira <
profdouglaso.del...@gmail.com> escreveu:

> Olá caros amigos,
> preciso de uma ajuda pra criar uma fórmula que seja congruente (módulo p)
> ao somatório
> S_a=sum{(a^k)/k}, com k de 1 a p-1, sendo p primo ímpar.
>
> Saudações
> Douglas Oliveira
>
> --
> Esta mensagem foi verificada pelo sistema de antivírus e
> acredita-se estar livre de perigo.

-- 
Esta mensagem foi verificada pelo sistema de antiv�rus e
 acredita-se estar livre de perigo.



[obm-l] Re: [obm-l] Re: [obm-l] Re: [obm-l] Re: [obm-l] Re: [obm-l] teoria dos números curiosidade

2019-07-04 Por tôpico Israel Meireles Chrisostomo
muito obrigado!!!

Em qui, 4 de jul de 2019 às 09:13, Claudio Buffara <
claudio.buff...@gmail.com> escreveu:

> Considere o seguinte algoritmo:
> Dada a/b (acho q precisa ser entre 0 e 1), tome o menor n1 tal que 1/n1 <=
> a/b.
> Daí, tome o menor n2 tal que 1/n2 <= a/b - 1/n1.
> Daí tome o menor n3 tal que 1/n3 <= a/b - 1/n1 - 1/n2
> Etc...
> Esse processo eventualmente para (quando uma desigualdade <= se torna uma
> igualdade), com:
> a/b = 1/n1 + 1/n2 + 1/n3 + ... + 1/nk, para algum k.
> Resta saber se produz uma fração egípcia (ou seja, se n1 < n2 < n3 < ...)
> e se a fração egípcia resultante é a menor possível.
> Vou pensar melhor é tentar simular alguns casos numa planilha.
>
>
> Enviado do meu iPhone
>
> Em 3 de jul de 2019, à(s) 22:11, Bernardo Freitas Paulo da Costa <
> bernardo...@gmail.com> escreveu:
>
> > On Wed, Jul 3, 2019 at 8:34 PM Claudio Buffara
> >  wrote:
> >> Infinitas.
> >> Basta usar recursivamente a relação  1/n = 1/(n+1) + 1/(n(n+1)), que
> cada vez você obtém uma representação mais longa.
> >> 1/2 = 1/3 + 1/6 = 1/3 + 1/7 + 1/42 = 1/3 + 1/7 + 1/43 + 1/1806 = ...
> >
> > Mais difícil, talvez, seria calcular qual o menor número de termos
> > necessários para representar p/q :)  Será que isso é NP completo?
> >
> > Abraços,
> > --
> > Bernardo Freitas Paulo da Costa
> >
> > --
> > Esta mensagem foi verificada pelo sistema de antivírus e
> > acredita-se estar livre de perigo.
> >
> >
> > =
> > Instruções para entrar na lista, sair da lista e usar a lista em
> > http://www.mat.puc-rio.br/~obmlistas/obm-l.html
> > =
>
> --
> Esta mensagem foi verificada pelo sistema de antivírus e
>  acredita-se estar livre de perigo.
>
>
> =
> Instru�ões para entrar na lista, sair da lista e usar a lista em
> http://www.mat.puc-rio.br/~obmlistas/obm-l.html
> =
>


-- 
Israel Meireles Chrisostomo

-- 
Esta mensagem foi verificada pelo sistema de antiv�rus e
 acredita-se estar livre de perigo.



Re: [obm-l] Re: [obm-l] Re: [obm-l] Re: [obm-l] Re: [obm-l] teoria dos números curiosidade

2019-07-04 Por tôpico Claudio Buffara
Considere o seguinte algoritmo:
Dada a/b (acho q precisa ser entre 0 e 1), tome o menor n1 tal que 1/n1 <= a/b.
Daí, tome o menor n2 tal que 1/n2 <= a/b - 1/n1.
Daí tome o menor n3 tal que 1/n3 <= a/b - 1/n1 - 1/n2
Etc...
Esse processo eventualmente para (quando uma desigualdade <= se torna uma 
igualdade), com:
a/b = 1/n1 + 1/n2 + 1/n3 + ... + 1/nk, para algum k.
Resta saber se produz uma fração egípcia (ou seja, se n1 < n2 < n3 < ...) e se 
a fração egípcia resultante é a menor possível.
Vou pensar melhor é tentar simular alguns casos numa planilha.


Enviado do meu iPhone

Em 3 de jul de 2019, à(s) 22:11, Bernardo Freitas Paulo da Costa 
 escreveu:

> On Wed, Jul 3, 2019 at 8:34 PM Claudio Buffara
>  wrote:
>> Infinitas.
>> Basta usar recursivamente a relação  1/n = 1/(n+1) + 1/(n(n+1)), que cada 
>> vez você obtém uma representação mais longa.
>> 1/2 = 1/3 + 1/6 = 1/3 + 1/7 + 1/42 = 1/3 + 1/7 + 1/43 + 1/1806 = ...
> 
> Mais difícil, talvez, seria calcular qual o menor número de termos
> necessários para representar p/q :)  Será que isso é NP completo?
> 
> Abraços,
> --
> Bernardo Freitas Paulo da Costa
> 
> -- 
> Esta mensagem foi verificada pelo sistema de antivírus e
> acredita-se estar livre de perigo.
> 
> 
> =
> Instruções para entrar na lista, sair da lista e usar a lista em
> http://www.mat.puc-rio.br/~obmlistas/obm-l.html
> =

-- 
Esta mensagem foi verificada pelo sistema de antiv�rus e
 acredita-se estar livre de perigo.


=
Instru��es para entrar na lista, sair da lista e usar a lista em
http://www.mat.puc-rio.br/~obmlistas/obm-l.html
=


[obm-l] Re: [obm-l] Re: [obm-l] Re: [obm-l] Re: [obm-l] teoria dos números curiosidade

2019-07-03 Por tôpico Bernardo Freitas Paulo da Costa
On Wed, Jul 3, 2019 at 8:34 PM Claudio Buffara
 wrote:
> Infinitas.
> Basta usar recursivamente a relação  1/n = 1/(n+1) + 1/(n(n+1)), que cada vez 
> você obtém uma representação mais longa.
> 1/2 = 1/3 + 1/6 = 1/3 + 1/7 + 1/42 = 1/3 + 1/7 + 1/43 + 1/1806 = ...

Mais difícil, talvez, seria calcular qual o menor número de termos
necessários para representar p/q :)  Será que isso é NP completo?

Abraços,
--
Bernardo Freitas Paulo da Costa

-- 
Esta mensagem foi verificada pelo sistema de antiv�rus e
 acredita-se estar livre de perigo.


=
Instru��es para entrar na lista, sair da lista e usar a lista em
http://www.mat.puc-rio.br/~obmlistas/obm-l.html
=


[obm-l] Re: [obm-l] Re: [obm-l] Re: [obm-l] teoria dos números curiosidade

2019-07-03 Por tôpico Claudio Buffara
Infinitas.
Basta usar recursivamente a relação  1/n = 1/(n+1) + 1/(n(n+1)), que cada
vez você obtém uma representação mais longa.
1/2 = 1/3 + 1/6 = 1/3 + 1/7 + 1/42 = 1/3 + 1/7 + 1/43 + 1/1806 = ...

On Wed, Jul 3, 2019 at 7:16 PM Israel Meireles Chrisostomo <
israelmchrisost...@gmail.com> wrote:

> Eu estive pensando para comigo mesmo, e então me perguntei qual é o número
> mínimo de representações distintas que se pode fazer com uma fração em suas
> representações unitárias.Alguém consegue chegar a alguma resposta?
>
>
> 
>  Livre
> de vírus. www.avast.com
> .
> <#m_-8728523408577579589_DAB4FAD8-2DD7-40BB-A1B8-4E2AA1F9FDF2>
>
> Em qua, 3 de jul de 2019 às 16:36, Israel Meireles Chrisostomo <
> israelmchrisost...@gmail.com> escreveu:
>
>> Muito obrigado pessoal!
>>
>>
>> 
>>  Livre
>> de vírus. www.avg.com
>> .
>>
>> <#m_-8728523408577579589_m_-7761868182500660031_DAB4FAD8-2DD7-40BB-A1B8-4E2AA1F9FDF2>
>>
>> Em qua, 3 de jul de 2019 às 16:06, Prof. Douglas Oliveira <
>> profdouglaso.del...@gmail.com> escreveu:
>>
>>> Lembro-me de uma resolucao feita por amigo aqui da lista, o Carlos
>>> Victor, na eureka número 2, no finalzinho, de uma olhada.
>>>
>>> Att
>>> Douglas Oliveira.
>>>
>>> Em qua, 3 de jul de 2019 15:08, Israel Meireles Chrisostomo <
>>> israelmchrisost...@gmail.com> escreveu:
>>>
 Esses dias eu estava estudando sobre frações unitárias, e assisti a um
 vídeo do pessoal impa sobre o assunto e fiquei sinceramente maravilhado com
 a engenhosidade dos egípcios.Mas uma questão não saiu da minha cabeça: um
 número inteiro pode ser separado em frações unitárias?Quais são as
 propriedades necessárias que uma fração deve ter para ser decomposta em
 frações egípcias
 --
 Israel Meireles Chrisostomo


 
  Livre
 de vírus. www.avg.com
 .

 <#m_-8728523408577579589_m_-7761868182500660031_m_-6615042783469650117_m_8002768564935167525_DAB4FAD8-2DD7-40BB-A1B8-4E2AA1F9FDF2>

 --
 Esta mensagem foi verificada pelo sistema de antivírus e
 acredita-se estar livre de perigo.
>>>
>>>
>>> --
>>> Esta mensagem foi verificada pelo sistema de antivírus e
>>> acredita-se estar livre de perigo.
>>
>>
>>
>> --
>> Israel Meireles Chrisostomo
>>
>
>
> --
> Israel Meireles Chrisostomo
>
> --
> Esta mensagem foi verificada pelo sistema de antivírus e
> acredita-se estar livre de perigo.

-- 
Esta mensagem foi verificada pelo sistema de antiv�rus e
 acredita-se estar livre de perigo.



[obm-l] Re: [obm-l] Re: [obm-l] teoria dos números curiosidade

2019-07-03 Por tôpico Israel Meireles Chrisostomo
Eu estive pensando para comigo mesmo, e então me perguntei qual é o número
mínimo de representações distintas que se pode fazer com uma fração em suas
representações unitárias.Alguém consegue chegar a alguma resposta?


Livre
de vírus. www.avast.com
.
<#DAB4FAD8-2DD7-40BB-A1B8-4E2AA1F9FDF2>

Em qua, 3 de jul de 2019 às 16:36, Israel Meireles Chrisostomo <
israelmchrisost...@gmail.com> escreveu:

> Muito obrigado pessoal!
>
>
> 
>  Livre
> de vírus. www.avg.com
> .
> <#m_-7761868182500660031_DAB4FAD8-2DD7-40BB-A1B8-4E2AA1F9FDF2>
>
> Em qua, 3 de jul de 2019 às 16:06, Prof. Douglas Oliveira <
> profdouglaso.del...@gmail.com> escreveu:
>
>> Lembro-me de uma resolucao feita por amigo aqui da lista, o Carlos
>> Victor, na eureka número 2, no finalzinho, de uma olhada.
>>
>> Att
>> Douglas Oliveira.
>>
>> Em qua, 3 de jul de 2019 15:08, Israel Meireles Chrisostomo <
>> israelmchrisost...@gmail.com> escreveu:
>>
>>> Esses dias eu estava estudando sobre frações unitárias, e assisti a um
>>> vídeo do pessoal impa sobre o assunto e fiquei sinceramente maravilhado com
>>> a engenhosidade dos egípcios.Mas uma questão não saiu da minha cabeça: um
>>> número inteiro pode ser separado em frações unitárias?Quais são as
>>> propriedades necessárias que uma fração deve ter para ser decomposta em
>>> frações egípcias
>>> --
>>> Israel Meireles Chrisostomo
>>>
>>>
>>> 
>>>  Livre
>>> de vírus. www.avg.com
>>> .
>>>
>>> <#m_-7761868182500660031_m_-6615042783469650117_m_8002768564935167525_DAB4FAD8-2DD7-40BB-A1B8-4E2AA1F9FDF2>
>>>
>>> --
>>> Esta mensagem foi verificada pelo sistema de antivírus e
>>> acredita-se estar livre de perigo.
>>
>>
>> --
>> Esta mensagem foi verificada pelo sistema de antivírus e
>> acredita-se estar livre de perigo.
>
>
>
> --
> Israel Meireles Chrisostomo
>


-- 
Israel Meireles Chrisostomo

-- 
Esta mensagem foi verificada pelo sistema de antiv�rus e
 acredita-se estar livre de perigo.



[obm-l] Re: [obm-l] Re: [obm-l] teoria dos números curiosidade

2019-07-03 Por tôpico Israel Meireles Chrisostomo
Muito obrigado pessoal!


Livre
de vírus. www.avg.com
.
<#DAB4FAD8-2DD7-40BB-A1B8-4E2AA1F9FDF2>

Em qua, 3 de jul de 2019 às 16:06, Prof. Douglas Oliveira <
profdouglaso.del...@gmail.com> escreveu:

> Lembro-me de uma resolucao feita por amigo aqui da lista, o Carlos Victor,
> na eureka número 2, no finalzinho, de uma olhada.
>
> Att
> Douglas Oliveira.
>
> Em qua, 3 de jul de 2019 15:08, Israel Meireles Chrisostomo <
> israelmchrisost...@gmail.com> escreveu:
>
>> Esses dias eu estava estudando sobre frações unitárias, e assisti a um
>> vídeo do pessoal impa sobre o assunto e fiquei sinceramente maravilhado com
>> a engenhosidade dos egípcios.Mas uma questão não saiu da minha cabeça: um
>> número inteiro pode ser separado em frações unitárias?Quais são as
>> propriedades necessárias que uma fração deve ter para ser decomposta em
>> frações egípcias
>> --
>> Israel Meireles Chrisostomo
>>
>>
>> 
>>  Livre
>> de vírus. www.avg.com
>> .
>>
>> <#m_-6615042783469650117_m_8002768564935167525_DAB4FAD8-2DD7-40BB-A1B8-4E2AA1F9FDF2>
>>
>> --
>> Esta mensagem foi verificada pelo sistema de antivírus e
>> acredita-se estar livre de perigo.
>
>
> --
> Esta mensagem foi verificada pelo sistema de antivírus e
> acredita-se estar livre de perigo.



-- 
Israel Meireles Chrisostomo

-- 
Esta mensagem foi verificada pelo sistema de antiv�rus e
 acredita-se estar livre de perigo.



[obm-l] Re: [obm-l] teoria dos números curiosidade

2019-07-03 Por tôpico Claudio Buffara
Qualquer fração pode ser decomposta em frações egípcias (com numerador = 1).
a/b = 1/b + 1/b + ... + 1/b (a parcelas).
Como as parcelas devem ser distintas, use a identidade 1/n = 1/(n+1) +
1(n(n+1)), com n natural.

Por exemplo:
3/7
= 1/7 + 1/7 + 1/7
= 1/7 + 1/8 + 1/56 + 1/8 + 1/56
= 1/7 + 1/8 + 1/56 + 1/9 + 1/72 + 1/57 + 1/3192

[]s,
Claudio.


On Wed, Jul 3, 2019 at 3:08 PM Israel Meireles Chrisostomo <
israelmchrisost...@gmail.com> wrote:

> Esses dias eu estava estudando sobre frações unitárias, e assisti a um
> vídeo do pessoal impa sobre o assunto e fiquei sinceramente maravilhado com
> a engenhosidade dos egípcios.Mas uma questão não saiu da minha cabeça: um
> número inteiro pode ser separado em frações unitárias?Quais são as
> propriedades necessárias que uma fração deve ter para ser decomposta em
> frações egípcias
> --
> Israel Meireles Chrisostomo
>
>
> 
>  Livre
> de vírus. www.avg.com
> .
> <#m_3122403889933163443_DAB4FAD8-2DD7-40BB-A1B8-4E2AA1F9FDF2>
>
> --
> Esta mensagem foi verificada pelo sistema de antivírus e
> acredita-se estar livre de perigo.

-- 
Esta mensagem foi verificada pelo sistema de antiv�rus e
 acredita-se estar livre de perigo.



[obm-l] Re: [obm-l] teoria dos números curiosidade

2019-07-03 Por tôpico Prof. Douglas Oliveira
Lembro-me de uma resolucao feita por amigo aqui da lista, o Carlos Victor,
na eureka número 2, no finalzinho, de uma olhada.

Att
Douglas Oliveira.

Em qua, 3 de jul de 2019 15:08, Israel Meireles Chrisostomo <
israelmchrisost...@gmail.com> escreveu:

> Esses dias eu estava estudando sobre frações unitárias, e assisti a um
> vídeo do pessoal impa sobre o assunto e fiquei sinceramente maravilhado com
> a engenhosidade dos egípcios.Mas uma questão não saiu da minha cabeça: um
> número inteiro pode ser separado em frações unitárias?Quais são as
> propriedades necessárias que uma fração deve ter para ser decomposta em
> frações egípcias
> --
> Israel Meireles Chrisostomo
>
>
> 
>  Livre
> de vírus. www.avg.com
> .
> <#m_8002768564935167525_DAB4FAD8-2DD7-40BB-A1B8-4E2AA1F9FDF2>
>
> --
> Esta mensagem foi verificada pelo sistema de antivírus e
> acredita-se estar livre de perigo.

-- 
Esta mensagem foi verificada pelo sistema de antiv�rus e
 acredita-se estar livre de perigo.



[obm-l] Re: [obm-l] Re: [obm-l] Teoria dos números revisado

2019-06-21 Por tôpico Israel Meireles Chrisostomo
Muito obrigado Pedro José


Livre
de vírus. www.avg.com
.
<#DAB4FAD8-2DD7-40BB-A1B8-4E2AA1F9FDF2>

Em sex, 21 de jun de 2019 às 11:45, Pedro José 
escreveu:

> Bom dia!
>
> Se x for fixo, falha. Seja x=1. 1 é racional e não há como atender 1 = 1 +
> 1/y com y inteiro.
>
> E mesmo com x,y inteiros livres.
>
> Seja r, u,m racional então r = p/q com p e q inteiros e (p,q)=1 ==> p/q =
> x + 1/y para algum par (x,y) inteiros
> Então py = qxy + q
> py - qxy = q.   Como (xy,x) = y, temos y | q.
>
> se q é primo y =1 ou y = q
>
> Para y=1 temos: p/q = x + 1, só atende para p/q inteiro.
>
> Para y = q temos que (p-1)/q = x, só atenderá para alguns casos.
>
> Acho que a proposição não está correta, mas aguarde a posição de alguém
> com mais bagagem.
>
> Na verdade, ainda há y = -1 ou y = -q
>
> Saudações,
>
> PJMS.
>
>
> Em qui, 20 de jun de 2019 às 16:13, Israel Meireles Chrisostomo <
> israelmchrisost...@gmail.com> escreveu:
>
>> Desconsidere meu outro email. e considere esse aqui
>>
>> 13:43 (há 2 horas)
>> Dado um x fixo, mostrar que todo racional pode ser escrito na forma x+1/y,
>>  com x e y inteiros.Com x sendo o produto de  um número fixo c, por
>> qualquer outro inteiro.
>>
>> --
>> Israel Meireles Chrisostomo
>>
>>
>> 
>>  Livre
>> de vírus. www.avg.com
>> .
>>
>> <#m_9206374589607315600_m_-7461724785975542434_DAB4FAD8-2DD7-40BB-A1B8-4E2AA1F9FDF2>
>>
>> --
>> Esta mensagem foi verificada pelo sistema de antivírus e
>> acredita-se estar livre de perigo.
>
>
> --
> Esta mensagem foi verificada pelo sistema de antivírus e
> acredita-se estar livre de perigo.



-- 
Israel Meireles Chrisostomo

-- 
Esta mensagem foi verificada pelo sistema de antiv�rus e
 acredita-se estar livre de perigo.



[obm-l] Re: [obm-l] Teoria dos números revisado

2019-06-21 Por tôpico Pedro José
Bom dia!

Se x for fixo, falha. Seja x=1. 1 é racional e não há como atender 1 = 1 +
1/y com y inteiro.

E mesmo com x,y inteiros livres.

Seja r, u,m racional então r = p/q com p e q inteiros e (p,q)=1 ==> p/q = x
+ 1/y para algum par (x,y) inteiros
Então py = qxy + q
py - qxy = q.   Como (xy,x) = y, temos y | q.

se q é primo y =1 ou y = q

Para y=1 temos: p/q = x + 1, só atende para p/q inteiro.

Para y = q temos que (p-1)/q = x, só atenderá para alguns casos.

Acho que a proposição não está correta, mas aguarde a posição de alguém com
mais bagagem.

Na verdade, ainda há y = -1 ou y = -q

Saudações,

PJMS.


Em qui, 20 de jun de 2019 às 16:13, Israel Meireles Chrisostomo <
israelmchrisost...@gmail.com> escreveu:

> Desconsidere meu outro email. e considere esse aqui
>
> 13:43 (há 2 horas)
> Dado um x fixo, mostrar que todo racional pode ser escrito na forma x+1/y,
>  com x e y inteiros.Com x sendo o produto de  um número fixo c, por
> qualquer outro inteiro.
>
> --
> Israel Meireles Chrisostomo
>
>
> 
>  Livre
> de vírus. www.avg.com
> .
> <#m_-7461724785975542434_DAB4FAD8-2DD7-40BB-A1B8-4E2AA1F9FDF2>
>
> --
> Esta mensagem foi verificada pelo sistema de antivírus e
> acredita-se estar livre de perigo.

-- 
Esta mensagem foi verificada pelo sistema de antiv�rus e
 acredita-se estar livre de perigo.



[obm-l] Re: [obm-l] Re: [obm-l] teoria dos números

2019-05-28 Por tôpico Pedro José
Boa noite!

Ops! "O que antecede antes[sic].." Dessa feita me superei...

Saudações,
PJMS

Em ter, 28 de mai de 2019 às 18:36, Pedro José 
escreveu:

> Boa tarde!
>
>
> 2003 = 3 mod1000 e ordem1000 3  =  100
> então supondo que a falta de parêntesis está correta.
> Resta calcular 2002^2001 mod100
> 2002= 2 mod100; então temos que procurar o período e o que acontece antes
> do período.
> há 21 ocorrências e depois aparece um período de 20 termos.
> 2^2001=2^(21+r) onde (2001-21) = 20q + r com 0<= r< 20 o que dá r=0
> Então temos que 2^2021= 2^21 = 52 mod100
>
> 2003^2002^2001 = 2^(52) = 241 mod 1000, logo a soma dos algarismos dá 7.
>
> Dá até para fazer no braço, mas são 100 cálculos para achar ordem1000 3 e
> mais 22 cálculos para achar a parte não periódica e a periódica de 2^2001
> mod 100.
> Confesso que fiz uma tabela do Excel.
>
> Saudações,
> PJMS.
>
>
>
>
>
>
> Em ter, 28 de mai de 2019 às 11:47, Bernardo Freitas Paulo da Costa <
> bernardo...@gmail.com> escreveu:
>
>> On Sun, May 26, 2019 at 8:50 AM Daniel Quevedo 
>> wrote:
>> >
>> > Calcule a soma dos 3 últimos algarismos do número 2003^2002^2001.
>>
>> Oi Daniel.  Você já ouviu falar de congruências?  E do "pequeno
>> teorema de Fermat"?
>>
>> Abraços,
>> --
>> Bernardo Freitas Paulo da Costa
>>
>> --
>> Esta mensagem foi verificada pelo sistema de antivírus e
>>  acredita-se estar livre de perigo.
>>
>>
>> =
>> Instruções para entrar na lista, sair da lista e usar a lista em
>> http://www.mat.puc-rio.br/~obmlistas/obm-l.html
>> =
>>
>

-- 
Esta mensagem foi verificada pelo sistema de antiv�rus e
 acredita-se estar livre de perigo.



[obm-l] Re: [obm-l] Re: [obm-l] teoria dos números

2019-05-28 Por tôpico Pedro José
Boa tarde!


2003 = 3 mod1000 e ordem1000 3  =  100
então supondo que a falta de parêntesis está correta.
Resta calcular 2002^2001 mod100
2002= 2 mod100; então temos que procurar o período e o que acontece antes
do período.
há 21 ocorrências e depois aparece um período de 20 termos.
2^2001=2^(21+r) onde (2001-21) = 20q + r com 0<= r< 20 o que dá r=0
Então temos que 2^2021= 2^21 = 52 mod100

2003^2002^2001 = 2^(52) = 241 mod 1000, logo a soma dos algarismos dá 7.

Dá até para fazer no braço, mas são 100 cálculos para achar ordem1000 3 e
mais 22 cálculos para achar a parte não periódica e a periódica de 2^2001
mod 100.
Confesso que fiz uma tabela do Excel.

Saudações,
PJMS.






Em ter, 28 de mai de 2019 às 11:47, Bernardo Freitas Paulo da Costa <
bernardo...@gmail.com> escreveu:

> On Sun, May 26, 2019 at 8:50 AM Daniel Quevedo 
> wrote:
> >
> > Calcule a soma dos 3 últimos algarismos do número 2003^2002^2001.
>
> Oi Daniel.  Você já ouviu falar de congruências?  E do "pequeno
> teorema de Fermat"?
>
> Abraços,
> --
> Bernardo Freitas Paulo da Costa
>
> --
> Esta mensagem foi verificada pelo sistema de antivírus e
>  acredita-se estar livre de perigo.
>
>
> =
> Instruções para entrar na lista, sair da lista e usar a lista em
> http://www.mat.puc-rio.br/~obmlistas/obm-l.html
> =
>

-- 
Esta mensagem foi verificada pelo sistema de antiv�rus e
 acredita-se estar livre de perigo.



[obm-l] Re: [obm-l] teoria dos números

2019-05-28 Por tôpico Bernardo Freitas Paulo da Costa
On Sun, May 26, 2019 at 8:50 AM Daniel Quevedo  wrote:
>
> Calcule a soma dos 3 últimos algarismos do número 2003^2002^2001.

Oi Daniel.  Você já ouviu falar de congruências?  E do "pequeno
teorema de Fermat"?

Abraços,
-- 
Bernardo Freitas Paulo da Costa

-- 
Esta mensagem foi verificada pelo sistema de antiv�rus e
 acredita-se estar livre de perigo.


=
Instru��es para entrar na lista, sair da lista e usar a lista em
http://www.mat.puc-rio.br/~obmlistas/obm-l.html
=


[obm-l] Re: [obm-l] Re: [obm-l] Re: [obm-l] Re: [obm-l] Re: [obm-l] Re: [obm-l] Teoria dos números

2019-05-04 Por tôpico Pedro José
Bom dia!
Obrigado!
Encontrei uma demonstração, mas não tive bagavem para enrender. Vou ler as
publicações.

Saudações,
PJMS

Em sáb, 4 de mai de 2019 11:57, Anderson Torres <
torres.anderson...@gmail.com escreveu:

> Em seg, 29 de abr de 2019 às 16:38, Pedro José 
> escreveu:
> >
> > Boa tarde!
> > Pelo menos consegui descobrir que se um inteiro z >= não puder ser
> escrito da forma z=4^k (8m+7), com m,k >=0 e m,k inteiros então ele pode
> ser representado por uma soma de três parcelas, todas quadrados perfeitos.
> > Já a demonstração, não consegui compreender.
> >
>
> Essa é a parte chata. Mas tem paper pra caramba!
>
> https://en.wikipedia.org/wiki/Legendre%27s_three-square_theorem
> Legendre's three-square theorem - Wikipedia
> https://www.sciencedirect.com/science/article/pii/0022314X74900249
> A new proof of the three squares theorem - ScienceDirect
> https://brilliant.org/wiki/fermats-sum-of-two-squares-theorem/
> Sum of Squares Theorems | Brilliant Math & Science Wiki
>
> https://mathoverflow.net/questions/223939/proving-legendres-sum-of-3-squares-theorem-via-geometry-of-numbers
> nt.number theory - Proving Legendre's Sum of 3 Squares Theorem via
> Geometry of Numbers - MathOverflow
> https://core.ac.uk/download/pdf/82306476.pdf
> PII: 0022-314X(74)90024-9 - 82306476.pdf
>
> https://www.ams.org/journals/proc/1957-008-02/S0002-9939-1957-0085275-8/S0002-9939-1957-0085275-8.pdf
> S0002-9939-1957-0085275-8.pdf
> http://pollack.uga.edu/finding3squares-6.pdf
> finding3squares-6.pdf
> https://arxiv.org/pdf/0812.0540.pdf
> () - 0812.0540.pdf
>
>
>
> > Saudações,
> > PJMS
> >
> > Em seg, 29 de abr de 2019 às 14:14,  escreveu:
> >>
> >>
> >> Em 29 de abr de 2019 11:37, Pedro José  escreveu:
> >>
> >> Bom dia!
> >>
> >> Gostei desse problema. Fiz um montão de exemplos com números que não
> podem ser escritos como 4^n(8n+7) e todos puderam ser escritos como a soma
> de três quadrados.
> >> Vale para todos? Se sim, alguém poderia indicar uma demonstração?
> >>
> >> Saudações,
> >> PJMS
> >>
> >> Em dom, 7 de abr de 2019 às 16:16, Pedro José 
> escreveu:
> >>
> >> Boa tarde!
> >> Fiquei na dúvida se algoritmo valia para demonstração. Mas salvo engano
> para demonstração de quais números aceitam raízes primitivas usa-se
> algoritmo.
> >> Mas, agora com mais calma, poderia ter usado indução.
> >> 1) Foi provado que não vale para n=0.
> >> 2) Supondo que não vale para n, não valeria para n+1, por absurdo.
> Pois, se valesse, teria que valer para n.
> >> Creio que teria ficado mais elegante.
> >>
> >> Saudações,
> >> PJMS
> >>
> >>
> >> Em dom, 7 de abr de 2019 às 07:41, matematica10complicada <
> profdouglaso.del...@gmail.com> escreveu:
> >>
> >> Obrigado irmão. Está correto sim.
> >> Douglas O.
> >>
> >> Em qui, 4 de abr de 2019 às 19:44, Pedro José 
> escreveu:
> >>
> >> Boa noite!
> >> Estou mal, mesmo. Ao invés de nenhum li qualquer. Tinha simulado dois,
> três, quatro e deram fora, já iria questionar.
> >> Mas vamos lá:
> >> 0^2 = 0 mod8; 1^2 = 1 mod8; 2^2 = 4 mod8 3^2= 1 mod8; 4^2 = 0 mod 8;
> 5^2 = 1 mod 8 6^2 = 4 mod 8 e 7^2 = 1 mod8;
> >> Portanto o quadrado de um número, ou dá 0 ou da 1 ou 4 na equivalência
> mod8.
> >>
> >> Caso n=0 ==> x=8k+7= 7 mod8. Como mod conserva a soma, não há como
> somar 3 parcelas do conjunto, mesmo com repetição, {0,1,4} e obter 7. Então
> n>0
> >>
> >> Para n>0
> >> x = 4^n*(8K+7) ==> x pertence a 2 |N seja x = a^2 + b^2 + c^2 com a, b,
> c pertencentes a |N - {0}. teríamos que ter a,b,c pares ou um deles par e
> dois ímpares.
> >> mas 4 | x ==> x= 0 mod4. Mas se w pertence a 2|N + 1 ==> w^2 = 1 mod4.
> e se y pertence a 2 |N ==> y^2 = 0 mod 4. Como temos dois ímpares e um par
> e como a soma se conserva temos que x = 2 mod4, absurdo. Portanto só sobra
> a, b, c pares Se a,b,c pares podemos escrevê-los como a= 2s; b=2t e c=2u
> com s,t,u naturais.
> >> x = a^2+b^2+c^2= 4(s^2+t^2+u^2) ==> x1 = 4^(n-1) * (8m+7) = s^2+t^2+u^2
> e vale o mesmo raciocínio de que s,t,u são pares e poderão ser escritos
> como s=2f; t=2g; u= 2h, com f, g, h naturais e seguir nesse algoritmo até
> que tenhamos xj=4^0(8m+7)= p^2+q^2+r^2, absurdo. Pois, já vimos que n= 0
> não atende.
> >>
> >> Espero estar correto.
> >>
> >> Saudações.
> >>
> >>
> >>
> >>
> >>
> >> Em qua, 3 de abr de 2019 às 15:36, matematica10complicada <
> profdouglaso.del...@gmail.com> escreveu:
> >>
> >> Mostre que nenhum número da forma (4^n)(8k+7) , com n e k naturais pode
> ser escrito como soma de 3 tres quadrados
> >>
> >> Douglas Oliveira
> >>
> >> --
> >> Esta mensagem foi verificada pelo sistema de antivírus e
> >> acredita-se estar livre de perigo.
> >>
> >>
> >> --
> >> Esta mensagem foi verificada pelo sistema de antivírus e
> >> acredita-se estar livre de perigo.
> >>
> >>
> >> --
> >> Esta mensagem foi verificada pelo sistema de antivírus e
> >> acredita-se estar livre de perigo.
> >>
> >>
> >> --
> >> Esta mensagem foi verificada pelo sistema de antiv�rus e
> >> acredita-se estar livre de perigo.
> >>
> >>
> >>
> >> 

[obm-l] Re: [obm-l] Re: [obm-l] Re: [obm-l] Re: [obm-l] Re: [obm-l] Teoria dos números

2019-05-04 Por tôpico Anderson Torres
Em seg, 29 de abr de 2019 às 16:38, Pedro José  escreveu:
>
> Boa tarde!
> Pelo menos consegui descobrir que se um inteiro z >= não puder ser escrito da 
> forma z=4^k (8m+7), com m,k >=0 e m,k inteiros então ele pode ser 
> representado por uma soma de três parcelas, todas quadrados perfeitos.
> Já a demonstração, não consegui compreender.
>

Essa é a parte chata. Mas tem paper pra caramba!

https://en.wikipedia.org/wiki/Legendre%27s_three-square_theorem
Legendre's three-square theorem - Wikipedia
https://www.sciencedirect.com/science/article/pii/0022314X74900249
A new proof of the three squares theorem - ScienceDirect
https://brilliant.org/wiki/fermats-sum-of-two-squares-theorem/
Sum of Squares Theorems | Brilliant Math & Science Wiki
https://mathoverflow.net/questions/223939/proving-legendres-sum-of-3-squares-theorem-via-geometry-of-numbers
nt.number theory - Proving Legendre's Sum of 3 Squares Theorem via
Geometry of Numbers - MathOverflow
https://core.ac.uk/download/pdf/82306476.pdf
PII: 0022-314X(74)90024-9 - 82306476.pdf
https://www.ams.org/journals/proc/1957-008-02/S0002-9939-1957-0085275-8/S0002-9939-1957-0085275-8.pdf
S0002-9939-1957-0085275-8.pdf
http://pollack.uga.edu/finding3squares-6.pdf
finding3squares-6.pdf
https://arxiv.org/pdf/0812.0540.pdf
() - 0812.0540.pdf



> Saudações,
> PJMS
>
> Em seg, 29 de abr de 2019 às 14:14,  escreveu:
>>
>>
>> Em 29 de abr de 2019 11:37, Pedro José  escreveu:
>>
>> Bom dia!
>>
>> Gostei desse problema. Fiz um montão de exemplos com números que não podem 
>> ser escritos como 4^n(8n+7) e todos puderam ser escritos como a soma de três 
>> quadrados.
>> Vale para todos? Se sim, alguém poderia indicar uma demonstração?
>>
>> Saudações,
>> PJMS
>>
>> Em dom, 7 de abr de 2019 às 16:16, Pedro José  escreveu:
>>
>> Boa tarde!
>> Fiquei na dúvida se algoritmo valia para demonstração. Mas salvo engano para 
>> demonstração de quais números aceitam raízes primitivas usa-se algoritmo.
>> Mas, agora com mais calma, poderia ter usado indução.
>> 1) Foi provado que não vale para n=0.
>> 2) Supondo que não vale para n, não valeria para n+1, por absurdo. Pois, se 
>> valesse, teria que valer para n.
>> Creio que teria ficado mais elegante.
>>
>> Saudações,
>> PJMS
>>
>>
>> Em dom, 7 de abr de 2019 às 07:41, matematica10complicada 
>>  escreveu:
>>
>> Obrigado irmão. Está correto sim.
>> Douglas O.
>>
>> Em qui, 4 de abr de 2019 às 19:44, Pedro José  escreveu:
>>
>> Boa noite!
>> Estou mal, mesmo. Ao invés de nenhum li qualquer. Tinha simulado dois, três, 
>> quatro e deram fora, já iria questionar.
>> Mas vamos lá:
>> 0^2 = 0 mod8; 1^2 = 1 mod8; 2^2 = 4 mod8 3^2= 1 mod8; 4^2 = 0 mod 8; 5^2 = 1 
>> mod 8 6^2 = 4 mod 8 e 7^2 = 1 mod8;
>> Portanto o quadrado de um número, ou dá 0 ou da 1 ou 4 na equivalência  mod8.
>>
>> Caso n=0 ==> x=8k+7= 7 mod8. Como mod conserva a soma, não há como somar 3 
>> parcelas do conjunto, mesmo com repetição, {0,1,4} e obter 7. Então n>0
>>
>> Para n>0
>> x = 4^n*(8K+7) ==> x pertence a 2 |N seja x = a^2 + b^2 + c^2 com a, b, c 
>> pertencentes a |N - {0}. teríamos que ter a,b,c pares ou um deles par e dois 
>> ímpares.
>> mas 4 | x ==> x= 0 mod4. Mas se w pertence a 2|N + 1 ==> w^2 = 1 mod4. e se 
>> y pertence a 2 |N ==> y^2 = 0 mod 4. Como temos dois ímpares e um par e como 
>> a soma se conserva temos que x = 2 mod4, absurdo. Portanto só sobra a, b, c 
>> pares Se a,b,c pares podemos escrevê-los como a= 2s; b=2t e c=2u com s,t,u 
>> naturais.
>> x = a^2+b^2+c^2= 4(s^2+t^2+u^2) ==> x1 = 4^(n-1) * (8m+7) = s^2+t^2+u^2 e 
>> vale o mesmo raciocínio de que s,t,u são pares e poderão ser escritos como 
>> s=2f; t=2g; u= 2h, com f, g, h naturais e seguir nesse algoritmo até que 
>> tenhamos xj=4^0(8m+7)= p^2+q^2+r^2, absurdo. Pois, já vimos que n= 0 não 
>> atende.
>>
>> Espero estar correto.
>>
>> Saudações.
>>
>>
>>
>>
>>
>> Em qua, 3 de abr de 2019 às 15:36, matematica10complicada 
>>  escreveu:
>>
>> Mostre que nenhum número da forma (4^n)(8k+7) , com n e k naturais pode ser 
>> escrito como soma de 3 tres quadrados
>>
>> Douglas Oliveira
>>
>> --
>> Esta mensagem foi verificada pelo sistema de antivírus e
>> acredita-se estar livre de perigo.
>>
>>
>> --
>> Esta mensagem foi verificada pelo sistema de antivírus e
>> acredita-se estar livre de perigo.
>>
>>
>> --
>> Esta mensagem foi verificada pelo sistema de antivírus e
>> acredita-se estar livre de perigo.
>>
>>
>> --
>> Esta mensagem foi verificada pelo sistema de antiv�rus e
>> acredita-se estar livre de perigo.
>>
>>
>>
>> --
>> Esta mensagem foi verificada pelo sistema de antivírus e
>> acredita-se estar livre de perigo.
>
>
> --
> Esta mensagem foi verificada pelo sistema de antivírus e
> acredita-se estar livre de perigo.

-- 
Esta mensagem foi verificada pelo sistema de antiv�rus e
 acredita-se estar livre de perigo.


=
Instru��es para entrar na lista, sair da lista e usar a lista em

[obm-l] Re: [obm-l] Re: [obm-l] Re: [obm-l] Re: [obm-l] Teoria dos números

2019-04-29 Por tôpico Pedro José
Boa tarde!
Pelo menos consegui descobrir que se um inteiro z >= não puder ser escrito
da forma z=4^k (8m+7), com m,k >=0 e m,k inteiros então ele pode ser
representado por uma soma de três parcelas, todas quadrados perfeitos.
Já a demonstração, não consegui compreender.

Saudações,
PJMS

Em seg, 29 de abr de 2019 às 14:14,  escreveu:

>
> Em 29 de abr de 2019 11:37, Pedro José  escreveu:
>
> Bom dia!
>
> Gostei desse problema. Fiz um montão de exemplos com números que não podem
> ser escritos como 4^n(8n+7) e todos puderam ser escritos como a soma de
> três quadrados.
> Vale para todos? Se sim, alguém poderia indicar uma demonstração?
>
> Saudações,
> PJMS
>
> Em dom, 7 de abr de 2019 às 16:16, Pedro José 
> escreveu:
>
> Boa tarde!
> Fiquei na dúvida se algoritmo valia para demonstração. Mas salvo engano
> para demonstração de quais números aceitam raízes primitivas usa-se
> algoritmo.
> Mas, agora com mais calma, poderia ter usado indução.
> 1) Foi provado que não vale para n=0.
> 2) Supondo que não vale para n, não valeria para n+1, por absurdo. Pois,
> se valesse, teria que valer para n.
> Creio que teria ficado mais elegante.
>
> Saudações,
> PJMS
>
>
> Em dom, 7 de abr de 2019 às 07:41, matematica10complicada <
> profdouglaso.del...@gmail.com> escreveu:
>
> Obrigado irmão. Está correto sim.
> Douglas O.
>
> Em qui, 4 de abr de 2019 às 19:44, Pedro José 
> escreveu:
>
> Boa noite!
> Estou mal, mesmo. Ao invés de nenhum li qualquer. Tinha simulado dois,
> três, quatro e deram fora, já iria questionar.
> Mas vamos lá:
> 0^2 = 0 mod8; 1^2 = 1 mod8; 2^2 = 4 mod8 3^2= 1 mod8; 4^2 = 0 mod 8; 5^2 =
> 1 mod 8 6^2 = 4 mod 8 e 7^2 = 1 mod8;
> Portanto o quadrado de um número, ou dá 0 ou da 1 ou 4 na equivalência
> mod8.
>
> Caso n=0 ==> x=8k+7= 7 mod8. Como mod conserva a soma, não há como somar 3
> parcelas do conjunto, mesmo com repetição, {0,1,4} e obter 7. Então n>0
>
> Para n>0
> x = 4^n*(8K+7) ==> x pertence a 2 |N seja x = a^2 + b^2 + c^2 com a, b, c
> pertencentes a |N - {0}. teríamos que ter a,b,c pares ou um deles par e
> dois ímpares.
> mas 4 | x ==> x= 0 mod4. Mas se w pertence a 2|N + 1 ==> w^2 = 1 mod4. e
> se y pertence a 2 |N ==> y^2 = 0 mod 4. Como temos dois ímpares e um par e
> como a soma se conserva temos que x = 2 mod4, absurdo. Portanto só sobra a,
> b, c pares Se a,b,c pares podemos escrevê-los como a= 2s; b=2t e c=2u com
> s,t,u naturais.
> x = a^2+b^2+c^2= 4(s^2+t^2+u^2) ==> x1 = 4^(n-1) * (8m+7) = s^2+t^2+u^2 e
> vale o mesmo raciocínio de que s,t,u são pares e poderão ser escritos como
> s=2f; t=2g; u= 2h, com f, g, h naturais e seguir nesse algoritmo até que
> tenhamos xj=4^0(8m+7)= p^2+q^2+r^2, absurdo. Pois, já vimos que n= 0 não
> atende.
>
> Espero estar correto.
>
> Saudações.
>
>
>
>
>
> Em qua, 3 de abr de 2019 às 15:36, matematica10complicada <
> profdouglaso.del...@gmail.com> escreveu:
>
> Mostre que nenhum número da forma (4^n)(8k+7) , com n e k naturais pode
> ser escrito como soma de 3 tres quadrados
>
> Douglas Oliveira
>
> --
> Esta mensagem foi verificada pelo sistema de antivírus e
> acredita-se estar livre de perigo.
>
>
> --
> Esta mensagem foi verificada pelo sistema de antivírus e
> acredita-se estar livre de perigo.
>
>
> --
> Esta mensagem foi verificada pelo sistema de antivírus e
> acredita-se estar livre de perigo.
>
>
> --
> Esta mensagem foi verificada pelo sistema de antiv�rus e
> acredita-se estar livre de perigo.
>
>
>
> --
> Esta mensagem foi verificada pelo sistema de antivírus e
> acredita-se estar livre de perigo.

-- 
Esta mensagem foi verificada pelo sistema de antiv�rus e
 acredita-se estar livre de perigo.



Re: [obm-l] Re: [obm-l] Re: [obm-l] Re: [obm-l] Teoria dos números

2019-04-29 Por tôpico prmanso
Em 29 de abr de 2019 11:37, Pedro José  escreveu:Bom dia!Gostei desse problema. Fiz um montão de exemplos com números que não podem ser escritos como 4^n(8n+7) e todos puderam ser escritos como a soma de três quadrados.Vale para todos? Se sim, alguém poderia indicar uma demonstração?Saudações,PJMSEm dom, 7 de abr de 2019 às 16:16, Pedro José  escreveu:Boa tarde!Fiquei na dúvida se algoritmo valia para demonstração. Mas salvo engano para demonstração de quais números aceitam raízes primitivas usa-se algoritmo.Mas, agora com mais calma, poderia ter usado indução.1) Foi provado que não vale para n=0.2) Supondo que não vale para n, não valeria para n+1, por absurdo. Pois, se valesse, teria que valer para n.Creio que teria ficado mais elegante.Saudações,PJMSEm dom, 7 de abr de 2019 às 07:41, matematica10complicada  escreveu:Obrigado irmão. Está correto sim. Douglas O.Em qui, 4 de abr de 2019 às 19:44, Pedro José  escreveu:Boa noite!Estou mal, mesmo. Ao invés de nenhum li qualquer. Tinha simulado dois, três, quatro e deram fora, já iria questionar.Mas vamos lá:0^2 = 0 mod8; 1^2 = 1 mod8; 2^2 = 4 mod8 3^2= 1 mod8; 4^2 = 0 mod 8; 5^2 = 1 mod 8 6^2 = 4 mod 8 e 7^2 = 1 mod8;Portanto o quadrado de um número, ou dá 0 ou da 1 ou 4 na equivalência  mod8.Caso n=0 ==> x=8k+7= 7 mod8. Como mod conserva a soma, não há como somar 3 parcelas do conjunto, mesmo com repetição, {0,1,4} e obter 7. Então n>0Para n>0x = 4^n*(8K+7) ==> x pertence a 2 |N seja x = a^2 + b^2 + c^2 com a, b, c pertencentes a |N - {0}. teríamos que ter a,b,c pares ou um deles par e dois ímpares.mas 4 | x ==> x= 0 mod4. Mas se w pertence a 2|N + 1 ==> w^2 = 1 mod4. e se y pertence a 2 |N ==> y^2 = 0 mod 4. Como temos dois ímpares e um par e como a soma se conserva temos que x = 2 mod4, absurdo. Portanto só sobra a, b, c pares Se a,b,c pares podemos escrevê-los como a= 2s; b=2t e c=2u com s,t,u naturais.x = a^2+b^2+c^2= 4(s^2+t^2+u^2) ==> x1 = 4^(n-1) * (8m+7) = s^2+t^2+u^2 e vale o mesmo raciocínio de que s,t,u são pares e poderão ser escritos como s=2f; t=2g; u= 2h, com f, g, h naturais e seguir nesse algoritmo até que tenhamos xj=4^0(8m+7)= p^2+q^2+r^2, absurdo. Pois, já vimos que n= 0 não atende.Espero estar correto.Saudações. Em qua, 3 de abr de 2019 às 15:36, matematica10complicada  escreveu:Mostre que nenhum número da forma (4^n)(8k+7) , com n e k naturais pode ser escrito como soma de 3 tres quadradosDouglas Oliveira
--
Esta mensagem foi verificada pelo sistema de antivírus e 
 acredita-se estar livre de perigo.


--
Esta mensagem foi verificada pelo sistema de antivírus e 
 acredita-se estar livre de perigo.


--
Esta mensagem foi verificada pelo sistema de antivírus e 
 acredita-se estar livre de perigo.



--
Esta mensagem foi verificada pelo sistema de antiv�rus e 
 acredita-se estar livre de perigo.

--
Esta mensagem foi verificada pelo sistema de antivírus e 
 acredita-se estar livre de perigo.




[obm-l] Re: [obm-l] Re: [obm-l] Re: [obm-l] Teoria dos números

2019-04-29 Por tôpico Pedro José
Bom dia!

Gostei desse problema. Fiz um montão de exemplos com números que não podem
ser escritos como 4^n(8n+7) e todos puderam ser escritos como a soma de
três quadrados.
Vale para todos? Se sim, alguém poderia indicar uma demonstração?

Saudações,
PJMS

Em dom, 7 de abr de 2019 às 16:16, Pedro José 
escreveu:

> Boa tarde!
> Fiquei na dúvida se algoritmo valia para demonstração. Mas salvo engano
> para demonstração de quais números aceitam raízes primitivas usa-se
> algoritmo.
> Mas, agora com mais calma, poderia ter usado indução.
> 1) Foi provado que não vale para n=0.
> 2) Supondo que não vale para n, não valeria para n+1, por absurdo. Pois,
> se valesse, teria que valer para n.
> Creio que teria ficado mais elegante.
>
> Saudações,
> PJMS
>
>
> Em dom, 7 de abr de 2019 às 07:41, matematica10complicada <
> profdouglaso.del...@gmail.com> escreveu:
>
>> Obrigado irmão. Está correto sim.
>> Douglas O.
>>
>> Em qui, 4 de abr de 2019 às 19:44, Pedro José 
>> escreveu:
>>
>>> Boa noite!
>>> Estou mal, mesmo. Ao invés de nenhum li qualquer. Tinha simulado dois,
>>> três, quatro e deram fora, já iria questionar.
>>> Mas vamos lá:
>>> 0^2 = 0 mod8; 1^2 = 1 mod8; 2^2 = 4 mod8 3^2= 1 mod8; 4^2 = 0 mod 8; 5^2
>>> = 1 mod 8 6^2 = 4 mod 8 e 7^2 = 1 mod8;
>>> Portanto o quadrado de um número, ou dá 0 ou da 1 ou 4 na equivalência
>>> mod8.
>>>
>>> Caso n=0 ==> x=8k+7= 7 mod8. Como mod conserva a soma, não há como somar
>>> 3 parcelas do conjunto, mesmo com repetição, {0,1,4} e obter 7. Então n>0
>>>
>>> Para n>0
>>> x = 4^n*(8K+7) ==> x pertence a 2 |N seja x = a^2 + b^2 + c^2 com a, b,
>>> c pertencentes a |N - {0}. teríamos que ter a,b,c pares ou um deles par e
>>> dois ímpares.
>>> mas 4 | x ==> x= 0 mod4. Mas se w pertence a 2|N + 1 ==> w^2 = 1 mod4. e
>>> se y pertence a 2 |N ==> y^2 = 0 mod 4. Como temos dois ímpares e um par e
>>> como a soma se conserva temos que x = 2 mod4, absurdo. Portanto só sobra a,
>>> b, c pares Se a,b,c pares podemos escrevê-los como a= 2s; b=2t e c=2u com
>>> s,t,u naturais.
>>> x = a^2+b^2+c^2= 4(s^2+t^2+u^2) ==> x1 = 4^(n-1) * (8m+7) = s^2+t^2+u^2
>>> e vale o mesmo raciocínio de que s,t,u são pares e poderão ser escritos
>>> como s=2f; t=2g; u= 2h, com f, g, h naturais e seguir nesse algoritmo até
>>> que tenhamos xj=4^0(8m+7)= p^2+q^2+r^2, absurdo. Pois, já vimos que n= 0
>>> não atende.
>>>
>>> Espero estar correto.
>>>
>>> Saudações.
>>>
>>>
>>>
>>>
>>>
>>> Em qua, 3 de abr de 2019 às 15:36, matematica10complicada <
>>> profdouglaso.del...@gmail.com> escreveu:
>>>
 Mostre que nenhum número da forma (4^n)(8k+7) , com n e k naturais pode
 ser escrito como soma de 3 tres quadrados

 Douglas Oliveira

 --
 Esta mensagem foi verificada pelo sistema de antivírus e
 acredita-se estar livre de perigo.
>>>
>>>
>>> --
>>> Esta mensagem foi verificada pelo sistema de antivírus e
>>> acredita-se estar livre de perigo.
>>
>>
>> --
>> Esta mensagem foi verificada pelo sistema de antivírus e
>> acredita-se estar livre de perigo.
>
>

-- 
Esta mensagem foi verificada pelo sistema de antiv�rus e
 acredita-se estar livre de perigo.



[obm-l] Re: [obm-l] Re: [obm-l] Re: [obm-l] Teoria dos números

2019-04-07 Por tôpico Pedro José
Boa tarde!
Fiquei na dúvida se algoritmo valia para demonstração. Mas salvo engano
para demonstração de quais números aceitam raízes primitivas usa-se
algoritmo.
Mas, agora com mais calma, poderia ter usado indução.
1) Foi provado que não vale para n=0.
2) Supondo que não vale para n, não valeria para n+1, por absurdo. Pois, se
valesse, teria que valer para n.
Creio que teria ficado mais elegante.

Saudações,
PJMS


Em dom, 7 de abr de 2019 às 07:41, matematica10complicada <
profdouglaso.del...@gmail.com> escreveu:

> Obrigado irmão. Está correto sim.
> Douglas O.
>
> Em qui, 4 de abr de 2019 às 19:44, Pedro José 
> escreveu:
>
>> Boa noite!
>> Estou mal, mesmo. Ao invés de nenhum li qualquer. Tinha simulado dois,
>> três, quatro e deram fora, já iria questionar.
>> Mas vamos lá:
>> 0^2 = 0 mod8; 1^2 = 1 mod8; 2^2 = 4 mod8 3^2= 1 mod8; 4^2 = 0 mod 8; 5^2
>> = 1 mod 8 6^2 = 4 mod 8 e 7^2 = 1 mod8;
>> Portanto o quadrado de um número, ou dá 0 ou da 1 ou 4 na equivalência
>> mod8.
>>
>> Caso n=0 ==> x=8k+7= 7 mod8. Como mod conserva a soma, não há como somar
>> 3 parcelas do conjunto, mesmo com repetição, {0,1,4} e obter 7. Então n>0
>>
>> Para n>0
>> x = 4^n*(8K+7) ==> x pertence a 2 |N seja x = a^2 + b^2 + c^2 com a, b, c
>> pertencentes a |N - {0}. teríamos que ter a,b,c pares ou um deles par e
>> dois ímpares.
>> mas 4 | x ==> x= 0 mod4. Mas se w pertence a 2|N + 1 ==> w^2 = 1 mod4. e
>> se y pertence a 2 |N ==> y^2 = 0 mod 4. Como temos dois ímpares e um par e
>> como a soma se conserva temos que x = 2 mod4, absurdo. Portanto só sobra a,
>> b, c pares Se a,b,c pares podemos escrevê-los como a= 2s; b=2t e c=2u com
>> s,t,u naturais.
>> x = a^2+b^2+c^2= 4(s^2+t^2+u^2) ==> x1 = 4^(n-1) * (8m+7) = s^2+t^2+u^2 e
>> vale o mesmo raciocínio de que s,t,u são pares e poderão ser escritos como
>> s=2f; t=2g; u= 2h, com f, g, h naturais e seguir nesse algoritmo até que
>> tenhamos xj=4^0(8m+7)= p^2+q^2+r^2, absurdo. Pois, já vimos que n= 0 não
>> atende.
>>
>> Espero estar correto.
>>
>> Saudações.
>>
>>
>>
>>
>>
>> Em qua, 3 de abr de 2019 às 15:36, matematica10complicada <
>> profdouglaso.del...@gmail.com> escreveu:
>>
>>> Mostre que nenhum número da forma (4^n)(8k+7) , com n e k naturais pode
>>> ser escrito como soma de 3 tres quadrados
>>>
>>> Douglas Oliveira
>>>
>>> --
>>> Esta mensagem foi verificada pelo sistema de antivírus e
>>> acredita-se estar livre de perigo.
>>
>>
>> --
>> Esta mensagem foi verificada pelo sistema de antivírus e
>> acredita-se estar livre de perigo.
>
>
> --
> Esta mensagem foi verificada pelo sistema de antivírus e
> acredita-se estar livre de perigo.

-- 
Esta mensagem foi verificada pelo sistema de antiv�rus e
 acredita-se estar livre de perigo.



[obm-l] Re: [obm-l] Re: [obm-l] Teoria dos números

2019-04-07 Por tôpico matematica10complicada
Obrigado irmão. Está correto sim.
Douglas O.

Em qui, 4 de abr de 2019 às 19:44, Pedro José 
escreveu:

> Boa noite!
> Estou mal, mesmo. Ao invés de nenhum li qualquer. Tinha simulado dois,
> três, quatro e deram fora, já iria questionar.
> Mas vamos lá:
> 0^2 = 0 mod8; 1^2 = 1 mod8; 2^2 = 4 mod8 3^2= 1 mod8; 4^2 = 0 mod 8; 5^2 =
> 1 mod 8 6^2 = 4 mod 8 e 7^2 = 1 mod8;
> Portanto o quadrado de um número, ou dá 0 ou da 1 ou 4 na equivalência
> mod8.
>
> Caso n=0 ==> x=8k+7= 7 mod8. Como mod conserva a soma, não há como somar 3
> parcelas do conjunto, mesmo com repetição, {0,1,4} e obter 7. Então n>0
>
> Para n>0
> x = 4^n*(8K+7) ==> x pertence a 2 |N seja x = a^2 + b^2 + c^2 com a, b, c
> pertencentes a |N - {0}. teríamos que ter a,b,c pares ou um deles par e
> dois ímpares.
> mas 4 | x ==> x= 0 mod4. Mas se w pertence a 2|N + 1 ==> w^2 = 1 mod4. e
> se y pertence a 2 |N ==> y^2 = 0 mod 4. Como temos dois ímpares e um par e
> como a soma se conserva temos que x = 2 mod4, absurdo. Portanto só sobra a,
> b, c pares Se a,b,c pares podemos escrevê-los como a= 2s; b=2t e c=2u com
> s,t,u naturais.
> x = a^2+b^2+c^2= 4(s^2+t^2+u^2) ==> x1 = 4^(n-1) * (8m+7) = s^2+t^2+u^2 e
> vale o mesmo raciocínio de que s,t,u são pares e poderão ser escritos como
> s=2f; t=2g; u= 2h, com f, g, h naturais e seguir nesse algoritmo até que
> tenhamos xj=4^0(8m+7)= p^2+q^2+r^2, absurdo. Pois, já vimos que n= 0 não
> atende.
>
> Espero estar correto.
>
> Saudações.
>
>
>
>
>
> Em qua, 3 de abr de 2019 às 15:36, matematica10complicada <
> profdouglaso.del...@gmail.com> escreveu:
>
>> Mostre que nenhum número da forma (4^n)(8k+7) , com n e k naturais pode
>> ser escrito como soma de 3 tres quadrados
>>
>> Douglas Oliveira
>>
>> --
>> Esta mensagem foi verificada pelo sistema de antivírus e
>> acredita-se estar livre de perigo.
>
>
> --
> Esta mensagem foi verificada pelo sistema de antivírus e
> acredita-se estar livre de perigo.

-- 
Esta mensagem foi verificada pelo sistema de antiv�rus e
 acredita-se estar livre de perigo.



[obm-l] Re: [obm-l] Teoria dos números

2019-04-05 Por tôpico Claudio Buffara
Ou seja, existem m e n inteiros positivos tais que:
8a + 1 = mb
e
8b + 1 = na

De cara, dá pra ver que a e b precisam ser ímpares (caso contrário, não
dividiriam 8b+1 e 8a+1, respectivamente).

Além disso...
b = (8a+1)/m ==>
8(8a+1)/m + 1 = na ==>
64a + 8 + m = mna ==>
a = (m+8)/(mn-64) (A)

Analogamente, b = (n+8)/(mn-64)   (B)

Isso significa que mn > 64  e que  mn-64 divide m+8 e n+8 ==>
mn - 64 <= m + 8   e   mn - 64 <= n + 8 ==>
64 < mn <= 72 + m   e   64 < mn <= 72 + n ==>
64/m < n < 72/m + 1   e   64/n < m <= 72/n + 1

Agora, m e n não podem ser ambos <= 8, caso contrário seria mn <= 64.

Suponhamos que m > 8. Então  n < 72/m + 1 < 72/8 + 1 = 9 + 1 = 10.
Mas m não pode ser muito grande, pois mn - 64 <= n + 8 < 18 ==>
mn < 82 ==>
m < 82/n <= 82

Ou seja...
Se m > 8, então m < 82 (ou seja, m >= 9 ==> m <= 81).
Neste caso, 64/81 <= 64/m < n < 72/m + 1 <= 72/9 + 1 = 8 + 1 = 9.
Ou seja 1 <= n <= 8.
E sempre mn > 64, ou seja mn >= 65.

Resumindo: se m >= 9  então  m <= 81  e  65/m <= n <= 8.

Analogamente, se n >= 9  então  n <= 81  e  65/n <= m <= 8.

Agora, é só plugar estes valores de m e n nas fórmulas (A) e (B) acima (de
preferência, com um computador) e ver quais resultam em a e b inteiros.

Com uma planilha Excel, eu achei as seguintes 11 soluções:
 ab
 11
 13
 19
 31
 3   25
 91
 9   73
13  21
21  13
25   3
73   9

[]s,
Claudio.


On Thu, Apr 4, 2019 at 8:30 PM Daniel Quevedo  wrote:

> O número de pares ordenados  de inteiros positivos (*a, b*) tais que 8*b*
> + 1 é múltiplo de *a* e 8*a* + 1 é múltiplo de *b* é igual a:
>
> R: 11
>
> --
> Daniel
>
>
> 
>  Livre
> de vírus. www.avast.com
> .
>
> <#m_137718337439888202_m_2719591652256721004_m_5267545371236784792_m_8044354502181798805_DAB4FAD8-2DD7-40BB-A1B8-4E2AA1F9FDF2>
>
> --
> Esta mensagem foi verificada pelo sistema de antivírus e
> acredita-se estar livre de perigo.

-- 
Esta mensagem foi verificada pelo sistema de antiv�rus e
 acredita-se estar livre de perigo.



[obm-l] Re: [obm-l] Teoria dos números

2019-04-04 Por tôpico Pedro José
Boa noite!
Estou mal, mesmo. Ao invés de nenhum li qualquer. Tinha simulado dois,
três, quatro e deram fora, já iria questionar.
Mas vamos lá:
0^2 = 0 mod8; 1^2 = 1 mod8; 2^2 = 4 mod8 3^2= 1 mod8; 4^2 = 0 mod 8; 5^2 =
1 mod 8 6^2 = 4 mod 8 e 7^2 = 1 mod8;
Portanto o quadrado de um número, ou dá 0 ou da 1 ou 4 na equivalência
mod8.

Caso n=0 ==> x=8k+7= 7 mod8. Como mod conserva a soma, não há como somar 3
parcelas do conjunto, mesmo com repetição, {0,1,4} e obter 7. Então n>0

Para n>0
x = 4^n*(8K+7) ==> x pertence a 2 |N seja x = a^2 + b^2 + c^2 com a, b, c
pertencentes a |N - {0}. teríamos que ter a,b,c pares ou um deles par e
dois ímpares.
mas 4 | x ==> x= 0 mod4. Mas se w pertence a 2|N + 1 ==> w^2 = 1 mod4. e se
y pertence a 2 |N ==> y^2 = 0 mod 4. Como temos dois ímpares e um par e
como a soma se conserva temos que x = 2 mod4, absurdo. Portanto só sobra a,
b, c pares Se a,b,c pares podemos escrevê-los como a= 2s; b=2t e c=2u com
s,t,u naturais.
x = a^2+b^2+c^2= 4(s^2+t^2+u^2) ==> x1 = 4^(n-1) * (8m+7) = s^2+t^2+u^2 e
vale o mesmo raciocínio de que s,t,u são pares e poderão ser escritos como
s=2f; t=2g; u= 2h, com f, g, h naturais e seguir nesse algoritmo até que
tenhamos xj=4^0(8m+7)= p^2+q^2+r^2, absurdo. Pois, já vimos que n= 0 não
atende.

Espero estar correto.

Saudações.





Em qua, 3 de abr de 2019 às 15:36, matematica10complicada <
profdouglaso.del...@gmail.com> escreveu:

> Mostre que nenhum número da forma (4^n)(8k+7) , com n e k naturais pode
> ser escrito como soma de 3 tres quadrados
>
> Douglas Oliveira
>
> --
> Esta mensagem foi verificada pelo sistema de antivírus e
> acredita-se estar livre de perigo.

-- 
Esta mensagem foi verificada pelo sistema de antiv�rus e
 acredita-se estar livre de perigo.



[obm-l] Re: [obm-l] Teoria dos números

2019-02-05 Por tôpico Rodrigo Ângelo
Não consegui resolver inteiro, mas uma possível conjectura é que seria 9n

Como 9+...+9 = 9n, então o número 999...999 é divisível por 9, logo o
produto (888...888)×(999...999) também tem 9 como fator.

Então temos que a soma dos algarismos do produto em questão também é
divisível por 9.

Fazendo para n 1, 2 e 3, temos:

8 x 9 = 72  -- 7 + 2 = 9 = 9x1
88 x 99 = 8712 --- 8+7+1+2 = 18= 9x2
888 x 999 = 887112 - 8+8+7+1+1+2 = 27 = 9x3

Dá pra provar esse critério de divisibilidade por 9... Vi num vídeo do
Numberphile esses dias.

Boa sorte com o problema.

On Tue, Feb 5, 2019 at 5:33 PM matematica10complicada <
profdouglaso.del...@gmail.com> wrote:

> Amigos preciso de uma ajuda.
>
> PROBLEMA:
>
> Determinar a soma dos algarismos do produto (888...888)×(999...999), onde
> cada parcela possui "n" algarismos.
>
> Douglas Oliveira.
>
> --
> Esta mensagem foi verificada pelo sistema de antivírus e
> acredita-se estar livre de perigo.

-- 
Esta mensagem foi verificada pelo sistema de antiv�rus e
 acredita-se estar livre de perigo.



[obm-l] Re: [obm-l] Teoria dos números

2019-02-05 Por tôpico Pedro Cardoso
Ah, a soma fica sendo 8n-1+1n+1, ou seja, 9n

Em ter, 5 de fev de 2019 15:33, matematica10complicada <
profdouglaso.del...@gmail.com> escreveu:

> Amigos preciso de uma ajuda.
>
> PROBLEMA:
>
> Determinar a soma dos algarismos do produto (888...888)×(999...999), onde
> cada parcela possui "n" algarismos.
>
> Douglas Oliveira.
>
> --
> Esta mensagem foi verificada pelo sistema de antivírus e
> acredita-se estar livre de perigo.

-- 
Esta mensagem foi verificada pelo sistema de antiv�rus e
 acredita-se estar livre de perigo.



[obm-l] Re: [obm-l] Teoria dos números

2019-02-05 Por tôpico Pedro Cardoso
Basta substituir (999...999) por (10^n-1)
O produto será 888...888000...000-888...888, ou seja, 888...887111...112

Em ter, 5 de fev de 2019 15:33, matematica10complicada <
profdouglaso.del...@gmail.com> escreveu:

> Amigos preciso de uma ajuda.
>
> PROBLEMA:
>
> Determinar a soma dos algarismos do produto (888...888)×(999...999), onde
> cada parcela possui "n" algarismos.
>
> Douglas Oliveira.
>
> --
> Esta mensagem foi verificada pelo sistema de antivírus e
> acredita-se estar livre de perigo.

-- 
Esta mensagem foi verificada pelo sistema de antiv�rus e
 acredita-se estar livre de perigo.



[obm-l] Re: [obm-l] Teoria dos números

2018-12-11 Por tôpico matematica10complicada
Então...

Usando MA>=MG temos

(x+y)(y+z)=xz+xy+y^2+yz>=2sqrt( xz(xy+y^2+yz))=2sqrt(xyz(x+y+x))=2

Resposta :2

Um abraço
Douglas Oliveira.

Em ter, 11 de dez de 2018 11:53, Daniel Quevedo  Se x, y e z são números reais positivos tais que xyz(x+y+z) = 1, o menor
> valor da expressão (x+y)(y+z) é:
> A) 1/2
> B) 2/3
> C) 4/3
> D) 3/2
> E) 2
>
> R: e
> --
> Fiscal: Daniel Quevedo
>
> --
> Esta mensagem foi verificada pelo sistema de antivírus e
> acredita-se estar livre de perigo.

-- 
Esta mensagem foi verificada pelo sistema de antiv�rus e
 acredita-se estar livre de perigo.



[obm-l] Re: [obm-l] Re: [obm-l] Re: [obm-l] Re: [obm-l] Teoria dos números

2018-11-28 Por tôpico Pedro José
Boa tarde!
Perdoem-me pela insistência.
Mas outra forma de pensar.
Se k>0, e se a>b e se pensarmos em duas soluções positivas logicamente
estamos assumindo que a seja máximo. Pois, se existe a1 solução e a1>=a
então a1.a=b^2-k>b^2, absurdo.
Portanto quando dizemos que a>b, estamos escolhendo conscientemente a
solução máxima. Então não é ocaso de pegar uma solução hipotética, supor
que é solução mínima  e provar que existe uma menor, gerando absurdo.
Saudações,
PJMS

Em Qua, 28 de nov de 2018 15:42, Pedro José  escreveu:

> Boa tarde!
> Preciso de ajuda.
> Após pensar mais um pouco é bem razoável que dada uma solução hipotética e
> se consiga provar que há uma menor, que seja um absurdo. Absurdo no
> sentido, que não há solução. Gostaria até que me sugerissem material
> didático sobre o tópico.
> Não obstante existe solução para a1 e
> x inteiro.
> Então há um paradoxo. Que por um lado se a é solução para a 0 Mas quando se achou a foi feita uma restrição, SPG, que a >=b e após
> estudar o caso a=b, ficamos com a restrição a>b, que é usada para provar
> que a1b. Só que:
> a1=(b^2-k)/ab.
> Então esse é o ponto a1 mesmo sendo maior que zero,  não é solução pois
> a1 solução. A prova para a  Por favor, alguém poderia opinar.
> Saudações,
> PJMS
>
> Em Seg, 26 de nov de 2018 01:59, Pedro José 
> escreveu:
>
>> Bom dia!
>> Refiro-me a solução recomendada por Israel.
>> A princípio eu encrenquei com a solução. Pois, sem nenhuma caracterização
>> definiu que a era mínimo. Então achei que a solução para a > capenga. Mas como não houve nenhuma crítica, julguei ser cisma minha. Mas
>> depois me veio o pensamento, usando a técnica usada na resolução sempre que
>> houver duas soluções(digo duas mesmo, distintas) haveria um absurdo. Pois
>> ele supôs que a era mínimo e provou que a1, solução, a1> como ele não usou nenhum argumento para supor que a era mínimo, apenas
>> arbitrou, poderia ter arbitrado que a era máximo e se a1>a, também seria
>> absurdo.
>> Aí, encrenquei mesmo com a soluçao e achei essa família de soluções para
>> a> K=x^2; b=x^3 e a =x^5-x, que para x>1, xinteiro, atende a> k=(a^2+b^2)/(ab+1); continua dando um quadrado perfeito, mas se não fosse?
>> A linha de argumento da solução,  desprezou essa possibilidade.
>> Preciso ajuda, estou correto ou errado?
>> Grato,
>> PJMS
>>
>> Em Seg, 27 de ago de 2018 11:01, Pedro José 
>> escreveu:
>>
>>> Bom dia!
>>>
>>> Linda solução pela simplicidade de ferramentas utilizadas.
>>> Todavia, creio eu que não foi de bom alvitre utilizar a imagem de um
>>> matemático famoso e divulgar que ele só ganhou um ponto na questão.
>>> A mensagem, não explícita, mas é uma mensagem:"Ele não resolveu mas eu
>>> sim."
>>> As condições de resolução são totalmente diversas. Inclusive, devido às
>>> questões anteriores ele já pode ter chegado a essa com o ponterinho do
>>> relógio pendurado.
>>> A solução do problema, mesmo tardia, sem a carga emocional que uma IMO
>>> deve impor aos seus participantes, ainda é carregada de méritos, e na minha
>>> visão, essa em particular, com uma beleza maior por ser sutil e singela.
>>> Mas usar a imagem da "fera", não obstante não ser dono da verdade, foi
>>> bola fora.
>>>
>>> Saudações,
>>> PJMS
>>>
>>>
>>> Em qui, 23 de ago de 2018 às 15:50, Israel Meireles Chrisostomo <
>>> israelmchrisost...@gmail.com> escreveu:
>>>
 Assista a esse vídeo:
 https://www.youtube.com/watch?v=Cy3Vyl-jxpk

 Em qui, 23 de ago de 2018 às 14:09, Daniel Quevedo 
 escreveu:

> Blz não sabia q era de lá, vou consultar. Obrigado!
>
> Em qui, 23 de ago de 2018 às 10:30, Claudio Buffara <
> claudio.buff...@gmail.com> escreveu:
>
>> Esse é clássico. Foi o problema 6 da IMO de 1988 e é usualmente
>> considerado o problema mais difícil proposto numa IMO, pelo menos até
>> aquela data.
>>
>> Um bom ponto de partida pode ser este:
>> https://en.wikipedia.org/wiki/Vieta_jumping
>> Ou então: https://mks.mff.cuni.cz/kalva/imo.html
>>
>> []s,
>> Claudio.
>>
>>
>>
>> 2018-08-23 9:57 GMT-03:00 Daniel Quevedo :
>>
>>> Sejam a e b inteiros estritamente positivos tais que (ab + 1) é um
>>> divisor de (a^2 + b^2). Sobre o número  (a^2 +b^2)/(ab +1) podemos 
>>> afirmar
>>> que é um quadrado perfeito:
>>> A) se, e só se, a e b também o forem.
>>> B) se, e só se, a e b tiverem acreana paridade
>>> C) se, e só se, a e b tiverem paridades distintas
>>> D) somente para um número finito de valores de a e b
>>> E) sempre
>>>
>>> R: e
>>>
>> --
>>> Fiscal: Daniel Quevedo
>>>
>>
>>> --
>>> Esta mensagem foi verificada pelo sistema de antivírus e
>>> acredita-se estar livre de perigo.
>>
>>
>>
>> --
>> Esta mensagem foi verificada pelo sistema de antivírus e
>> acredita-se estar livre de perigo.
>
> --
> Fiscal: Daniel Quevedo
>
> --
> Esta mensagem foi verificada pelo 

[obm-l] Re: [obm-l] Re: [obm-l] Re: [obm-l] Re: [obm-l] Teoria dos números

2018-11-28 Por tôpico Pedro José
Boa tarde!
Preciso de ajuda.
Após pensar mais um pouco é bem razoável que dada uma solução hipotética e
se consiga provar que há uma menor, que seja um absurdo. Absurdo no
sentido, que não há solução. Gostaria até que me sugerissem material
didático sobre o tópico.
Não obstante existe solução para a1 e
x inteiro.
Então há um paradoxo. Que por um lado se a é solução para a=b e após
estudar o caso a=b, ficamos com a restrição a>b, que é usada para provar
que a1b. Só que:
a1=(b^2-k)/ab.
Então esse é o ponto a1 mesmo sendo maior que zero,  não é solução pois
a1 escreveu:

> Bom dia!
> Refiro-me a solução recomendada por Israel.
> A princípio eu encrenquei com a solução. Pois, sem nenhuma caracterização
> definiu que a era mínimo. Então achei que a solução para a  capenga. Mas como não houve nenhuma crítica, julguei ser cisma minha. Mas
> depois me veio o pensamento, usando a técnica usada na resolução sempre que
> houver duas soluções(digo duas mesmo, distintas) haveria um absurdo. Pois
> ele supôs que a era mínimo e provou que a1, solução, a1 como ele não usou nenhum argumento para supor que a era mínimo, apenas
> arbitrou, poderia ter arbitrado que a era máximo e se a1>a, também seria
> absurdo.
> Aí, encrenquei mesmo com a soluçao e achei essa família de soluções para
> a K=x^2; b=x^3 e a =x^5-x, que para x>1, xinteiro, atende a k=(a^2+b^2)/(ab+1); continua dando um quadrado perfeito, mas se não fosse?
> A linha de argumento da solução,  desprezou essa possibilidade.
> Preciso ajuda, estou correto ou errado?
> Grato,
> PJMS
>
> Em Seg, 27 de ago de 2018 11:01, Pedro José 
> escreveu:
>
>> Bom dia!
>>
>> Linda solução pela simplicidade de ferramentas utilizadas.
>> Todavia, creio eu que não foi de bom alvitre utilizar a imagem de um
>> matemático famoso e divulgar que ele só ganhou um ponto na questão.
>> A mensagem, não explícita, mas é uma mensagem:"Ele não resolveu mas eu
>> sim."
>> As condições de resolução são totalmente diversas. Inclusive, devido às
>> questões anteriores ele já pode ter chegado a essa com o ponterinho do
>> relógio pendurado.
>> A solução do problema, mesmo tardia, sem a carga emocional que uma IMO
>> deve impor aos seus participantes, ainda é carregada de méritos, e na minha
>> visão, essa em particular, com uma beleza maior por ser sutil e singela.
>> Mas usar a imagem da "fera", não obstante não ser dono da verdade, foi
>> bola fora.
>>
>> Saudações,
>> PJMS
>>
>>
>> Em qui, 23 de ago de 2018 às 15:50, Israel Meireles Chrisostomo <
>> israelmchrisost...@gmail.com> escreveu:
>>
>>> Assista a esse vídeo:
>>> https://www.youtube.com/watch?v=Cy3Vyl-jxpk
>>>
>>> Em qui, 23 de ago de 2018 às 14:09, Daniel Quevedo 
>>> escreveu:
>>>
 Blz não sabia q era de lá, vou consultar. Obrigado!

 Em qui, 23 de ago de 2018 às 10:30, Claudio Buffara <
 claudio.buff...@gmail.com> escreveu:

> Esse é clássico. Foi o problema 6 da IMO de 1988 e é usualmente
> considerado o problema mais difícil proposto numa IMO, pelo menos até
> aquela data.
>
> Um bom ponto de partida pode ser este:
> https://en.wikipedia.org/wiki/Vieta_jumping
> Ou então: https://mks.mff.cuni.cz/kalva/imo.html
>
> []s,
> Claudio.
>
>
>
> 2018-08-23 9:57 GMT-03:00 Daniel Quevedo :
>
>> Sejam a e b inteiros estritamente positivos tais que (ab + 1) é um
>> divisor de (a^2 + b^2). Sobre o número  (a^2 +b^2)/(ab +1) podemos 
>> afirmar
>> que é um quadrado perfeito:
>> A) se, e só se, a e b também o forem.
>> B) se, e só se, a e b tiverem acreana paridade
>> C) se, e só se, a e b tiverem paridades distintas
>> D) somente para um número finito de valores de a e b
>> E) sempre
>>
>> R: e
>>
> --
>> Fiscal: Daniel Quevedo
>>
>
>> --
>> Esta mensagem foi verificada pelo sistema de antivírus e
>> acredita-se estar livre de perigo.
>
>
>
> --
> Esta mensagem foi verificada pelo sistema de antivírus e
> acredita-se estar livre de perigo.

 --
 Fiscal: Daniel Quevedo

 --
 Esta mensagem foi verificada pelo sistema de antivírus e
 acredita-se estar livre de perigo.
>>>
>>>
>>>
>>> --
>>> Israel Meireles Chrisostomo
>>>
>>> --
>>> Esta mensagem foi verificada pelo sistema de antivírus e
>>> acredita-se estar livre de perigo.
>>
>>

-- 
Esta mensagem foi verificada pelo sistema de antiv�rus e
 acredita-se estar livre de perigo.



[obm-l] Re: [obm-l] Re: [obm-l] Re: [obm-l] Re: [obm-l] Teoria dos números

2018-11-25 Por tôpico Pedro José
Bom dia!
Refiro-me a solução recomendada por Israel.
A princípio eu encrenquei com a solução. Pois, sem nenhuma caracterização
definiu que a era mínimo. Então achei que a solução para a a, também seria
absurdo.
Aí, encrenquei mesmo com a soluçao e achei essa família de soluções para
a1, xinteiro, atende a escreveu:

> Bom dia!
>
> Linda solução pela simplicidade de ferramentas utilizadas.
> Todavia, creio eu que não foi de bom alvitre utilizar a imagem de um
> matemático famoso e divulgar que ele só ganhou um ponto na questão.
> A mensagem, não explícita, mas é uma mensagem:"Ele não resolveu mas eu
> sim."
> As condições de resolução são totalmente diversas. Inclusive, devido às
> questões anteriores ele já pode ter chegado a essa com o ponterinho do
> relógio pendurado.
> A solução do problema, mesmo tardia, sem a carga emocional que uma IMO
> deve impor aos seus participantes, ainda é carregada de méritos, e na minha
> visão, essa em particular, com uma beleza maior por ser sutil e singela.
> Mas usar a imagem da "fera", não obstante não ser dono da verdade, foi
> bola fora.
>
> Saudações,
> PJMS
>
>
> Em qui, 23 de ago de 2018 às 15:50, Israel Meireles Chrisostomo <
> israelmchrisost...@gmail.com> escreveu:
>
>> Assista a esse vídeo:
>> https://www.youtube.com/watch?v=Cy3Vyl-jxpk
>>
>> Em qui, 23 de ago de 2018 às 14:09, Daniel Quevedo 
>> escreveu:
>>
>>> Blz não sabia q era de lá, vou consultar. Obrigado!
>>>
>>> Em qui, 23 de ago de 2018 às 10:30, Claudio Buffara <
>>> claudio.buff...@gmail.com> escreveu:
>>>
 Esse é clássico. Foi o problema 6 da IMO de 1988 e é usualmente
 considerado o problema mais difícil proposto numa IMO, pelo menos até
 aquela data.

 Um bom ponto de partida pode ser este:
 https://en.wikipedia.org/wiki/Vieta_jumping
 Ou então: https://mks.mff.cuni.cz/kalva/imo.html

 []s,
 Claudio.



 2018-08-23 9:57 GMT-03:00 Daniel Quevedo :

> Sejam a e b inteiros estritamente positivos tais que (ab + 1) é um
> divisor de (a^2 + b^2). Sobre o número  (a^2 +b^2)/(ab +1) podemos afirmar
> que é um quadrado perfeito:
> A) se, e só se, a e b também o forem.
> B) se, e só se, a e b tiverem acreana paridade
> C) se, e só se, a e b tiverem paridades distintas
> D) somente para um número finito de valores de a e b
> E) sempre
>
> R: e
>
 --
> Fiscal: Daniel Quevedo
>

> --
> Esta mensagem foi verificada pelo sistema de antivírus e
> acredita-se estar livre de perigo.



 --
 Esta mensagem foi verificada pelo sistema de antivírus e
 acredita-se estar livre de perigo.
>>>
>>> --
>>> Fiscal: Daniel Quevedo
>>>
>>> --
>>> Esta mensagem foi verificada pelo sistema de antivírus e
>>> acredita-se estar livre de perigo.
>>
>>
>>
>> --
>> Israel Meireles Chrisostomo
>>
>> --
>> Esta mensagem foi verificada pelo sistema de antivírus e
>> acredita-se estar livre de perigo.
>
>

-- 
Esta mensagem foi verificada pelo sistema de antiv�rus e
 acredita-se estar livre de perigo.



[obm-l] Re: [obm-l] Re: [obm-l] Re: [obm-l] Re: [obm-l] Teoria dos números

2018-08-27 Por tôpico Pedro José
Bom dia!

Linda solução pela simplicidade de ferramentas utilizadas.
Todavia, creio eu que não foi de bom alvitre utilizar a imagem de um
matemático famoso e divulgar que ele só ganhou um ponto na questão.
A mensagem, não explícita, mas é uma mensagem:"Ele não resolveu mas eu sim."
As condições de resolução são totalmente diversas. Inclusive, devido às
questões anteriores ele já pode ter chegado a essa com o ponterinho do
relógio pendurado.
A solução do problema, mesmo tardia, sem a carga emocional que uma IMO deve
impor aos seus participantes, ainda é carregada de méritos, e na minha
visão, essa em particular, com uma beleza maior por ser sutil e singela.
Mas usar a imagem da "fera", não obstante não ser dono da verdade, foi bola
fora.

Saudações,
PJMS


Em qui, 23 de ago de 2018 às 15:50, Israel Meireles Chrisostomo <
israelmchrisost...@gmail.com> escreveu:

> Assista a esse vídeo:
> https://www.youtube.com/watch?v=Cy3Vyl-jxpk
>
> Em qui, 23 de ago de 2018 às 14:09, Daniel Quevedo 
> escreveu:
>
>> Blz não sabia q era de lá, vou consultar. Obrigado!
>>
>> Em qui, 23 de ago de 2018 às 10:30, Claudio Buffara <
>> claudio.buff...@gmail.com> escreveu:
>>
>>> Esse é clássico. Foi o problema 6 da IMO de 1988 e é usualmente
>>> considerado o problema mais difícil proposto numa IMO, pelo menos até
>>> aquela data.
>>>
>>> Um bom ponto de partida pode ser este:
>>> https://en.wikipedia.org/wiki/Vieta_jumping
>>> Ou então: https://mks.mff.cuni.cz/kalva/imo.html
>>>
>>> []s,
>>> Claudio.
>>>
>>>
>>>
>>> 2018-08-23 9:57 GMT-03:00 Daniel Quevedo :
>>>
 Sejam a e b inteiros estritamente positivos tais que (ab + 1) é um
 divisor de (a^2 + b^2). Sobre o número  (a^2 +b^2)/(ab +1) podemos afirmar
 que é um quadrado perfeito:
 A) se, e só se, a e b também o forem.
 B) se, e só se, a e b tiverem acreana paridade
 C) se, e só se, a e b tiverem paridades distintas
 D) somente para um número finito de valores de a e b
 E) sempre

 R: e

>>> --
 Fiscal: Daniel Quevedo

>>>
 --
 Esta mensagem foi verificada pelo sistema de antivírus e
 acredita-se estar livre de perigo.
>>>
>>>
>>>
>>> --
>>> Esta mensagem foi verificada pelo sistema de antivírus e
>>> acredita-se estar livre de perigo.
>>
>> --
>> Fiscal: Daniel Quevedo
>>
>> --
>> Esta mensagem foi verificada pelo sistema de antivírus e
>> acredita-se estar livre de perigo.
>
>
>
> --
> Israel Meireles Chrisostomo
>
> --
> Esta mensagem foi verificada pelo sistema de antivírus e
> acredita-se estar livre de perigo.

-- 
Esta mensagem foi verificada pelo sistema de antiv�rus e
 acredita-se estar livre de perigo.



[obm-l] Re: [obm-l] Re: [obm-l] Re: [obm-l] Teoria dos números

2018-08-23 Por tôpico Israel Meireles Chrisostomo
Assista a esse vídeo:
https://www.youtube.com/watch?v=Cy3Vyl-jxpk

Em qui, 23 de ago de 2018 às 14:09, Daniel Quevedo 
escreveu:

> Blz não sabia q era de lá, vou consultar. Obrigado!
>
> Em qui, 23 de ago de 2018 às 10:30, Claudio Buffara <
> claudio.buff...@gmail.com> escreveu:
>
>> Esse é clássico. Foi o problema 6 da IMO de 1988 e é usualmente
>> considerado o problema mais difícil proposto numa IMO, pelo menos até
>> aquela data.
>>
>> Um bom ponto de partida pode ser este:
>> https://en.wikipedia.org/wiki/Vieta_jumping
>> Ou então: https://mks.mff.cuni.cz/kalva/imo.html
>>
>> []s,
>> Claudio.
>>
>>
>>
>> 2018-08-23 9:57 GMT-03:00 Daniel Quevedo :
>>
>>> Sejam a e b inteiros estritamente positivos tais que (ab + 1) é um
>>> divisor de (a^2 + b^2). Sobre o número  (a^2 +b^2)/(ab +1) podemos afirmar
>>> que é um quadrado perfeito:
>>> A) se, e só se, a e b também o forem.
>>> B) se, e só se, a e b tiverem acreana paridade
>>> C) se, e só se, a e b tiverem paridades distintas
>>> D) somente para um número finito de valores de a e b
>>> E) sempre
>>>
>>> R: e
>>>
>> --
>>> Fiscal: Daniel Quevedo
>>>
>>
>>> --
>>> Esta mensagem foi verificada pelo sistema de antivírus e
>>> acredita-se estar livre de perigo.
>>
>>
>>
>> --
>> Esta mensagem foi verificada pelo sistema de antivírus e
>> acredita-se estar livre de perigo.
>
> --
> Fiscal: Daniel Quevedo
>
> --
> Esta mensagem foi verificada pelo sistema de antivírus e
> acredita-se estar livre de perigo.



-- 
Israel Meireles Chrisostomo

-- 
Esta mensagem foi verificada pelo sistema de antiv�rus e
 acredita-se estar livre de perigo.



[obm-l] Re: [obm-l] Re: [obm-l] Teoria dos números

2018-08-23 Por tôpico Daniel Quevedo
Blz não sabia q era de lá, vou consultar. Obrigado!

Em qui, 23 de ago de 2018 às 10:30, Claudio Buffara <
claudio.buff...@gmail.com> escreveu:

> Esse é clássico. Foi o problema 6 da IMO de 1988 e é usualmente
> considerado o problema mais difícil proposto numa IMO, pelo menos até
> aquela data.
>
> Um bom ponto de partida pode ser este:
> https://en.wikipedia.org/wiki/Vieta_jumping
> Ou então: https://mks.mff.cuni.cz/kalva/imo.html
>
> []s,
> Claudio.
>
>
>
> 2018-08-23 9:57 GMT-03:00 Daniel Quevedo :
>
>> Sejam a e b inteiros estritamente positivos tais que (ab + 1) é um
>> divisor de (a^2 + b^2). Sobre o número  (a^2 +b^2)/(ab +1) podemos afirmar
>> que é um quadrado perfeito:
>> A) se, e só se, a e b também o forem.
>> B) se, e só se, a e b tiverem acreana paridade
>> C) se, e só se, a e b tiverem paridades distintas
>> D) somente para um número finito de valores de a e b
>> E) sempre
>>
>> R: e
>>
> --
>> Fiscal: Daniel Quevedo
>>
>
>> --
>> Esta mensagem foi verificada pelo sistema de antivírus e
>> acredita-se estar livre de perigo.
>
>
>
> --
> Esta mensagem foi verificada pelo sistema de antivírus e
> acredita-se estar livre de perigo.

-- 
Fiscal: Daniel Quevedo

-- 
Esta mensagem foi verificada pelo sistema de antiv�rus e
 acredita-se estar livre de perigo.



[obm-l] Re: [obm-l] Teoria dos números

2018-08-23 Por tôpico Claudio Buffara
Esse é clássico. Foi o problema 6 da IMO de 1988 e é usualmente considerado
o problema mais difícil proposto numa IMO, pelo menos até aquela data.

Um bom ponto de partida pode ser este:
https://en.wikipedia.org/wiki/Vieta_jumping
Ou então: https://mks.mff.cuni.cz/kalva/imo.html

[]s,
Claudio.



2018-08-23 9:57 GMT-03:00 Daniel Quevedo :

> Sejam a e b inteiros estritamente positivos tais que (ab + 1) é um divisor
> de (a^2 + b^2). Sobre o número  (a^2 +b^2)/(ab +1) podemos afirmar que é um
> quadrado perfeito:
> A) se, e só se, a e b também o forem.
> B) se, e só se, a e b tiverem acreana paridade
> C) se, e só se, a e b tiverem paridades distintas
> D) somente para um número finito de valores de a e b
> E) sempre
>
> R: e
> --
> Fiscal: Daniel Quevedo
>
> --
> Esta mensagem foi verificada pelo sistema de antivírus e
> acredita-se estar livre de perigo.

-- 
Esta mensagem foi verificada pelo sistema de antiv�rus e
 acredita-se estar livre de perigo.



[obm-l] Re: [obm-l] Re: [obm-l] Re: [obm-l] Re: [obm-l] Re: [obm-l] Teoria dos números

2018-06-02 Por tôpico Claudio Buffara
Eu acho que o enunciado pede a soma dos elementos simplesmente porque é uma
questão de múltipla escolha.
Já vi isso antes.

E perguntei a proveniência porque me parece muito difícil para ser uma
questão de vestibular. Talvez do ITA ou da OBM (1a fase)...

***

Sobre as soluções, acho interessante que certos "preconceitos" nos levam a
caminhos totalmente distintos.

Por exemplo, ao ver aquela expressão horrorosa, com X, X5 e X5^5, a
primeira coisa que me ocorreu foi resolver numericamente, com uma planilha.
Daí me pareceu natural procurar uma cota inferior e uma cota superior pra
X, já que a planilha é finita...
Foi aí que me ocorreu a ideia de mudar a notação pra N e R, que facilitou
muito a obtenção das cotas.
Tendo obtido a solução numericamente e redigido a mensagem, dei o caso por
encerrado.
Simplesmente não me ocorreu a ideia de manipular algebricamente o N e o R
pra chegar numa expressão mais reveladora, que foi o caminho que o Bernardo
seguiu, e conseguiu resolver o problema sem planilha.

Enfim, acho interessantíssimo este tema de como surgem as idéias na
matemática, mesmo num problema elementar (mas não trivial) como esse.

***

Com relação à observação do Pedro José, me parece que existe uma diferença
entre o "conjunto solução" e "as raízes" de uma equação.

Como, em se tratando de conjuntos, {1} = {1,1,1}, eu diria que, em relação
à equação x^3 - 3x^2 + 3x - 1 = 0, por exemplo,
o conjunto solução é {1} (igual a {1,1,1}) e a soma das raízes é 3.
Ou seja, esta equação tem 3 raízes (iguais) e um conjunto solução unitário.

Sobre retas coincidentes serem paralelas, acho que esta convenção permite
que paralelismo seja uma relação de equivalência.
E ser expulso por causa disso...ou o professor era um idiota ou, num ímpeto
de adolescente, você usou um linguajar forte demais pra defender sua
posição...

Tem uma passagem no Curso de Análise, do Elon, da qual eu nunca me esqueci:
"No caso de conjuntos, escrever A = B significa que A e B são o mesmo
conjunto, ou seja, que A e B possuem os mesmos elementos." Em outras
palavras, A e B são nomes diferentes para a mesma entidade.


[]s,
Claudio.


2018-06-02 18:08 GMT-03:00 Pedro José :

> Boa noite!
> O que também achei legal nesse problema foi o fato do questionamento ser
> quanto a soma dos elementos do conjunto  solução. Embora bem sutil,
> filosoficamente falando é forte. Pois, ela descarta a interpretação de n
> raízes iguais ao invés de uma raiz de multiplicidade n.
> Todas demonstrações de que algo era único, suponha-se que havia dois e se
> provava, por absudo, que eram iguais. Então se são iguais é único.
> Fui expulso de sala de aula, uma feita, no científico, pois, argumentei
> com o professor que não estava correto o conceito de posição relativa de
> duas retas, chamando-as de duas retas paralelas coincidentes. É uma reta só
> e ela é paralela a ela mesma. Nesse exercício a raiz um tem multiplicidade
> 2, e 0 tem multiplicidade 6, mas não interfere na soma. As demais raízes
> têm multiplicidade 1.
> Para os que seguem que a soma das raízes de um polinômios é
> -An-1/An ao invés de o somatório do produto das raízes pela sua
> multiplicidade, daria 1301, o que, considero incorreto.
> Desculpem-me pela lamúria, mas até hoje não me conformo com n entidades
> iguais com n >1. Talvez a expulsão tenha gerado um trauma.
> Saudações,
> PJMS
>
>
> Em Sáb, 2 de jun de 2018 17:29, Claudio Buffara 
> escreveu:
>
>> De onde é este problema?
>> 1a fase de alguma olimpíada?
>>
>> Abs
>>
>> Enviado do meu iPhone
>>
>> Em 2 de jun de 2018, à(s) 16:15, Daniel Quevedo 
>> escreveu:
>>
>> Muito obrigado a todos. De fato com a mudança de variável fica td mais
>> fácil. Não tinha visto isso.Â
>> Obrigado
>>
>> Em sáb, 2 de jun de 2018 às 16:02, Pedro José 
>> escreveu:
>>
>>> Boa tarde.
>>> A limitação para X5 obviamente não inclui 5, foi lambança.Â
>>> Saudações,Â
>>> PJMSÂ
>>>
>>> Em Sáb, 2 de jun de 2018 15:22, Claudio Buffara <
>>> claudio.buff...@gmail.com> escreveu:
>>>
 Para |X| suficientemente grande, X^6 domina a soma dos outros termos.

 Mudando a notação, eu pus N = X e R = X5.

 Então: R^5*N^5 + R*N = N^6 + R^6.

 Caso 1: N >= 0:
 R <= 4, de modo que o lado esquerdo <= N*(1024*N^4 + 4).
 Já o lado direito >= N^6.
 N*(1024*N^4 + 4) < N^6Â
 ==> 1024*N^4 + 4 < N^5Â Â
 ==> 1024 + 4/N^4 < N
 ==> N >= 1025.
 Então, para a equação ser satisfeita, é necessário que N <= 1024.

 Caso 2: N < 0.
 Então o lado esquerdo <= 0 (com igualdade sss R = 0) e o lado direito
 é positivo.
 Logo, a equação não tem soluções com N < 0.

 Com uma planilha, eu achei apenas 5 soluções:
 0, 1, 32, 243, 1024.

 A soma destes três números é 1300.

 []s,
 Claudio.


 2018-06-02 14:10 GMT-03:00 Daniel Quevedo :

> Seja Xn o resto da divisão de X por n. ParavX inteiro a soma de todos
> os elementos do conjunto solução da 

[obm-l] Re: [obm-l] Re: [obm-l] Re: [obm-l] Re: [obm-l] Teoria dos números

2018-06-02 Por tôpico Pedro José
Boa noite!
O que também achei legal nesse problema foi o fato do questionamento ser
quanto a soma dos elementos do conjunto  solução. Embora bem sutil,
filosoficamente falando é forte. Pois, ela descarta a interpretação de n
raízes iguais ao invés de uma raiz de multiplicidade n.
Todas demonstrações de que algo era único, suponha-se que havia dois e se
provava, por absudo, que eram iguais. Então se são iguais é único.
Fui expulso de sala de aula, uma feita, no científico, pois, argumentei com
o professor que não estava correto o conceito de posição relativa de duas
retas, chamando-as de duas retas paralelas coincidentes. É uma reta só e
ela é paralela a ela mesma. Nesse exercício a raiz um tem multiplicidade 2,
e 0 tem multiplicidade 6, mas não interfere na soma. As demais raízes têm
multiplicidade 1.
Para os que seguem que a soma das raízes de um polinômios é
-An-1/An ao invés de o somatório do produto das raízes pela sua
multiplicidade, daria 1301, o que, considero incorreto.
Desculpem-me pela lamúria, mas até hoje não me conformo com n entidades
iguais com n >1. Talvez a expulsão tenha gerado um trauma.
Saudações,
PJMS

Em Sáb, 2 de jun de 2018 17:29, Claudio Buffara 
escreveu:

> De onde é este problema?
> 1a fase de alguma olimpíada?
>
> Abs
>
> Enviado do meu iPhone
>
> Em 2 de jun de 2018, à(s) 16:15, Daniel Quevedo 
> escreveu:
>
> Muito obrigado a todos. De fato com a mudança de variável fica td mais
> fácil. Não tinha visto isso.Â
> Obrigado
>
> Em sáb, 2 de jun de 2018 às 16:02, Pedro José 
> escreveu:
>
>> Boa tarde.
>> A limitação para X5 obviamente não inclui 5, foi lambança.Â
>> Saudações,Â
>> PJMSÂ
>>
>> Em Sáb, 2 de jun de 2018 15:22, Claudio Buffara <
>> claudio.buff...@gmail.com> escreveu:
>>
>>> Para |X| suficientemente grande, X^6 domina a soma dos outros termos.
>>>
>>> Mudando a notação, eu pus N = X e R = X5.
>>>
>>> Então: R^5*N^5 + R*N = N^6 + R^6.
>>>
>>> Caso 1: N >= 0:
>>> R <= 4, de modo que o lado esquerdo <= N*(1024*N^4 + 4).
>>> Já o lado direito >= N^6.
>>> N*(1024*N^4 + 4) < N^6Â
>>> ==> 1024*N^4 + 4 < N^5Â Â
>>> ==> 1024 + 4/N^4 < N
>>> ==> N >= 1025.
>>> Então, para a equação ser satisfeita, é necessário que N <= 1024.
>>>
>>> Caso 2: N < 0.
>>> Então o lado esquerdo <= 0 (com igualdade sss R = 0) e o lado direito
>>> é positivo.
>>> Logo, a equação não tem soluções com N < 0.
>>>
>>> Com uma planilha, eu achei apenas 5 soluções:
>>> 0, 1, 32, 243, 1024.
>>>
>>> A soma destes três números é 1300.
>>>
>>> []s,
>>> Claudio.
>>>
>>>
>>> 2018-06-02 14:10 GMT-03:00 Daniel Quevedo :
>>>
 Seja Xn o resto da divisão de X por n. ParavX inteiro a soma de todos
 os elementos do conjunto solução da equação: [(X5)^5].X^5 - X^6 -
 (X5)^6 +X.(X5) = 0
 É igual a:
 A) 1100
 B) 1300
 C) 1500
 D) 1700
 E) 1900

 R: b
 --
 Fiscal: Daniel Quevedo

 --
 Esta mensagem foi verificada pelo sistema de antivírus e
 acredita-se estar livre de perigo.
>>>
>>>
>>>
>>> --
>>> Esta mensagem foi verificada pelo sistema de antivírus e
>>> acredita-se estar livre de perigo.
>>
>>
>> --
>> Esta mensagem foi verificada pelo sistema de antivírus e
>> acredita-se estar livre de perigo.
>
> --
> Fiscal: Daniel Quevedo
>
> --
> Esta mensagem foi verificada pelo sistema de antivírus e
> acredita-se estar livre de perigo.
>
>
> --
> Esta mensagem foi verificada pelo sistema de antivírus e
> acredita-se estar livre de perigo.
>

-- 
Esta mensagem foi verificada pelo sistema de antiv�rus e
 acredita-se estar livre de perigo.



[obm-l] Re: [obm-l] Re: [obm-l] Re: [obm-l] Re: [obm-l] Teoria dos números

2018-06-02 Por tôpico Daniel Quevedo
De um livro q tenho. Não duvidaria q fosse d alguma olimpíada pq há muitas
questões q são tiradas daí. O nome é Problemas Selecionados de Matemática,
do Gandhi

Em sáb, 2 de jun de 2018 às 17:29, Claudio Buffara <
claudio.buff...@gmail.com> escreveu:

> De onde é este problema?
> 1a fase de alguma olimpíada?
>
> Abs
>
> Enviado do meu iPhone
>
> Em 2 de jun de 2018, à(s) 16:15, Daniel Quevedo 
> escreveu:
>
> Muito obrigado a todos. De fato com a mudança de variável fica td mais
> fácil. Não tinha visto isso.Â
> Obrigado
>
> Em sáb, 2 de jun de 2018 às 16:02, Pedro José 
> escreveu:
>
>> Boa tarde.
>> A limitação para X5 obviamente não inclui 5, foi lambança.Â
>> Saudações,Â
>> PJMSÂ
>>
>> Em Sáb, 2 de jun de 2018 15:22, Claudio Buffara <
>> claudio.buff...@gmail.com> escreveu:
>>
>>> Para |X| suficientemente grande, X^6 domina a soma dos outros termos.
>>>
>>> Mudando a notação, eu pus N = X e R = X5.
>>>
>>> Então: R^5*N^5 + R*N = N^6 + R^6.
>>>
>>> Caso 1: N >= 0:
>>> R <= 4, de modo que o lado esquerdo <= N*(1024*N^4 + 4).
>>> Já o lado direito >= N^6.
>>> N*(1024*N^4 + 4) < N^6Â
>>> ==> 1024*N^4 + 4 < N^5Â Â
>>> ==> 1024 + 4/N^4 < N
>>> ==> N >= 1025.
>>> Então, para a equação ser satisfeita, é necessário que N <= 1024.
>>>
>>> Caso 2: N < 0.
>>> Então o lado esquerdo <= 0 (com igualdade sss R = 0) e o lado direito
>>> é positivo.
>>> Logo, a equação não tem soluções com N < 0.
>>>
>>> Com uma planilha, eu achei apenas 5 soluções:
>>> 0, 1, 32, 243, 1024.
>>>
>>> A soma destes três números é 1300.
>>>
>>> []s,
>>> Claudio.
>>>
>>>
>>> 2018-06-02 14:10 GMT-03:00 Daniel Quevedo :
>>>
 Seja Xn o resto da divisão de X por n. ParavX inteiro a soma de todos
 os elementos do conjunto solução da equação: [(X5)^5].X^5 - X^6 -
 (X5)^6 +X.(X5) = 0
 É igual a:
 A) 1100
 B) 1300
 C) 1500
 D) 1700
 E) 1900

 R: b
 --
 Fiscal: Daniel Quevedo

 --
 Esta mensagem foi verificada pelo sistema de antivírus e
 acredita-se estar livre de perigo.
>>>
>>>
>>>
>>> --
>>> Esta mensagem foi verificada pelo sistema de antivírus e
>>> acredita-se estar livre de perigo.
>>
>>
>> --
>> Esta mensagem foi verificada pelo sistema de antivírus e
>> acredita-se estar livre de perigo.
>
> --
> Fiscal: Daniel Quevedo
>
> --
> Esta mensagem foi verificada pelo sistema de antivírus e
> acredita-se estar livre de perigo.
>
>
> --
> Esta mensagem foi verificada pelo sistema de antivírus e
> acredita-se estar livre de perigo.
>
-- 
Fiscal: Daniel Quevedo

-- 
Esta mensagem foi verificada pelo sistema de antiv�rus e
 acredita-se estar livre de perigo.



Re: [obm-l] Re: [obm-l] Re: [obm-l] Re: [obm-l] Teoria dos números

2018-06-02 Por tôpico Claudio Buffara
De onde é este problema?
1a fase de alguma olimpíada?

Abs

Enviado do meu iPhone

Em 2 de jun de 2018, à(s) 16:15, Daniel Quevedo  escreveu:

> Muito obrigado a todos. De fato com a mudança de variável fica td mais 
> fácil. Não tinha visto isso. 
> Obrigado
> 
> Em sáb, 2 de jun de 2018 às 16:02, Pedro José  
> escreveu:
>> Boa tarde.
>> A limitação para X5 obviamente não inclui 5, foi lambança. 
>> Saudações, 
>> PJMSÂ 
>> 
>> Em Sáb, 2 de jun de 2018 15:22, Claudio Buffara  
>> escreveu:
>>> Para |X| suficientemente grande, X^6 domina a soma dos outros termos.
>>> 
>>> Mudando a notação, eu pus N = X e R = X5.
>>> 
>>> Então: R^5*N^5 + R*N = N^6 + R^6.
>>> 
>>> Caso 1: N >= 0:
>>> R <= 4, de modo que o lado esquerdo <= N*(1024*N^4 + 4).
>>> Já o lado direito >= N^6.
>>> N*(1024*N^4 + 4) < N^6Â 
>>> ==> 1024*N^4 + 4 < N^5Â Â 
>>> ==> 1024 + 4/N^4 < N
>>> ==> N >= 1025.
>>> Então, para a equação ser satisfeita, é necessário que N <= 1024.
>>> 
>>> Caso 2: N < 0.
>>> Então o lado esquerdo <= 0 (com igualdade sss R = 0) e o lado direito é 
>>> positivo.
>>> Logo, a equação não tem soluções com N < 0.
>>> 
>>> Com uma planilha, eu achei apenas 5 soluções:
>>> 0, 1, 32, 243, 1024.
>>> 
>>> A soma destes três números é 1300.
>>> 
>>> []s,
>>> Claudio.
>>> 
>>> 
>>> 2018-06-02 14:10 GMT-03:00 Daniel Quevedo :
 Seja Xn o resto da divisão de X por n. ParavX inteiro a soma de todos os 
 elementos do conjunto solução da equação: [(X5)^5].X^5 - X^6 - (X5)^6 
 +X.(X5) = 0
 É igual a:
 A) 1100
 B) 1300
 C) 1500
 D) 1700
 E) 1900
 
 R: b
 -- 
 Fiscal: Daniel Quevedo
 
 -- 
 Esta mensagem foi verificada pelo sistema de antivírus e 
 acredita-se estar livre de perigo.
>>> 
>>> 
>>> -- 
>>> Esta mensagem foi verificada pelo sistema de antivírus e 
>>> acredita-se estar livre de perigo.
>> 
>> -- 
>> Esta mensagem foi verificada pelo sistema de antivírus e 
>> acredita-se estar livre de perigo.
> -- 
> Fiscal: Daniel Quevedo
> 
> -- 
> Esta mensagem foi verificada pelo sistema de antivírus e 
> acredita-se estar livre de perigo.

-- 
Esta mensagem foi verificada pelo sistema de antiv�rus e
 acredita-se estar livre de perigo.



[obm-l] Re: [obm-l] Re: [obm-l] Re: [obm-l] Teoria dos números

2018-06-02 Por tôpico Daniel Quevedo
Muito obrigado a todos. De fato com a mudança de variável fica td mais
fácil. Não tinha visto isso.
Obrigado

Em sáb, 2 de jun de 2018 às 16:02, Pedro José 
escreveu:

> Boa tarde.
> A limitação para X5 obviamente não inclui 5, foi lambança.
> Saudações,
> PJMS
>
> Em Sáb, 2 de jun de 2018 15:22, Claudio Buffara 
> escreveu:
>
>> Para |X| suficientemente grande, X^6 domina a soma dos outros termos.
>>
>> Mudando a notação, eu pus N = X e R = X5.
>>
>> Então: R^5*N^5 + R*N = N^6 + R^6.
>>
>> Caso 1: N >= 0:
>> R <= 4, de modo que o lado esquerdo <= N*(1024*N^4 + 4).
>> Já o lado direito >= N^6.
>> N*(1024*N^4 + 4) < N^6
>> ==> 1024*N^4 + 4 < N^5
>> ==> 1024 + 4/N^4 < N
>> ==> N >= 1025.
>> Então, para a equação ser satisfeita, é necessário que N <= 1024.
>>
>> Caso 2: N < 0.
>> Então o lado esquerdo <= 0 (com igualdade sss R = 0) e o lado direito é
>> positivo.
>> Logo, a equação não tem soluções com N < 0.
>>
>> Com uma planilha, eu achei apenas 5 soluções:
>> 0, 1, 32, 243, 1024.
>>
>> A soma destes três números é 1300.
>>
>> []s,
>> Claudio.
>>
>>
>> 2018-06-02 14:10 GMT-03:00 Daniel Quevedo :
>>
>>> Seja Xn o resto da divisão de X por n. ParavX inteiro a soma de todos os
>>> elementos do conjunto solução da equação: [(X5)^5].X^5 - X^6 - (X5)^6
>>> +X.(X5) = 0
>>> É igual a:
>>> A) 1100
>>> B) 1300
>>> C) 1500
>>> D) 1700
>>> E) 1900
>>>
>>> R: b
>>> --
>>> Fiscal: Daniel Quevedo
>>>
>>> --
>>> Esta mensagem foi verificada pelo sistema de antivírus e
>>> acredita-se estar livre de perigo.
>>
>>
>>
>> --
>> Esta mensagem foi verificada pelo sistema de antivírus e
>> acredita-se estar livre de perigo.
>
>
> --
> Esta mensagem foi verificada pelo sistema de antivírus e
> acredita-se estar livre de perigo.

-- 
Fiscal: Daniel Quevedo

-- 
Esta mensagem foi verificada pelo sistema de antiv�rus e
 acredita-se estar livre de perigo.



[obm-l] Re: [obm-l] Re: [obm-l] Teoria dos números

2018-06-02 Por tôpico Pedro José
Boa tarde!
A propósito, é necessária a verificação se X5 = X mod5.
Para o exemplo foi simples pois eram potências 5 de X5.
Mas em outras situações, poderia haver uma solução inteira em que X5<>X
mod5 e não atenderia o problema.
Saudações
PJMS


Em Sáb, 2 de jun de 2018 15:55, Pedro José  escreveu:

> Boa tarde.
> A limitação para X5 obviamente não inclui 5, foi lambança.
> Saudações,
> PJMS
>
> Em Sáb, 2 de jun de 2018 15:22, Claudio Buffara 
> escreveu:
>
>> Para |X| suficientemente grande, X^6 domina a soma dos outros termos.
>>
>> Mudando a notação, eu pus N = X e R = X5.
>>
>> Então: R^5*N^5 + R*N = N^6 + R^6.
>>
>> Caso 1: N >= 0:
>> R <= 4, de modo que o lado esquerdo <= N*(1024*N^4 + 4).
>> Já o lado direito >= N^6.
>> N*(1024*N^4 + 4) < N^6
>> ==> 1024*N^4 + 4 < N^5
>> ==> 1024 + 4/N^4 < N
>> ==> N >= 1025.
>> Então, para a equação ser satisfeita, é necessário que N <= 1024.
>>
>> Caso 2: N < 0.
>> Então o lado esquerdo <= 0 (com igualdade sss R = 0) e o lado direito é
>> positivo.
>> Logo, a equação não tem soluções com N < 0.
>>
>> Com uma planilha, eu achei apenas 5 soluções:
>> 0, 1, 32, 243, 1024.
>>
>> A soma destes três números é 1300.
>>
>> []s,
>> Claudio.
>>
>>
>> 2018-06-02 14:10 GMT-03:00 Daniel Quevedo :
>>
>>> Seja Xn o resto da divisão de X por n. ParavX inteiro a soma de todos os
>>> elementos do conjunto solução da equação: [(X5)^5].X^5 - X^6 - (X5)^6
>>> +X.(X5) = 0
>>> É igual a:
>>> A) 1100
>>> B) 1300
>>> C) 1500
>>> D) 1700
>>> E) 1900
>>>
>>> R: b
>>> --
>>> Fiscal: Daniel Quevedo
>>>
>>> --
>>> Esta mensagem foi verificada pelo sistema de antivírus e
>>> acredita-se estar livre de perigo.
>>
>>
>>
>> --
>> Esta mensagem foi verificada pelo sistema de antivírus e
>> acredita-se estar livre de perigo.
>
>

-- 
Esta mensagem foi verificada pelo sistema de antiv�rus e
 acredita-se estar livre de perigo.



[obm-l] Re: [obm-l] Re: [obm-l] Teoria dos números

2018-06-02 Por tôpico Pedro José
Boa tarde.
A limitação para X5 obviamente não inclui 5, foi lambança.
Saudações,
PJMS

Em Sáb, 2 de jun de 2018 15:22, Claudio Buffara 
escreveu:

> Para |X| suficientemente grande, X^6 domina a soma dos outros termos.
>
> Mudando a notação, eu pus N = X e R = X5.
>
> Então: R^5*N^5 + R*N = N^6 + R^6.
>
> Caso 1: N >= 0:
> R <= 4, de modo que o lado esquerdo <= N*(1024*N^4 + 4).
> Já o lado direito >= N^6.
> N*(1024*N^4 + 4) < N^6
> ==> 1024*N^4 + 4 < N^5
> ==> 1024 + 4/N^4 < N
> ==> N >= 1025.
> Então, para a equação ser satisfeita, é necessário que N <= 1024.
>
> Caso 2: N < 0.
> Então o lado esquerdo <= 0 (com igualdade sss R = 0) e o lado direito é
> positivo.
> Logo, a equação não tem soluções com N < 0.
>
> Com uma planilha, eu achei apenas 5 soluções:
> 0, 1, 32, 243, 1024.
>
> A soma destes três números é 1300.
>
> []s,
> Claudio.
>
>
> 2018-06-02 14:10 GMT-03:00 Daniel Quevedo :
>
>> Seja Xn o resto da divisão de X por n. ParavX inteiro a soma de todos os
>> elementos do conjunto solução da equação: [(X5)^5].X^5 - X^6 - (X5)^6
>> +X.(X5) = 0
>> É igual a:
>> A) 1100
>> B) 1300
>> C) 1500
>> D) 1700
>> E) 1900
>>
>> R: b
>> --
>> Fiscal: Daniel Quevedo
>>
>> --
>> Esta mensagem foi verificada pelo sistema de antivírus e
>> acredita-se estar livre de perigo.
>
>
>
> --
> Esta mensagem foi verificada pelo sistema de antivírus e
> acredita-se estar livre de perigo.

-- 
Esta mensagem foi verificada pelo sistema de antiv�rus e
 acredita-se estar livre de perigo.



[obm-l] Re: [obm-l] Teoria dos números

2018-06-02 Por tôpico Pedro José
Boa tarde!
Temos uma limitação para X5, só pode ser 0, 1, 2, 3, 4 e 5.
Para zero não adianta que dá x=0, não contribui para soma.
Pode-se observar que não aceita raízes negativas, pois -X^6+X5^5*X^5 assume
um valor negativo muito elevado para valores <>-1 É não poderá ser zerado
pelas parcelas restantes.
As soluções inteiras possíveis para X são, 1, X5, X5^2, X5^3, X5^4, X5^5 e
X5^6, pois tem de dividir o termo independente do polinômios X5^6.
De cara para X5=1, X=1.
Para valores de X5 > 1, o valor
 |-X^6+X5^5.X^5| >> |X5.X-X5^6|  se o expoente de -X^6 não for igual ao
deX5^5. X^5.
Como X=X5^a temos  que a=5.
É note que esse valor também anula X5.X-X5^6. E portanto é a única raiz
inteira.
Como
K^5=k mod5 se mdc(k,5)=1, as raízes são 0, 1, 2^5, 3^5 e 4^5, cuja soma é
1.300.
Saudações,
PJMS

Em Sáb, 2 de jun de 2018 14:18, Daniel Quevedo 
escreveu:

> Seja Xn o resto da divisão de X por n. ParavX inteiro a soma de todos os
> elementos do conjunto solução da equação: [(X5)^5].X^5 - X^6 - (X5)^6
> +X.(X5) = 0
> É igual a:
> A) 1100
> B) 1300
> C) 1500
> D) 1700
> E) 1900
>
> R: b
> --
> Fiscal: Daniel Quevedo
>
> --
> Esta mensagem foi verificada pelo sistema de antivírus e
> acredita-se estar livre de perigo.

-- 
Esta mensagem foi verificada pelo sistema de antiv�rus e
 acredita-se estar livre de perigo.



[obm-l] Re: [obm-l] Re: [obm-l] Teoria dos números

2018-06-02 Por tôpico Bernardo Freitas Paulo da Costa
2018-06-02 15:14 GMT-03:00 Claudio Buffara :
> Mudando a notação, eu pus N = X e R = X5.
>
> Então: R^5*N^5 + R*N = N^6 + R^6.

Essa mudança de notação é o pulo do gato!  Daqui, um pouco de
tentativa e erro faz a seguinte dedução:

R^5 N^5 - R^6 = N^6 - RN
R^5(N^5 - R) = N(N^5 - R)
(N - R^5)(N^5 - R) = 0

Então ou N^5 = R, ou N = R^5.
A primeira dá R = 0 ou 1 (únicas potências quintas entre 0 e 4), e com
isso N = 0 ou 1
A segunda dá R = 0,1,2,3,4, e N = respectivas potências 5as.  Que são
justamente as soluções que você encontrou!

> Com uma planilha, eu achei apenas 5 soluções:
> 0, 1, 32, 243, 1024.

Abraços,
-- 
Bernardo Freitas Paulo da Costa

-- 
Esta mensagem foi verificada pelo sistema de antiv�rus e
 acredita-se estar livre de perigo.


=
Instru��es para entrar na lista, sair da lista e usar a lista em
http://www.mat.puc-rio.br/~obmlistas/obm-l.html
=


[obm-l] Re: [obm-l] Teoria dos números

2018-06-02 Por tôpico Claudio Buffara
Para |X| suficientemente grande, X^6 domina a soma dos outros termos.

Mudando a notação, eu pus N = X e R = X5.

Então: R^5*N^5 + R*N = N^6 + R^6.

Caso 1: N >= 0:
R <= 4, de modo que o lado esquerdo <= N*(1024*N^4 + 4).
Já o lado direito >= N^6.
N*(1024*N^4 + 4) < N^6
==> 1024*N^4 + 4 < N^5
==> 1024 + 4/N^4 < N
==> N >= 1025.
Então, para a equação ser satisfeita, é necessário que N <= 1024.

Caso 2: N < 0.
Então o lado esquerdo <= 0 (com igualdade sss R = 0) e o lado direito é
positivo.
Logo, a equação não tem soluções com N < 0.

Com uma planilha, eu achei apenas 5 soluções:
0, 1, 32, 243, 1024.

A soma destes três números é 1300.

[]s,
Claudio.


2018-06-02 14:10 GMT-03:00 Daniel Quevedo :

> Seja Xn o resto da divisão de X por n. ParavX inteiro a soma de todos os
> elementos do conjunto solução da equação: [(X5)^5].X^5 - X^6 - (X5)^6
> +X.(X5) = 0
> É igual a:
> A) 1100
> B) 1300
> C) 1500
> D) 1700
> E) 1900
>
> R: b
> --
> Fiscal: Daniel Quevedo
>
> --
> Esta mensagem foi verificada pelo sistema de antivírus e
> acredita-se estar livre de perigo.

-- 
Esta mensagem foi verificada pelo sistema de antiv�rus e
 acredita-se estar livre de perigo.



[obm-l] Re: [obm-l] Re: [obm-l] Re: [obm-l] Re: [obm-l] Re: [obm-l] Teoria dos números

2018-04-10 Por tôpico Pedro José
Bom dia!

Dei uma "roubadinha" e achei outra solução, pois veio de trás para a
frente. Veio da observação que nas respostas u=st.

(s-1)(t-1)(u-1) | ust-1 1=2 e só atende quando k(s,t,u) é
inteiro.
Fixando-se duas váriaveis  k é monótona decrescente para a outra; assim
kmax(s) = k(s,s+1,s+2)= (s(s+1)(s+2)-1)/(s-1)s(s+1)>=2, então
s(s+1)(s+2)/s(s-1)(s+1)>2; s < 4.

(s-1)(t-1)(u-1) | ust-1 então: (u-1) | ust-1, então: (u-1) | ust -st; então
(u-1) | st-1

temos que m (u-1) = st -1, (u-1) = (st-1)/m, com m inteiro. Então m < s,
para que u > t.

s=2, só serve m =1==> u=st. donde: (s-1)(t-1)(u-1) | (st)^2-1 ;
(s-1)(t-1)(u-1) | (u-1)(st+1); (s-1)(t-1) | (st+1)

(t-1)| | 2t+1; (t-1) | 3. t-1 <= 3 ==> t=4 (paridade -se uma das incógnitas
for par todas serão e se uma for ímpar todas serão - e t>s)  e u =st=8, que
verificando atende a proposição. (2,4,8)

s=3 há duas opções m=1 ou m=2

com m=1. (s-1)(t-1) | (st+1);  2(t-1) | 3t+1 ; 2(t-1) | t-1 <=4 ==> t=5
(paridade e t>s) c= st = 15 e atende a proposição (3,5,15)

com m=2 u= (st-1)/2 +1 ==> (s-1)(t-1) | ((st-1)/2 +1)st -1; 2(t-1) <=
(9t^2+3t)/2 -1; impossível.

Logo só há as soluções anteriores (2,4,8) e (3,5,15).

Saudações,
PJMS.

Em 26 de março de 2018 10:49, Pedro José  escreveu:

> Bom dia!
> Agora estou contente. Posso alardear que pelo menos matei um problema da
> IMO.
>
> (s-1)(t-1)(u-1) | ust-1 1 k(s,t,u) = (stu-1)/(s-1)(t-1)(u-1)
>
> Como k(s,t,u) > 1 temos que k(s,t,u) >=2 e só atende quando k(s,t,u) é
> inteiro.
> Fixando-se duas váriaveis  k é monótona decrescente para a outra; assim
> kmax(s) = k(s,s+1,s+2)= (s(s+1)(s+2)-1)/(s-1)s(s+1)>=2, então
> s(s+1)(s+2)/s(s-1)(s+1)>2; s < 4.
>
> fazendo um estudo de paridade: se uma das variáveis for par as outras duas
> também serão e k será ímpar. Se uma das variáveis for ímpar, todas serão
> ímpares e k poderá ser tanto ímpar quanto par.
>
> u   s   v  k
> P  P   P I
> III  -
>
> s=2. k>=3 Para kmax (2,t) = k(2,t,t+1) = (2t(t+1)-1)/t(t-1)>=3 então:
> 2t(t+1)/t(t-1) >3 : t < 5, pela paridade t=4 e kmax(2,4) = 47/15, só serve
> k = 3.
>
> s=2, t=4 e k=3 temos v=8. (2,4,8)
>
> s=3 k>=2 Para kmax (3,t) = k(3,t,t+1) = (3t(t+1)-1)/2t(t-1)>=2 então:
> 3t(t+1)/2t(t-1) >2 : t < 7, pela paridade t=5 e kmax(3,5) = 13/6, só serve
> k = 2.
>
> s=3, t= 5 e k=2 temos v= 15. (3,5,15)
>
> Só atendem: (2,4,8) e (3,5,15)
>
> Achei curioso que em ambas soluções, u=st.
>
> Saudações,
> PJMS
>
>
>
> Em 26 de março de 2018 09:44, Matheus Secco 
> escreveu:
>
>> De fato, trata-se do problema 1 da IMO 1992.
>>
>> Abs,
>>
>> Matheus Secco
>>
>> Em Seg, 26 de mar de 2018 09:24, Claudio Buffara <
>> claudio.buff...@gmail.com> escreveu:
>>
>>> Muito fácil pra ser de IMO...
>>>
>>> 2018-03-26 6:58 GMT-03:00 Anderson Torres 
>>> :
>>>
 Este não é o problema de alguma IMO não? Eu lembro de ter resolvido,
 quase igual à solução oficial: substituir s,t,u por a+1,b+1,c+1 e
 calcular os possiveis valores de
 1/a+1/b+1/c + 1/ab+1/ac+1/bc usando desigualdades - para daí limitar
 os valores de a,b,c.

 Em 23 de março de 2018 17:01, Claudio Buffara
  escreveu:
 > Enfim, nesse meio tempo acho que resolvi o problema...
 >
 > Devemos achar inteiros s, t, u, com 1 < s < t < u e tais que:
 > (stu -1)/((s-1)(t-1)(u-1)) = k  (k inteiro positivo)
 >
 > Após diversas aplicações do truque (método?) de somar e subtrair a
 mesma
 > coisa, chegamos a:
 > stu - 1 =  (s-1)(t-1)(u-1) + (s-1)(t-1) + (s-1)(u-1) + (t-1)(u-1) +
 (s-1) +
 > (t-1) + (u-1)
 >
 > Dividindo isso por (s-1)(t-1)(u-1), obtemos:
 > 1 + 1/(u-1) + 1/(t-1) + 1/(s-1) + 1/((t-1)(u-1)) + 1/((s-1)(u-1)) +
 > 1/((s-1)(t-1)) = k ==>
 >
 > 1/(u-1) + 1/(t-1) + 1/(s-1) + 1/((t-1)(u-1)) + 1/((s-1)(u-1)) +
 > 1/((s-1)(t-1)) = k-1
 >
 > Agora a ideia é achar cotas para s e para k.
 >
 > 1 < s < t < u ==> s >= 2, t >= 3 e u >= 4 ==> o lado esquerdo é menor
 ou
 > igual que:
 > 1/3 + 1/2 + 1 + 1/6 + 1/3 + 1/2 = 2+5/6
 >
 > Ou seja, como o lado esquerdo é inteiro (e positivo), só poderá ser
 igual a
 > 1 ou a 2 ==> k = 2 ou k = 3.
 >
 > Se s >= 4, então t >= 5 e u >= 6, e o lado esquerdo será, no máximo,
 igual
 > a:
 > 1/5 + 1/4 + 1/3 + 1/20 + 1/15 + 1/12 < 1.
 >
 > Logo, devemos ter s = 2 ou s = 3.
 >
 > s = 2 ==>
 > 1/(u-1) + 1/(t-1) + 1 + 1/((t-1)(u-1)) + 1/(u-1) + 1/(t-1) = k-1 ==>
 > 2/(t-1) + 2/(u-1) + 1/((t-1)(u-1)) = k-2 ==>
 > Como k-2 deve ser inteiro positivo, k só pode ser 3 e, portanto:
 > 2/(t-1) + 2/(u-1) + 1/((t-1)(u-1)) = 1 ==>
 > (2 + 1/(t-1))/(u-1) = 1 - 2/(t-1) ==>
 > u = 1 + (2t - 1)/(t - 3) = 3 + 5/(t-3) ==>
 > t = 4 e u = 8   ou   t = 8 e u = 4 (não serve pois t deve ser menor
 do que
 > u)
 >
 > s = 3 ==>
 > 1/(u-1) + 

[obm-l] Re: [obm-l] Re: [obm-l] Teoria dos números

2018-03-30 Por tôpico Pedro José
Bom dia!
Sua pergunta foi outra. Viajei.

Saudações,
PJMS

Em 29 de mar de 2018 11:10 PM, "Pedro José"  escreveu:

> Boa noite!
> Não consegui provar que se mdc(a,b)=1 Fi(ab)=Fi(a).Fi(b), para completar a
> demonstração.
> Porém pesquisando, encontrei essa pérola:
> A probabilidade de que um número inteiro d, 0< d <=m seja primo com m é
> igual a FI(m)/m.
> Se d é primo com m d <>0 mod p para todo p que divide m.
> Então a probabilidade de ser primo com p é (p-1)/p já que existem p restos
> possíveis da divisão euclidiana por p. Para atender todos p que dividem m
> segue o produtório. Como a probabilidade é Fi(m)/m, segue a fórmula.
> E dela dá para provar que Fi(ab)=Fi(a)Fi(b) se (a,b)=1.
> Já provar primeiro, para chegar na fórmula, não consegui.
>
> Saudações,
> PJMS.
>
>
> Em 29 de mar de 2018 22:30, "Pedro José"  escreveu:
>
>> Desculpe- me, não são divisores. São os únicos números que não são
>> co-primos de p^k.
>>
>> Em 29 de mar de 2018 22:25, "Pedro José"  escreveu:
>>
>>> Boa noite!
>>> Israel,
>>> você é detalhista.
>>> É fácil ver que se n = p^k, só haverá p^(k-1) divisores de p^k.
>>> Ou seja, d = m.p, onde 0>> -p^(k-1)=p^k.(1-1/p).
>>> Depois dá um pouquinho mais de trabalho. Que é provar que se mdc(a,b) =1
>>> então Fi(ab)=Fi(a).Fi(b).
>>>
>>> Saudações,
>>> PJMS.u
>>>
>>> Em 29 de mar de 2018 21:48, "Pedro José"  escreveu:
>>>
 Boa noite!
 Não tenho editor de símbolos. Portanto.
 Fi(n)= n . Produtório de ( p-1)/ p, onde p é primo e p divide n.

 Em 28 de mar de 2018 22:19, "Anderson Torres" <
 torres.anderson...@gmail.com> escreveu:

> Em 28 de março de 2018 21:24, Israel Meireles Chrisostomo
>  escreveu:
> > Existe alguma função na matemática que conta a quantidade de
> divisores
> > primos de um dado número n qualquer?Sabe-se que phi(n) -totiente-
> conta a
> > quantidade de números  primos menores ou iguais  a n então (n-
> phi(n)) é a
> > quantidade de divisores, certo?mas e a quantidade de divisores
> primos?
> >
>
> Existir, existe. Mas você espera o quê? Uma fórmula fácil? Isso seria
> meio insano, afinal muitas funções em teoria dos números dependem
> explicitamente da fatoração. Por exemplo, a phi de Euler depende que
> se saiba da fatoração, a contagem de divisores e a soma dos divisores
> também.
>
> > --
> > Israel Meireles Chrisostomo
> >
> > --
> > Esta mensagem foi verificada pelo sistema de antivírus e
> > acredita-se estar livre de perigo.
>
> --
> Esta mensagem foi verificada pelo sistema de antivírus e
>  acredita-se estar livre de perigo.
>
>
> 
> =
> Instru�ões para entrar na lista, sair da lista e usar a lista em
> http://www.mat.puc-rio.br/~obmlistas/obm-l.html
> 
> =
>


-- 
Esta mensagem foi verificada pelo sistema de antiv�rus e
 acredita-se estar livre de perigo.



[obm-l] Re: [obm-l] Re: [obm-l] Teoria dos números

2018-03-29 Por tôpico Pedro José
Boa noite!
Não consegui provar que se mdc(a,b)=1 Fi(ab)=Fi(a).Fi(b), para completar a
demonstração.
Porém pesquisando, encontrei essa pérola:
A probabilidade de que um número inteiro d, 0< d <=m seja primo com m é
igual a FI(m)/m.
Se d é primo com m d <>0 mod p para todo p que divide m.
Então a probabilidade de ser primo com p é (p-1)/p já que existem p restos
possíveis da divisão euclidiana por p. Para atender todos p que dividem m
segue o produtório. Como a probabilidade é Fi(m)/m, segue a fórmula.
E dela dá para provar que Fi(ab)=Fi(a)Fi(b) se (a,b)=1.
Já provar primeiro, para chegar na fórmula, não consegui.

Saudações,
PJMS.


Em 29 de mar de 2018 22:30, "Pedro José"  escreveu:

> Desculpe- me, não são divisores. São os únicos números que não são
> co-primos de p^k.
>
> Em 29 de mar de 2018 22:25, "Pedro José"  escreveu:
>
>> Boa noite!
>> Israel,
>> você é detalhista.
>> É fácil ver que se n = p^k, só haverá p^(k-1) divisores de p^k.
>> Ou seja, d = m.p, onde 0> -p^(k-1)=p^k.(1-1/p).
>> Depois dá um pouquinho mais de trabalho. Que é provar que se mdc(a,b) =1
>> então Fi(ab)=Fi(a).Fi(b).
>>
>> Saudações,
>> PJMS.u
>>
>> Em 29 de mar de 2018 21:48, "Pedro José"  escreveu:
>>
>>> Boa noite!
>>> Não tenho editor de símbolos. Portanto.
>>> Fi(n)= n . Produtório de ( p-1)/ p, onde p é primo e p divide n.
>>>
>>> Em 28 de mar de 2018 22:19, "Anderson Torres" <
>>> torres.anderson...@gmail.com> escreveu:
>>>
 Em 28 de março de 2018 21:24, Israel Meireles Chrisostomo
  escreveu:
 > Existe alguma função na matemática que conta a quantidade de divisores
 > primos de um dado número n qualquer?Sabe-se que phi(n) -totiente-
 conta a
 > quantidade de números  primos menores ou iguais  a n então (n-
 phi(n)) é a
 > quantidade de divisores, certo?mas e a quantidade de divisores primos?
 >

 Existir, existe. Mas você espera o quê? Uma fórmula fácil? Isso seria
 meio insano, afinal muitas funções em teoria dos números dependem
 explicitamente da fatoração. Por exemplo, a phi de Euler depende que
 se saiba da fatoração, a contagem de divisores e a soma dos divisores
 também.

 > --
 > Israel Meireles Chrisostomo
 >
 > --
 > Esta mensagem foi verificada pelo sistema de antivírus e
 > acredita-se estar livre de perigo.

 --
 Esta mensagem foi verificada pelo sistema de antivírus e
  acredita-se estar livre de perigo.


 
 =
 Instru�ões para entrar na lista, sair da lista e usar a lista em
 http://www.mat.puc-rio.br/~obmlistas/obm-l.html
 
 =

>>>

-- 
Esta mensagem foi verificada pelo sistema de antiv�rus e
 acredita-se estar livre de perigo.



[obm-l] Re: [obm-l] Re: [obm-l] Teoria dos números

2018-03-29 Por tôpico Pedro José
Desculpe- me, não são divisores. São os únicos números que não são
co-primos de p^k.

Em 29 de mar de 2018 22:25, "Pedro José"  escreveu:

> Boa noite!
> Israel,
> você é detalhista.
> É fácil ver que se n = p^k, só haverá p^(k-1) divisores de p^k.
> Ou seja, d = m.p, onde 0 Depois dá um pouquinho mais de trabalho. Que é provar que se mdc(a,b) =1
> então Fi(ab)=Fi(a).Fi(b).
>
> Saudações,
> PJMS.u
>
> Em 29 de mar de 2018 21:48, "Pedro José"  escreveu:
>
>> Boa noite!
>> Não tenho editor de símbolos. Portanto.
>> Fi(n)= n . Produtório de ( p-1)/ p, onde p é primo e p divide n.
>>
>> Em 28 de mar de 2018 22:19, "Anderson Torres" <
>> torres.anderson...@gmail.com> escreveu:
>>
>>> Em 28 de março de 2018 21:24, Israel Meireles Chrisostomo
>>>  escreveu:
>>> > Existe alguma função na matemática que conta a quantidade de divisores
>>> > primos de um dado número n qualquer?Sabe-se que phi(n) -totiente-
>>> conta a
>>> > quantidade de números  primos menores ou iguais  a n então (n- phi(n))
>>> é a
>>> > quantidade de divisores, certo?mas e a quantidade de divisores primos?
>>> >
>>>
>>> Existir, existe. Mas você espera o quê? Uma fórmula fácil? Isso seria
>>> meio insano, afinal muitas funções em teoria dos números dependem
>>> explicitamente da fatoração. Por exemplo, a phi de Euler depende que
>>> se saiba da fatoração, a contagem de divisores e a soma dos divisores
>>> também.
>>>
>>> > --
>>> > Israel Meireles Chrisostomo
>>> >
>>> > --
>>> > Esta mensagem foi verificada pelo sistema de antivírus e
>>> > acredita-se estar livre de perigo.
>>>
>>> --
>>> Esta mensagem foi verificada pelo sistema de antivírus e
>>>  acredita-se estar livre de perigo.
>>>
>>>
>>> 
>>> =
>>> Instru�ões para entrar na lista, sair da lista e usar a lista em
>>> http://www.mat.puc-rio.br/~obmlistas/obm-l.html
>>> 
>>> =
>>>
>>

-- 
Esta mensagem foi verificada pelo sistema de antiv�rus e
 acredita-se estar livre de perigo.



[obm-l] Re: [obm-l] Re: [obm-l] Teoria dos números

2018-03-29 Por tôpico Pedro José
Boa noite!
Israel,
você é detalhista.
É fácil ver que se n = p^k, só haverá p^(k-1) divisores de p^k.
Ou seja, d = m.p, onde 0 escreveu:

> Boa noite!
> Não tenho editor de símbolos. Portanto.
> Fi(n)= n . Produtório de ( p-1)/ p, onde p é primo e p divide n.
>
> Em 28 de mar de 2018 22:19, "Anderson Torres" <
> torres.anderson...@gmail.com> escreveu:
>
>> Em 28 de março de 2018 21:24, Israel Meireles Chrisostomo
>>  escreveu:
>> > Existe alguma função na matemática que conta a quantidade de divisores
>> > primos de um dado número n qualquer?Sabe-se que phi(n) -totiente- conta
>> a
>> > quantidade de números  primos menores ou iguais  a n então (n- phi(n))
>> é a
>> > quantidade de divisores, certo?mas e a quantidade de divisores primos?
>> >
>>
>> Existir, existe. Mas você espera o quê? Uma fórmula fácil? Isso seria
>> meio insano, afinal muitas funções em teoria dos números dependem
>> explicitamente da fatoração. Por exemplo, a phi de Euler depende que
>> se saiba da fatoração, a contagem de divisores e a soma dos divisores
>> também.
>>
>> > --
>> > Israel Meireles Chrisostomo
>> >
>> > --
>> > Esta mensagem foi verificada pelo sistema de antivírus e
>> > acredita-se estar livre de perigo.
>>
>> --
>> Esta mensagem foi verificada pelo sistema de antivírus e
>>  acredita-se estar livre de perigo.
>>
>>
>> =
>> Instru�ões para entrar na lista, sair da lista e usar a lista em
>> http://www.mat.puc-rio.br/~obmlistas/obm-l.html
>> =
>>
>

-- 
Esta mensagem foi verificada pelo sistema de antiv�rus e
 acredita-se estar livre de perigo.



[obm-l] Re: [obm-l] Re: [obm-l] Teoria dos números

2018-03-29 Por tôpico Pedro José
Boa noite!
Não tenho editor de símbolos. Portanto.
Fi(n)= n . Produtório de ( p-1)/ p, onde p é primo e p divide n.

Em 28 de mar de 2018 22:19, "Anderson Torres" 
escreveu:

> Em 28 de março de 2018 21:24, Israel Meireles Chrisostomo
>  escreveu:
> > Existe alguma função na matemática que conta a quantidade de divisores
> > primos de um dado número n qualquer?Sabe-se que phi(n) -totiente- conta a
> > quantidade de números  primos menores ou iguais  a n então (n- phi(n)) é
> a
> > quantidade de divisores, certo?mas e a quantidade de divisores primos?
> >
>
> Existir, existe. Mas você espera o quê? Uma fórmula fácil? Isso seria
> meio insano, afinal muitas funções em teoria dos números dependem
> explicitamente da fatoração. Por exemplo, a phi de Euler depende que
> se saiba da fatoração, a contagem de divisores e a soma dos divisores
> também.
>
> > --
> > Israel Meireles Chrisostomo
> >
> > --
> > Esta mensagem foi verificada pelo sistema de antivírus e
> > acredita-se estar livre de perigo.
>
> --
> Esta mensagem foi verificada pelo sistema de antivírus e
>  acredita-se estar livre de perigo.
>
>
> =
> Instru�ões para entrar na lista, sair da lista e usar a lista em
> http://www.mat.puc-rio.br/~obmlistas/obm-l.html
> =
>

-- 
Esta mensagem foi verificada pelo sistema de antiv�rus e
 acredita-se estar livre de perigo.



[obm-l] Re: [obm-l] Teoria dos números

2018-03-28 Por tôpico Anderson Torres
Em 28 de março de 2018 21:24, Israel Meireles Chrisostomo
 escreveu:
> Existe alguma função na matemática que conta a quantidade de divisores
> primos de um dado número n qualquer?Sabe-se que phi(n) -totiente- conta a
> quantidade de números  primos menores ou iguais  a n então (n- phi(n)) é a
> quantidade de divisores, certo?mas e a quantidade de divisores primos?
>

Existir, existe. Mas você espera o quê? Uma fórmula fácil? Isso seria
meio insano, afinal muitas funções em teoria dos números dependem
explicitamente da fatoração. Por exemplo, a phi de Euler depende que
se saiba da fatoração, a contagem de divisores e a soma dos divisores
também.

> --
> Israel Meireles Chrisostomo
>
> --
> Esta mensagem foi verificada pelo sistema de antivírus e
> acredita-se estar livre de perigo.

-- 
Esta mensagem foi verificada pelo sistema de antiv�rus e
 acredita-se estar livre de perigo.


=
Instru��es para entrar na lista, sair da lista e usar a lista em
http://www.mat.puc-rio.br/~obmlistas/obm-l.html
=


[obm-l] Re: [obm-l] Teoria dos números

2018-03-28 Por tôpico Claudio Buffara
Certamente existe. Basta definir f(n) = número de primos distintos que
dividem n.
Mas duvido que possa ser expressa por alguma fórmula simples.

n-phi(n) não é a quantidade de divisores.
Tome 10 por exemplo. Phi(10) = 4 ==> 10 - Phi(10) = 6, mas 10 tem apenas 4
divisores: 1, 2, 5 e 10.

2018-03-28 21:24 GMT-03:00 Israel Meireles Chrisostomo <
israelmchrisost...@gmail.com>:

> Existe alguma função na matemática que conta a quantidade de divisores
> primos de um dado número n qualquer?Sabe-se que phi(n) -totiente- conta a
> quantidade de números  primos menores ou iguais  a n então (n- phi(n)) é a
> quantidade de divisores, certo?mas e a quantidade de divisores primos?
>
> --
> Israel Meireles Chrisostomo
>
> --
> Esta mensagem foi verificada pelo sistema de antivírus e
> acredita-se estar livre de perigo.

-- 
Esta mensagem foi verificada pelo sistema de antiv�rus e
 acredita-se estar livre de perigo.



[obm-l] Re: [obm-l] Re: [obm-l] Re: [obm-l] Re: [obm-l] Re: [obm-l] Teoria dos números

2018-03-26 Por tôpico Pedro José
Bom dia!
Agora estou contente. Posso alardear que pelo menos matei um problema da
IMO.

(s-1)(t-1)(u-1) | ust-1 1=2 e só atende quando k(s,t,u) é
inteiro.
Fixando-se duas váriaveis  k é monótona decrescente para a outra; assim
kmax(s) = k(s,s+1,s+2)= (s(s+1)(s+2)-1)/(s-1)s(s+1)>=2, então
s(s+1)(s+2)/s(s-1)(s+1)>2; s < 4.

fazendo um estudo de paridade: se uma das variáveis for par as outras duas
também serão e k será ímpar. Se uma das variáveis for ímpar, todas serão
ímpares e k poderá ser tanto ímpar quanto par.

u   s   v  k
P  P   P I
III  -

s=2. k>=3 Para kmax (2,t) = k(2,t,t+1) = (2t(t+1)-1)/t(t-1)>=3 então:
2t(t+1)/t(t-1) >3 : t < 5, pela paridade t=4 e kmax(2,4) = 47/15, só serve
k = 3.

s=2, t=4 e k=3 temos v=8. (2,4,8)

s=3 k>=2 Para kmax (3,t) = k(3,t,t+1) = (3t(t+1)-1)/2t(t-1)>=2 então:
3t(t+1)/2t(t-1) >2 : t < 7, pela paridade t=5 e kmax(3,5) = 13/6, só serve
k = 2.

s=3, t= 5 e k=2 temos v= 15. (3,5,15)

Só atendem: (2,4,8) e (3,5,15)

Achei curioso que em ambas soluções, u=st.

Saudações,
PJMS



Em 26 de março de 2018 09:44, Matheus Secco 
escreveu:

> De fato, trata-se do problema 1 da IMO 1992.
>
> Abs,
>
> Matheus Secco
>
> Em Seg, 26 de mar de 2018 09:24, Claudio Buffara <
> claudio.buff...@gmail.com> escreveu:
>
>> Muito fácil pra ser de IMO...
>>
>> 2018-03-26 6:58 GMT-03:00 Anderson Torres :
>>
>>> Este não é o problema de alguma IMO não? Eu lembro de ter resolvido,
>>> quase igual à solução oficial: substituir s,t,u por a+1,b+1,c+1 e
>>> calcular os possiveis valores de
>>> 1/a+1/b+1/c + 1/ab+1/ac+1/bc usando desigualdades - para daí limitar
>>> os valores de a,b,c.
>>>
>>> Em 23 de março de 2018 17:01, Claudio Buffara
>>>  escreveu:
>>> > Enfim, nesse meio tempo acho que resolvi o problema...
>>> >
>>> > Devemos achar inteiros s, t, u, com 1 < s < t < u e tais que:
>>> > (stu -1)/((s-1)(t-1)(u-1)) = k  (k inteiro positivo)
>>> >
>>> > Após diversas aplicações do truque (método?) de somar e subtrair a
>>> mesma
>>> > coisa, chegamos a:
>>> > stu - 1 =  (s-1)(t-1)(u-1) + (s-1)(t-1) + (s-1)(u-1) + (t-1)(u-1) +
>>> (s-1) +
>>> > (t-1) + (u-1)
>>> >
>>> > Dividindo isso por (s-1)(t-1)(u-1), obtemos:
>>> > 1 + 1/(u-1) + 1/(t-1) + 1/(s-1) + 1/((t-1)(u-1)) + 1/((s-1)(u-1)) +
>>> > 1/((s-1)(t-1)) = k ==>
>>> >
>>> > 1/(u-1) + 1/(t-1) + 1/(s-1) + 1/((t-1)(u-1)) + 1/((s-1)(u-1)) +
>>> > 1/((s-1)(t-1)) = k-1
>>> >
>>> > Agora a ideia é achar cotas para s e para k.
>>> >
>>> > 1 < s < t < u ==> s >= 2, t >= 3 e u >= 4 ==> o lado esquerdo é menor
>>> ou
>>> > igual que:
>>> > 1/3 + 1/2 + 1 + 1/6 + 1/3 + 1/2 = 2+5/6
>>> >
>>> > Ou seja, como o lado esquerdo é inteiro (e positivo), só poderá ser
>>> igual a
>>> > 1 ou a 2 ==> k = 2 ou k = 3.
>>> >
>>> > Se s >= 4, então t >= 5 e u >= 6, e o lado esquerdo será, no máximo,
>>> igual
>>> > a:
>>> > 1/5 + 1/4 + 1/3 + 1/20 + 1/15 + 1/12 < 1.
>>> >
>>> > Logo, devemos ter s = 2 ou s = 3.
>>> >
>>> > s = 2 ==>
>>> > 1/(u-1) + 1/(t-1) + 1 + 1/((t-1)(u-1)) + 1/(u-1) + 1/(t-1) = k-1 ==>
>>> > 2/(t-1) + 2/(u-1) + 1/((t-1)(u-1)) = k-2 ==>
>>> > Como k-2 deve ser inteiro positivo, k só pode ser 3 e, portanto:
>>> > 2/(t-1) + 2/(u-1) + 1/((t-1)(u-1)) = 1 ==>
>>> > (2 + 1/(t-1))/(u-1) = 1 - 2/(t-1) ==>
>>> > u = 1 + (2t - 1)/(t - 3) = 3 + 5/(t-3) ==>
>>> > t = 4 e u = 8   ou   t = 8 e u = 4 (não serve pois t deve ser menor do
>>> que
>>> > u)
>>> >
>>> > s = 3 ==>
>>> > 1/(u-1) + 1/(t-1) + 1/2 + 1/((t-1)(u-1)) + 1/(2(u-1)) + 1/(2(t-1)) =
>>> k-1 ==>
>>> > (3/2)/(u-1) + (3/2)/(t-1) + 1/((t-1)(u-1)) = k - 3/2 ==>
>>> > 3/(u-1) + 3/(t-1) + 2/((t-1)(u-1)) = 2k - 3 ==>
>>> > (3 + 2/(t-1))/(u-1) = 2k - 3t/(t-1) ==>
>>> > (3t - 1)/(u-1) = 2k(t-1) - 3t ==>
>>> > u = 1 + (3t - 1)/((2k-3)t - 2k)
>>> >
>>> > k = 2 ==> u = 1 + (3t-1)/(t-4) = 4 + 11/(t-4) ==> t = 5 e u = 15
>>> >
>>> > k = 3 ==> u = 1 + (3t-1)/(3t-6) = 2 + 5/(3t-6) ==> XXX
>>> >
>>> > As únicas soluções são:
>>> > (2,4,8) e (3,5,15)
>>> >
>>> > []s,
>>> > Claudio.
>>> >
>>> > 2018-03-23 15:38 GMT-03:00 Pedro José :
>>> >>
>>> >> Boa tarde!
>>> >>
>>> >> Aproveitando que deu o que falar o problema postado pelo Douglas, tem
>>> um
>>> >> que achei mais interessante.
>>> >>
>>> >> (s-1)(t-1).(u-1) | stu -1, com s, t, u inteiros  e 1 > >>
>>> >> Saudações,
>>> >> Pedro
>>> >>
>>> >> --
>>> >> Esta mensagem foi verificada pelo sistema de antivírus e
>>> >> acredita-se estar livre de perigo.
>>> >
>>> >
>>> >
>>> > --
>>> > Esta mensagem foi verificada pelo sistema de antivírus e
>>> > acredita-se estar livre de perigo.
>>>
>>> --
>>> Esta mensagem foi verificada pelo sistema de antivírus e
>>>  acredita-se estar livre de perigo.
>>>
>>>
>>> 
>>> =
>>> Instru�ões para entrar na lista, sair da lista e usar a lista em
>>> http://www.mat.puc-rio.br/~obmlistas/obm-l.html
>>> 

[obm-l] Re: [obm-l] Re: [obm-l] Re: [obm-l] Re: [obm-l] Teoria dos números

2018-03-26 Por tôpico Matheus Secco
De fato, trata-se do problema 1 da IMO 1992.

Abs,

Matheus Secco

Em Seg, 26 de mar de 2018 09:24, Claudio Buffara 
escreveu:

> Muito fácil pra ser de IMO...
>
> 2018-03-26 6:58 GMT-03:00 Anderson Torres :
>
>> Este não é o problema de alguma IMO não? Eu lembro de ter resolvido,
>> quase igual à solução oficial: substituir s,t,u por a+1,b+1,c+1 e
>> calcular os possiveis valores de
>> 1/a+1/b+1/c + 1/ab+1/ac+1/bc usando desigualdades - para daí limitar
>> os valores de a,b,c.
>>
>> Em 23 de março de 2018 17:01, Claudio Buffara
>>  escreveu:
>> > Enfim, nesse meio tempo acho que resolvi o problema...
>> >
>> > Devemos achar inteiros s, t, u, com 1 < s < t < u e tais que:
>> > (stu -1)/((s-1)(t-1)(u-1)) = k  (k inteiro positivo)
>> >
>> > Após diversas aplicações do truque (método?) de somar e subtrair a mesma
>> > coisa, chegamos a:
>> > stu - 1 =  (s-1)(t-1)(u-1) + (s-1)(t-1) + (s-1)(u-1) + (t-1)(u-1) +
>> (s-1) +
>> > (t-1) + (u-1)
>> >
>> > Dividindo isso por (s-1)(t-1)(u-1), obtemos:
>> > 1 + 1/(u-1) + 1/(t-1) + 1/(s-1) + 1/((t-1)(u-1)) + 1/((s-1)(u-1)) +
>> > 1/((s-1)(t-1)) = k ==>
>> >
>> > 1/(u-1) + 1/(t-1) + 1/(s-1) + 1/((t-1)(u-1)) + 1/((s-1)(u-1)) +
>> > 1/((s-1)(t-1)) = k-1
>> >
>> > Agora a ideia é achar cotas para s e para k.
>> >
>> > 1 < s < t < u ==> s >= 2, t >= 3 e u >= 4 ==> o lado esquerdo é menor ou
>> > igual que:
>> > 1/3 + 1/2 + 1 + 1/6 + 1/3 + 1/2 = 2+5/6
>> >
>> > Ou seja, como o lado esquerdo é inteiro (e positivo), só poderá ser
>> igual a
>> > 1 ou a 2 ==> k = 2 ou k = 3.
>> >
>> > Se s >= 4, então t >= 5 e u >= 6, e o lado esquerdo será, no máximo,
>> igual
>> > a:
>> > 1/5 + 1/4 + 1/3 + 1/20 + 1/15 + 1/12 < 1.
>> >
>> > Logo, devemos ter s = 2 ou s = 3.
>> >
>> > s = 2 ==>
>> > 1/(u-1) + 1/(t-1) + 1 + 1/((t-1)(u-1)) + 1/(u-1) + 1/(t-1) = k-1 ==>
>> > 2/(t-1) + 2/(u-1) + 1/((t-1)(u-1)) = k-2 ==>
>> > Como k-2 deve ser inteiro positivo, k só pode ser 3 e, portanto:
>> > 2/(t-1) + 2/(u-1) + 1/((t-1)(u-1)) = 1 ==>
>> > (2 + 1/(t-1))/(u-1) = 1 - 2/(t-1) ==>
>> > u = 1 + (2t - 1)/(t - 3) = 3 + 5/(t-3) ==>
>> > t = 4 e u = 8   ou   t = 8 e u = 4 (não serve pois t deve ser menor do
>> que
>> > u)
>> >
>> > s = 3 ==>
>> > 1/(u-1) + 1/(t-1) + 1/2 + 1/((t-1)(u-1)) + 1/(2(u-1)) + 1/(2(t-1)) =
>> k-1 ==>
>> > (3/2)/(u-1) + (3/2)/(t-1) + 1/((t-1)(u-1)) = k - 3/2 ==>
>> > 3/(u-1) + 3/(t-1) + 2/((t-1)(u-1)) = 2k - 3 ==>
>> > (3 + 2/(t-1))/(u-1) = 2k - 3t/(t-1) ==>
>> > (3t - 1)/(u-1) = 2k(t-1) - 3t ==>
>> > u = 1 + (3t - 1)/((2k-3)t - 2k)
>> >
>> > k = 2 ==> u = 1 + (3t-1)/(t-4) = 4 + 11/(t-4) ==> t = 5 e u = 15
>> >
>> > k = 3 ==> u = 1 + (3t-1)/(3t-6) = 2 + 5/(3t-6) ==> XXX
>> >
>> > As únicas soluções são:
>> > (2,4,8) e (3,5,15)
>> >
>> > []s,
>> > Claudio.
>> >
>> > 2018-03-23 15:38 GMT-03:00 Pedro José :
>> >>
>> >> Boa tarde!
>> >>
>> >> Aproveitando que deu o que falar o problema postado pelo Douglas, tem
>> um
>> >> que achei mais interessante.
>> >>
>> >> (s-1)(t-1).(u-1) | stu -1, com s, t, u inteiros  e 1  >>
>> >> Saudações,
>> >> Pedro
>> >>
>> >> --
>> >> Esta mensagem foi verificada pelo sistema de antivírus e
>> >> acredita-se estar livre de perigo.
>> >
>> >
>> >
>> > --
>> > Esta mensagem foi verificada pelo sistema de antivírus e
>> > acredita-se estar livre de perigo.
>>
>> --
>> Esta mensagem foi verificada pelo sistema de antivírus e
>>  acredita-se estar livre de perigo.
>>
>>
>> =
>> Instru�ões para entrar na lista, sair da lista e usar a lista em
>> http://www.mat.puc-rio.br/~obmlistas/obm-l.html
>> =
>>
>
>
> --
> Esta mensagem foi verificada pelo sistema de antivírus e
> acredita-se estar livre de perigo.

-- 
Esta mensagem foi verificada pelo sistema de antiv�rus e
 acredita-se estar livre de perigo.



[obm-l] Re: [obm-l] Re: [obm-l] Re: [obm-l] Teoria dos números

2018-03-26 Por tôpico Claudio Buffara
Muito fácil pra ser de IMO...

2018-03-26 6:58 GMT-03:00 Anderson Torres :

> Este não é o problema de alguma IMO não? Eu lembro de ter resolvido,
> quase igual à solução oficial: substituir s,t,u por a+1,b+1,c+1 e
> calcular os possiveis valores de
> 1/a+1/b+1/c + 1/ab+1/ac+1/bc usando desigualdades - para daí limitar
> os valores de a,b,c.
>
> Em 23 de março de 2018 17:01, Claudio Buffara
>  escreveu:
> > Enfim, nesse meio tempo acho que resolvi o problema...
> >
> > Devemos achar inteiros s, t, u, com 1 < s < t < u e tais que:
> > (stu -1)/((s-1)(t-1)(u-1)) = k  (k inteiro positivo)
> >
> > Após diversas aplicações do truque (método?) de somar e subtrair a mesma
> > coisa, chegamos a:
> > stu - 1 =  (s-1)(t-1)(u-1) + (s-1)(t-1) + (s-1)(u-1) + (t-1)(u-1) +
> (s-1) +
> > (t-1) + (u-1)
> >
> > Dividindo isso por (s-1)(t-1)(u-1), obtemos:
> > 1 + 1/(u-1) + 1/(t-1) + 1/(s-1) + 1/((t-1)(u-1)) + 1/((s-1)(u-1)) +
> > 1/((s-1)(t-1)) = k ==>
> >
> > 1/(u-1) + 1/(t-1) + 1/(s-1) + 1/((t-1)(u-1)) + 1/((s-1)(u-1)) +
> > 1/((s-1)(t-1)) = k-1
> >
> > Agora a ideia é achar cotas para s e para k.
> >
> > 1 < s < t < u ==> s >= 2, t >= 3 e u >= 4 ==> o lado esquerdo é menor ou
> > igual que:
> > 1/3 + 1/2 + 1 + 1/6 + 1/3 + 1/2 = 2+5/6
> >
> > Ou seja, como o lado esquerdo é inteiro (e positivo), só poderá ser
> igual a
> > 1 ou a 2 ==> k = 2 ou k = 3.
> >
> > Se s >= 4, então t >= 5 e u >= 6, e o lado esquerdo será, no máximo,
> igual
> > a:
> > 1/5 + 1/4 + 1/3 + 1/20 + 1/15 + 1/12 < 1.
> >
> > Logo, devemos ter s = 2 ou s = 3.
> >
> > s = 2 ==>
> > 1/(u-1) + 1/(t-1) + 1 + 1/((t-1)(u-1)) + 1/(u-1) + 1/(t-1) = k-1 ==>
> > 2/(t-1) + 2/(u-1) + 1/((t-1)(u-1)) = k-2 ==>
> > Como k-2 deve ser inteiro positivo, k só pode ser 3 e, portanto:
> > 2/(t-1) + 2/(u-1) + 1/((t-1)(u-1)) = 1 ==>
> > (2 + 1/(t-1))/(u-1) = 1 - 2/(t-1) ==>
> > u = 1 + (2t - 1)/(t - 3) = 3 + 5/(t-3) ==>
> > t = 4 e u = 8   ou   t = 8 e u = 4 (não serve pois t deve ser menor do
> que
> > u)
> >
> > s = 3 ==>
> > 1/(u-1) + 1/(t-1) + 1/2 + 1/((t-1)(u-1)) + 1/(2(u-1)) + 1/(2(t-1)) = k-1
> ==>
> > (3/2)/(u-1) + (3/2)/(t-1) + 1/((t-1)(u-1)) = k - 3/2 ==>
> > 3/(u-1) + 3/(t-1) + 2/((t-1)(u-1)) = 2k - 3 ==>
> > (3 + 2/(t-1))/(u-1) = 2k - 3t/(t-1) ==>
> > (3t - 1)/(u-1) = 2k(t-1) - 3t ==>
> > u = 1 + (3t - 1)/((2k-3)t - 2k)
> >
> > k = 2 ==> u = 1 + (3t-1)/(t-4) = 4 + 11/(t-4) ==> t = 5 e u = 15
> >
> > k = 3 ==> u = 1 + (3t-1)/(3t-6) = 2 + 5/(3t-6) ==> XXX
> >
> > As únicas soluções são:
> > (2,4,8) e (3,5,15)
> >
> > []s,
> > Claudio.
> >
> > 2018-03-23 15:38 GMT-03:00 Pedro José :
> >>
> >> Boa tarde!
> >>
> >> Aproveitando que deu o que falar o problema postado pelo Douglas, tem um
> >> que achei mais interessante.
> >>
> >> (s-1)(t-1).(u-1) | stu -1, com s, t, u inteiros  e 1 >
> >> Saudações,
> >> Pedro
> >>
> >> --
> >> Esta mensagem foi verificada pelo sistema de antivírus e
> >> acredita-se estar livre de perigo.
> >
> >
> >
> > --
> > Esta mensagem foi verificada pelo sistema de antivírus e
> > acredita-se estar livre de perigo.
>
> --
> Esta mensagem foi verificada pelo sistema de antivírus e
>  acredita-se estar livre de perigo.
>
>
> =
> Instru�ões para entrar na lista, sair da lista e usar a lista em
> http://www.mat.puc-rio.br/~obmlistas/obm-l.html
> =
>

-- 
Esta mensagem foi verificada pelo sistema de antiv�rus e
 acredita-se estar livre de perigo.



[obm-l] Re: [obm-l] Re: [obm-l] Teoria dos números

2018-03-26 Por tôpico Anderson Torres
Este não é o problema de alguma IMO não? Eu lembro de ter resolvido,
quase igual à solução oficial: substituir s,t,u por a+1,b+1,c+1 e
calcular os possiveis valores de
1/a+1/b+1/c + 1/ab+1/ac+1/bc usando desigualdades - para daí limitar
os valores de a,b,c.

Em 23 de março de 2018 17:01, Claudio Buffara
 escreveu:
> Enfim, nesse meio tempo acho que resolvi o problema...
>
> Devemos achar inteiros s, t, u, com 1 < s < t < u e tais que:
> (stu -1)/((s-1)(t-1)(u-1)) = k  (k inteiro positivo)
>
> Após diversas aplicações do truque (método?) de somar e subtrair a mesma
> coisa, chegamos a:
> stu - 1 =  (s-1)(t-1)(u-1) + (s-1)(t-1) + (s-1)(u-1) + (t-1)(u-1) + (s-1) +
> (t-1) + (u-1)
>
> Dividindo isso por (s-1)(t-1)(u-1), obtemos:
> 1 + 1/(u-1) + 1/(t-1) + 1/(s-1) + 1/((t-1)(u-1)) + 1/((s-1)(u-1)) +
> 1/((s-1)(t-1)) = k ==>
>
> 1/(u-1) + 1/(t-1) + 1/(s-1) + 1/((t-1)(u-1)) + 1/((s-1)(u-1)) +
> 1/((s-1)(t-1)) = k-1
>
> Agora a ideia é achar cotas para s e para k.
>
> 1 < s < t < u ==> s >= 2, t >= 3 e u >= 4 ==> o lado esquerdo é menor ou
> igual que:
> 1/3 + 1/2 + 1 + 1/6 + 1/3 + 1/2 = 2+5/6
>
> Ou seja, como o lado esquerdo é inteiro (e positivo), só poderá ser igual a
> 1 ou a 2 ==> k = 2 ou k = 3.
>
> Se s >= 4, então t >= 5 e u >= 6, e o lado esquerdo será, no máximo, igual
> a:
> 1/5 + 1/4 + 1/3 + 1/20 + 1/15 + 1/12 < 1.
>
> Logo, devemos ter s = 2 ou s = 3.
>
> s = 2 ==>
> 1/(u-1) + 1/(t-1) + 1 + 1/((t-1)(u-1)) + 1/(u-1) + 1/(t-1) = k-1 ==>
> 2/(t-1) + 2/(u-1) + 1/((t-1)(u-1)) = k-2 ==>
> Como k-2 deve ser inteiro positivo, k só pode ser 3 e, portanto:
> 2/(t-1) + 2/(u-1) + 1/((t-1)(u-1)) = 1 ==>
> (2 + 1/(t-1))/(u-1) = 1 - 2/(t-1) ==>
> u = 1 + (2t - 1)/(t - 3) = 3 + 5/(t-3) ==>
> t = 4 e u = 8   ou   t = 8 e u = 4 (não serve pois t deve ser menor do que
> u)
>
> s = 3 ==>
> 1/(u-1) + 1/(t-1) + 1/2 + 1/((t-1)(u-1)) + 1/(2(u-1)) + 1/(2(t-1)) = k-1 ==>
> (3/2)/(u-1) + (3/2)/(t-1) + 1/((t-1)(u-1)) = k - 3/2 ==>
> 3/(u-1) + 3/(t-1) + 2/((t-1)(u-1)) = 2k - 3 ==>
> (3 + 2/(t-1))/(u-1) = 2k - 3t/(t-1) ==>
> (3t - 1)/(u-1) = 2k(t-1) - 3t ==>
> u = 1 + (3t - 1)/((2k-3)t - 2k)
>
> k = 2 ==> u = 1 + (3t-1)/(t-4) = 4 + 11/(t-4) ==> t = 5 e u = 15
>
> k = 3 ==> u = 1 + (3t-1)/(3t-6) = 2 + 5/(3t-6) ==> XXX
>
> As únicas soluções são:
> (2,4,8) e (3,5,15)
>
> []s,
> Claudio.
>
> 2018-03-23 15:38 GMT-03:00 Pedro José :
>>
>> Boa tarde!
>>
>> Aproveitando que deu o que falar o problema postado pelo Douglas, tem um
>> que achei mais interessante.
>>
>> (s-1)(t-1).(u-1) | stu -1, com s, t, u inteiros  e 1 
>> Saudações,
>> Pedro
>>
>> --
>> Esta mensagem foi verificada pelo sistema de antivírus e
>> acredita-se estar livre de perigo.
>
>
>
> --
> Esta mensagem foi verificada pelo sistema de antivírus e
> acredita-se estar livre de perigo.

-- 
Esta mensagem foi verificada pelo sistema de antiv�rus e
 acredita-se estar livre de perigo.


=
Instru��es para entrar na lista, sair da lista e usar a lista em
http://www.mat.puc-rio.br/~obmlistas/obm-l.html
=


[obm-l] Re: [obm-l] Re: [obm-l] Re: [obm-l] Re: [obm-l] Re: [obm-l] Teoria dos números

2018-03-26 Por tôpico Anderson Torres
Em 23 de março de 2018 10:35, Claudio Buffara
 escreveu:
> Na verdade os meus questionamentos surgiram por causa do meu interesse em
> ensino de matemática.
>
> Por exemplo, produtos notáveis e fatorações são notoriamente mal ensinados,
> pelo menos nos livros didáticos de 8o e 9o ano que eu examinei.

Eu acho que uma motivação mais geométrica pode ser bastante útil para
muitos produtos notáveis.

Por exemplo, a diferença de quadrados é bem facilmente explicada de
forma geométrica:
um quadrado com um quadradinho a menos no canto pode ser quebrado em
dois trapézios que formam
um retângulo.
O quadrado da soma é mais fácil ainda.

Por outro lado, eu não penso que minha solução foi a mais mágica de
todas, apenas era desconhecida.
Sempre que noto uma expressão simétrica, eu penso em como escrevê-la
em função ou dos
polinômios simétricos elementares ou da soma de potências (x^k+y^k+z^k).

> Nenhum menciona que:
> a) as generalizações de (x+y)^2 = x^2 + 2xy + y^2  e  x^2 - y^2 = (x-y)(x+y)
> para expoentes maiores levam ao teorema do binômio (erroneamente chamado de
> binômio de Newton - nota histórica: Newton generalizou o teorema para
> expoentes racionais) e à fórmula da soma dos termos de uma PG;
> b) (x+y)^2 = x^2 + 2xy + y^2 é a base para a ideia de se completar
> quadrados, a qual, por sua vez, não só resulta na fórmula para as raízes de
> uma equação quadrática, mas também na elucidação das propriedades da função
> quadrática;
> c) o uso inteligente da expansão de (x+y)^3 leva à formula das raízes de uma
> equação cúbica.

Essa eu não conhecia. Ainda penso que as formas mais naturais de lidar
com a cúbica são o Método Gugu-Euler
(tentar uma solução da forma x=raizcúbica(y1)+raizcúbica(y2)) ou usar
a fatoração de x^3+y^3+z^3-3xyz.

Mas o cubo da soma, per se? Isso me parece mágico demais.

(Acho que até imagino o que seja: identificar o termo constante com a
soma de cubos e o termo "linear" com o triplo do produto...)

>
> ***
>
> Há tempos, o Hermann, participante desta lista, postou uma dúvida sobre
> produtos notáveis e pediu dicas de livros com exercícios sobre produtos
> notáveis e fatoração.
> Eu tenho duas sugestões, ambas em inglês:
> - Algebra, de I.M.Gelfand e A.Shen - Birkhäuser (este faz as generalizações
> que eu mencionei acima)
> - A Problem Book in Algebra, de V.A. Krechmar - Mir Publishers (pros
> entusiastas)
> Ambos estão disponíveis na Amazon.
>
> ***
>
> Anos atrás eu gostava de soluções "mágicas", obtidas por meio de alguma
> sacada brilhante que eu jamais conseguiria ter.
> Após me deparar com várias destas soluções, me ocorreu que elas talvez
> tivessem um efeito perverso na motivação dos estudantes de matemática, pois
> passavam a impressão de que é preciso ser um gênio para dominar a matéria.
> Daí o meu interesse em saber como vocês obtiveram certas fatorações.
> Entendo que trabalho braçal, experiência, alguma lógica e um pouco de
> otimismo são, para a maioria de nós, as únicas formas de progredir na
> resolução de um problema como o que deu origem a este thread.
>
> Dito isso (e posso estar enganado) nem o Pedro José e nem mais ninguém
> explicou de onde veio a conjectura (correta) de que:
> z = -(x+y)/2 é solução de (x + y)(y + z)(z + x)/2 + (x + y + z)3 =  – xyz
>
> []s,
> Claudio.
>
>
>
> 2018-03-23 6:20 GMT-03:00 Anderson Torres :
>>
>> Em 21 de março de 2018 09:47, Claudio Buffara
>>  escreveu:
>> > Como você passou de:
>> > 4abc + (a+b+c)^3 + (-a+b+c)(a-b+c)(a+b-c) = 1
>> >
>> > Para:
>> > 4(a+b+c)(ab+ac+bc) - 4abc = 1
>>
>> It's kind of magic. Eu simplesmente abri tudo com vontade e notei
>> certas repetições
>> que sempre aparecem em certas fatorações; ou melhor dizendo, estava
>> pensando em
>> escrever tudo em termos dos famigerados polinômios simétricos e cheguei
>> nisso.
>>
>> Sempre que vejo algo como (a^2b+ab^2), já escrevo ab(a+b) e tento
>> procurar um abc
>> para isso resultar em ab(a+b+c).
>>
>> Mas não avancei daí. Penso que dá para fatorar ainda mais...
>>
>> >
>> > ???
>> >
>> > []s,
>> > Claudio.
>> >
>> >
>> > 2018-03-20 23:14 GMT-03:00 Anderson Torres
>> > :
>> >>
>> >> Em 13 de março de 2018 20:19, Douglas Oliveira de Lima
>> >>  escreveu:
>> >> > Essa achei legal e estou postando.
>> >> >
>> >> > Resolva nos inteiros a seguinte equação:  (x + y)(y + z)(z + x)/2 +
>> >> > (x +
>> >> > y +
>> >> > z)3 = 1 – xyz .
>> >> >
>> >>
>> >> Substituição mágica: x=-a+b+c, y=a-b+c, z=a+b-c. Com isso, x+y=2c,
>> >> x+y+z=a+b+c e
>> >>
>> >> 4abc + (a+b+c)^3 + (-a+b+c)(a-b+c)(a+b-c) = 1
>> >>
>> >> Usando polinômios simétricos,
>> >>
>> >> 4(a+b+c)(ab+ac+bc) - 4abc = 1
>> >>
>> >> Agora estou confuso...
>> >>
>> >> > Abraço do
>> >> > Douglas Oliveira
>> >> >
>> >> > --
>> >> > Esta mensagem foi verificada pelo sistema de antivírus e
>> >> > acredita-se estar livre de perigo.
>> >>
>> >> --
>> >> Esta mensagem foi 

[obm-l] Re: [obm-l] Re: [obm-l] Re: [obm-l] Teoria dos números

2018-03-23 Por tôpico Claudio Buffara
Pois do jeito que você propôs, alguém poderia pensar que se trata de provar
que (s-1)(t-1)(u-1) divide stu - 1 para todos os inteiros s, t, u com 1 < s
< t < u, o que certamente não é verdade.


2018-03-23 16:55 GMT-03:00 Claudio Buffara :

> Sim. Eu só quis ter certeza de que o problema era:
> achar todos os inteiros s, t, u com 1 < s < t < u tais que (s-1)(t-1)(u-1)
> divide stu - 1
>
> 2018-03-23 16:45 GMT-03:00 Pedro José :
>
>> Boa tarde!
>> Seria bom desconsiderar o problema aqui, pois já tem um caminhão de
>> notas. Criei uma mensagem nova, não sei porque foi parar aqui, não sei se
>> pelo assunto ter o mesmo nome.
>> Alguém postá-lo independente dessa leva.
>> Cláudio, o que você propôs, não tem solução. Não creio que ajude.
>> Não compreendi a sua primeira pergunta, um esclarecimento o síbolo "|"
>> não é tal que é divide, ou seja (s-1)(t-1) divide st - 1,
>>
>> Saudações,
>> PJMS
>>
>> Em 23 de março de 2018 15:49, Claudio Buffara 
>> escreveu:
>>
>>> Será que ajuda começar com um mais simples?
>>>
>>> Achar s, t tais que (s-1)(t-1) | st - 1, com 1 < s < t.
>>>
>>> 2018-03-23 15:38 GMT-03:00 Pedro José :
>>>
 Boa tarde!

 Aproveitando que deu o que falar o problema postado pelo Douglas, tem
 um que achei mais interessante.

 (s-1)(t-1).(u-1) | stu -1, com s, t, u inteiros  e 1 >>
 Saudações,
 Pedro

 --
 Esta mensagem foi verificada pelo sistema de antivírus e
 acredita-se estar livre de perigo.
>>>
>>>
>>>
>>> --
>>> Esta mensagem foi verificada pelo sistema de antivírus e
>>> acredita-se estar livre de perigo.
>>>
>>
>>
>> --
>> Esta mensagem foi verificada pelo sistema de antivírus e
>> acredita-se estar livre de perigo.
>>
>
>

-- 
Esta mensagem foi verificada pelo sistema de antiv�rus e
 acredita-se estar livre de perigo.



[obm-l] Re: [obm-l] Re: [obm-l] Re: [obm-l] Teoria dos números

2018-03-23 Por tôpico Claudio Buffara
Sim. Eu só quis ter certeza de que o problema era:
achar todos os inteiros s, t, u com 1 < s < t < u tais que (s-1)(t-1)(u-1)
divide stu - 1

2018-03-23 16:45 GMT-03:00 Pedro José :

> Boa tarde!
> Seria bom desconsiderar o problema aqui, pois já tem um caminhão de notas.
> Criei uma mensagem nova, não sei porque foi parar aqui, não sei se pelo
> assunto ter o mesmo nome.
> Alguém postá-lo independente dessa leva.
> Cláudio, o que você propôs, não tem solução. Não creio que ajude.
> Não compreendi a sua primeira pergunta, um esclarecimento o síbolo "|" não
> é tal que é divide, ou seja (s-1)(t-1) divide st - 1,
>
> Saudações,
> PJMS
>
> Em 23 de março de 2018 15:49, Claudio Buffara 
> escreveu:
>
>> Será que ajuda começar com um mais simples?
>>
>> Achar s, t tais que (s-1)(t-1) | st - 1, com 1 < s < t.
>>
>> 2018-03-23 15:38 GMT-03:00 Pedro José :
>>
>>> Boa tarde!
>>>
>>> Aproveitando que deu o que falar o problema postado pelo Douglas, tem um
>>> que achei mais interessante.
>>>
>>> (s-1)(t-1).(u-1) | stu -1, com s, t, u inteiros  e 1 >
>>> Saudações,
>>> Pedro
>>>
>>> --
>>> Esta mensagem foi verificada pelo sistema de antivírus e
>>> acredita-se estar livre de perigo.
>>
>>
>>
>> --
>> Esta mensagem foi verificada pelo sistema de antivírus e
>> acredita-se estar livre de perigo.
>>
>
>
> --
> Esta mensagem foi verificada pelo sistema de antivírus e
> acredita-se estar livre de perigo.
>

-- 
Esta mensagem foi verificada pelo sistema de antiv�rus e
 acredita-se estar livre de perigo.



[obm-l] Re: [obm-l] Re: [obm-l] Teoria dos números

2018-03-23 Por tôpico Pedro José
Boa tarde!
Seria bom desconsiderar o problema aqui, pois já tem um caminhão de notas.
Criei uma mensagem nova, não sei porque foi parar aqui, não sei se pelo
assunto ter o mesmo nome.
Alguém postá-lo independente dessa leva.
Cláudio, o que você propôs, não tem solução. Não creio que ajude.
Não compreendi a sua primeira pergunta, um esclarecimento o síbolo "|" não
é tal que é divide, ou seja (s-1)(t-1) divide st - 1,

Saudações,
PJMS

Em 23 de março de 2018 15:49, Claudio Buffara 
escreveu:

> Será que ajuda começar com um mais simples?
>
> Achar s, t tais que (s-1)(t-1) | st - 1, com 1 < s < t.
>
> 2018-03-23 15:38 GMT-03:00 Pedro José :
>
>> Boa tarde!
>>
>> Aproveitando que deu o que falar o problema postado pelo Douglas, tem um
>> que achei mais interessante.
>>
>> (s-1)(t-1).(u-1) | stu -1, com s, t, u inteiros  e 1 
>> Saudações,
>> Pedro
>>
>> --
>> Esta mensagem foi verificada pelo sistema de antivírus e
>> acredita-se estar livre de perigo.
>
>
>
> --
> Esta mensagem foi verificada pelo sistema de antivírus e
> acredita-se estar livre de perigo.
>

-- 
Esta mensagem foi verificada pelo sistema de antiv�rus e
 acredita-se estar livre de perigo.



[obm-l] Re: [obm-l] Teoria dos números

2018-03-23 Por tôpico Claudio Buffara
Será que ajuda começar com um mais simples?

Achar s, t tais que (s-1)(t-1) | st - 1, com 1 < s < t.

2018-03-23 15:38 GMT-03:00 Pedro José :

> Boa tarde!
>
> Aproveitando que deu o que falar o problema postado pelo Douglas, tem um
> que achei mais interessante.
>
> (s-1)(t-1).(u-1) | stu -1, com s, t, u inteiros  e 1  Saudações,
> Pedro
>
> --
> Esta mensagem foi verificada pelo sistema de antivírus e
> acredita-se estar livre de perigo.

-- 
Esta mensagem foi verificada pelo sistema de antiv�rus e
 acredita-se estar livre de perigo.



[obm-l] Re: [obm-l] Re: [obm-l] Re: [obm-l] Re: [obm-l] Re: [obm-l] Teoria dos números

2018-03-23 Por tôpico Claudio Buffara
Você não havia explicado que* "fui fazer um experimento tirando o "1" da
equação. Usei um par (x,y) com a mesma paridade e achei um z inteiro.
Novamente usei outro par e deu outro z inteiro. Olhando para os
experimentos. Vi que nos  dois casos z = -(x+y)/2. Ai tornou-se uma
conjectura."*

Ou seja, experimentos numéricos levaram à conjectura. Beleza!

De toda forma, fazer experimentos tirando o "1" da equação foi uma bela
sacada. Você primeiro examinou a versão "homogênea". Boa ideia.

[]s,
Claudio.




2018-03-23 14:30 GMT-03:00 Pedro José :

> Boa tarde!
>
> Cláudio,
> desculpe-me discordar, mas eu disse  de onde veio. Só não veio de nenhuma
> técnica.
> Estava vendo que a parcela do problema: (x+y) (x+z) (y+z)/2 sempre seria
> inteira pois dois desses valores teriam paridade iguais.
> Aí fui fazer um experimento tirando o "1" da equação. Usei um par (x,y)
> com a mesma paridade e achei um z inteiro. Novamente usei outro par e deu
> outro z inteiro. Olhando para os experimentos. Vi que nos  dois casos z =
> -(x+y)/2. Ai tornou-se uma conjectura. Pura sorte.
> Mas, no braço, desenvolvi para para [x,y, -(x+y)/2] e atendeu para essa
> família, aqui deixou de ser conjectura. Foi provado, na grosseria, por
> substituição mas foi.
>
> Saudações,
> PJMS
>
> Em 23 de março de 2018 11:07, Pedro José  escreveu:
>
>> Bom dia!
>> Anderson,
>> o Gugu já avançou, em uma nota acima. E é passível.
>> Revendo a solução do Ralph, fica claro que essa transformação seria de
>> valia.
>> Pois essa transformação leva a :
>> a = (y+z)/2
>> b=  (x+z)/2
>> c= (x+y)/2
>>
>> Então na ordem que o Ralph apresentou: 1/2*(2x+y+z)(x+2y+z)(x+y+2z)=1
>>
>> (b+c) dá a metade do primeiro fator (excetuando-se o fator 1/2) , (a+c) a
>> metade do segundo e (a+b) a metade do terceiro.
>>
>> Saudações,
>>
>>
>>
>> Em 23 de março de 2018 06:20, Anderson Torres <
>> torres.anderson...@gmail.com> escreveu:
>>
>>> Em 21 de março de 2018 09:47, Claudio Buffara
>>>  escreveu:
>>> > Como você passou de:
>>> > 4abc + (a+b+c)^3 + (-a+b+c)(a-b+c)(a+b-c) = 1
>>> >
>>> > Para:
>>> > 4(a+b+c)(ab+ac+bc) - 4abc = 1
>>>
>>> It's kind of magic. Eu simplesmente abri tudo com vontade e notei
>>> certas repetições
>>> que sempre aparecem em certas fatorações; ou melhor dizendo, estava
>>> pensando em
>>> escrever tudo em termos dos famigerados polinômios simétricos e cheguei
>>> nisso.
>>>
>>> Sempre que vejo algo como (a^2b+ab^2), já escrevo ab(a+b) e tento
>>> procurar um abc
>>> para isso resultar em ab(a+b+c).
>>>
>>> Mas não avancei daí. Penso que dá para fatorar ainda mais...
>>>
>>> >
>>> > ???
>>> >
>>> > []s,
>>> > Claudio.
>>> >
>>> >
>>> > 2018-03-20 23:14 GMT-03:00 Anderson Torres <
>>> torres.anderson...@gmail.com>:
>>> >>
>>> >> Em 13 de março de 2018 20:19, Douglas Oliveira de Lima
>>> >>  escreveu:
>>> >> > Essa achei legal e estou postando.
>>> >> >
>>> >> > Resolva nos inteiros a seguinte equação:  (x + y)(y + z)(z + x)/2 +
>>> (x +
>>> >> > y +
>>> >> > z)3 = 1 – xyz .
>>> >> >
>>> >>
>>> >> Substituição mágica: x=-a+b+c, y=a-b+c, z=a+b-c. Com isso, x+y=2c,
>>> >> x+y+z=a+b+c e
>>> >>
>>> >> 4abc + (a+b+c)^3 + (-a+b+c)(a-b+c)(a+b-c) = 1
>>> >>
>>> >> Usando polinômios simétricos,
>>> >>
>>> >> 4(a+b+c)(ab+ac+bc) - 4abc = 1
>>> >>
>>> >> Agora estou confuso...
>>> >>
>>> >> > Abraço do
>>> >> > Douglas Oliveira
>>> >> >
>>> >> > --
>>> >> > Esta mensagem foi verificada pelo sistema de antivírus e
>>> >> > acredita-se estar livre de perigo.
>>> >>
>>> >> --
>>> >> Esta mensagem foi verificada pelo sistema de antivírus e
>>> >>  acredita-se estar livre de perigo.
>>> >>
>>> >>
>>> >> 
>>> =
>>> >> Instru�ões para entrar na lista, sair da lista e usar a lista em
>>> >> http://www.mat.puc-rio.br/~obmlistas/obm-l.html
>>> >> 
>>> =
>>> >
>>> >
>>> >
>>> > --
>>> > Esta mensagem foi verificada pelo sistema de antivírus e
>>> > acredita-se estar livre de perigo.
>>>
>>> --
>>> Esta mensagem foi verificada pelo sistema de antivírus e
>>>  acredita-se estar livre de perigo.
>>>
>>>
>>> 
>>> =
>>> Instru�ões para entrar na lista, sair da lista e usar a lista em
>>> http://www.mat.puc-rio.br/~obmlistas/obm-l.html
>>> 
>>> =
>>>
>>
>>
>
> --
> Esta mensagem foi verificada pelo sistema de antivírus e
> acredita-se estar livre de perigo.
>

-- 
Esta mensagem foi verificada pelo sistema de antiv�rus e
 acredita-se estar livre de perigo.



[obm-l] Re: [obm-l] Re: [obm-l] Re: [obm-l] Re: [obm-l] Teoria dos números

2018-03-23 Por tôpico Pedro José
Boa tarde!

Cláudio,
desculpe-me discordar, mas eu disse  de onde veio. Só não veio de nenhuma
técnica.
Estava vendo que a parcela do problema: (x+y) (x+z) (y+z)/2 sempre seria
inteira pois dois desses valores teriam paridade iguais.
Aí fui fazer um experimento tirando o "1" da equação. Usei um par (x,y) com
a mesma paridade e achei um z inteiro. Novamente usei outro par e deu outro
z inteiro. Olhando para os experimentos. Vi que nos  dois casos z =
-(x+y)/2. Ai tornou-se uma conjectura. Pura sorte.
Mas, no braço, desenvolvi para para [x,y, -(x+y)/2] e atendeu para essa
família, aqui deixou de ser conjectura. Foi provado, na grosseria, por
substituição mas foi.

Saudações,
PJMS

Em 23 de março de 2018 11:07, Pedro José  escreveu:

> Bom dia!
> Anderson,
> o Gugu já avançou, em uma nota acima. E é passível.
> Revendo a solução do Ralph, fica claro que essa transformação seria de
> valia.
> Pois essa transformação leva a :
> a = (y+z)/2
> b=  (x+z)/2
> c= (x+y)/2
>
> Então na ordem que o Ralph apresentou: 1/2*(2x+y+z)(x+2y+z)(x+y+2z)=1
>
> (b+c) dá a metade do primeiro fator (excetuando-se o fator 1/2) , (a+c) a
> metade do segundo e (a+b) a metade do terceiro.
>
> Saudações,
>
>
>
> Em 23 de março de 2018 06:20, Anderson Torres <
> torres.anderson...@gmail.com> escreveu:
>
>> Em 21 de março de 2018 09:47, Claudio Buffara
>>  escreveu:
>> > Como você passou de:
>> > 4abc + (a+b+c)^3 + (-a+b+c)(a-b+c)(a+b-c) = 1
>> >
>> > Para:
>> > 4(a+b+c)(ab+ac+bc) - 4abc = 1
>>
>> It's kind of magic. Eu simplesmente abri tudo com vontade e notei
>> certas repetições
>> que sempre aparecem em certas fatorações; ou melhor dizendo, estava
>> pensando em
>> escrever tudo em termos dos famigerados polinômios simétricos e cheguei
>> nisso.
>>
>> Sempre que vejo algo como (a^2b+ab^2), já escrevo ab(a+b) e tento
>> procurar um abc
>> para isso resultar em ab(a+b+c).
>>
>> Mas não avancei daí. Penso que dá para fatorar ainda mais...
>>
>> >
>> > ???
>> >
>> > []s,
>> > Claudio.
>> >
>> >
>> > 2018-03-20 23:14 GMT-03:00 Anderson Torres <
>> torres.anderson...@gmail.com>:
>> >>
>> >> Em 13 de março de 2018 20:19, Douglas Oliveira de Lima
>> >>  escreveu:
>> >> > Essa achei legal e estou postando.
>> >> >
>> >> > Resolva nos inteiros a seguinte equação:  (x + y)(y + z)(z + x)/2 +
>> (x +
>> >> > y +
>> >> > z)3 = 1 – xyz .
>> >> >
>> >>
>> >> Substituição mágica: x=-a+b+c, y=a-b+c, z=a+b-c. Com isso, x+y=2c,
>> >> x+y+z=a+b+c e
>> >>
>> >> 4abc + (a+b+c)^3 + (-a+b+c)(a-b+c)(a+b-c) = 1
>> >>
>> >> Usando polinômios simétricos,
>> >>
>> >> 4(a+b+c)(ab+ac+bc) - 4abc = 1
>> >>
>> >> Agora estou confuso...
>> >>
>> >> > Abraço do
>> >> > Douglas Oliveira
>> >> >
>> >> > --
>> >> > Esta mensagem foi verificada pelo sistema de antivírus e
>> >> > acredita-se estar livre de perigo.
>> >>
>> >> --
>> >> Esta mensagem foi verificada pelo sistema de antivírus e
>> >>  acredita-se estar livre de perigo.
>> >>
>> >>
>> >> 
>> =
>> >> Instru�ões para entrar na lista, sair da lista e usar a lista em
>> >> http://www.mat.puc-rio.br/~obmlistas/obm-l.html
>> >> 
>> =
>> >
>> >
>> >
>> > --
>> > Esta mensagem foi verificada pelo sistema de antivírus e
>> > acredita-se estar livre de perigo.
>>
>> --
>> Esta mensagem foi verificada pelo sistema de antivírus e
>>  acredita-se estar livre de perigo.
>>
>>
>> =
>> Instru�ões para entrar na lista, sair da lista e usar a lista em
>> http://www.mat.puc-rio.br/~obmlistas/obm-l.html
>> =
>>
>
>

-- 
Esta mensagem foi verificada pelo sistema de antiv�rus e
 acredita-se estar livre de perigo.



[obm-l] Re: [obm-l] Re: [obm-l] Re: [obm-l] Re: [obm-l] Teoria dos números

2018-03-23 Por tôpico Pedro José
Bom dia!
Anderson,
o Gugu já avançou, em uma nota acima. E é passível.
Revendo a solução do Ralph, fica claro que essa transformação seria de
valia.
Pois essa transformação leva a :
a = (y+z)/2
b=  (x+z)/2
c= (x+y)/2

Então na ordem que o Ralph apresentou: 1/2*(2x+y+z)(x+2y+z)(x+y+2z)=1

(b+c) dá a metade do primeiro fator (excetuando-se o fator 1/2) , (a+c) a
metade do segundo e (a+b) a metade do terceiro.

Saudações,



Em 23 de março de 2018 06:20, Anderson Torres 
escreveu:

> Em 21 de março de 2018 09:47, Claudio Buffara
>  escreveu:
> > Como você passou de:
> > 4abc + (a+b+c)^3 + (-a+b+c)(a-b+c)(a+b-c) = 1
> >
> > Para:
> > 4(a+b+c)(ab+ac+bc) - 4abc = 1
>
> It's kind of magic. Eu simplesmente abri tudo com vontade e notei
> certas repetições
> que sempre aparecem em certas fatorações; ou melhor dizendo, estava
> pensando em
> escrever tudo em termos dos famigerados polinômios simétricos e cheguei
> nisso.
>
> Sempre que vejo algo como (a^2b+ab^2), já escrevo ab(a+b) e tento
> procurar um abc
> para isso resultar em ab(a+b+c).
>
> Mas não avancei daí. Penso que dá para fatorar ainda mais...
>
> >
> > ???
> >
> > []s,
> > Claudio.
> >
> >
> > 2018-03-20 23:14 GMT-03:00 Anderson Torres  >:
> >>
> >> Em 13 de março de 2018 20:19, Douglas Oliveira de Lima
> >>  escreveu:
> >> > Essa achei legal e estou postando.
> >> >
> >> > Resolva nos inteiros a seguinte equação:  (x + y)(y + z)(z + x)/2 +
> (x +
> >> > y +
> >> > z)3 = 1 – xyz .
> >> >
> >>
> >> Substituição mágica: x=-a+b+c, y=a-b+c, z=a+b-c. Com isso, x+y=2c,
> >> x+y+z=a+b+c e
> >>
> >> 4abc + (a+b+c)^3 + (-a+b+c)(a-b+c)(a+b-c) = 1
> >>
> >> Usando polinômios simétricos,
> >>
> >> 4(a+b+c)(ab+ac+bc) - 4abc = 1
> >>
> >> Agora estou confuso...
> >>
> >> > Abraço do
> >> > Douglas Oliveira
> >> >
> >> > --
> >> > Esta mensagem foi verificada pelo sistema de antivírus e
> >> > acredita-se estar livre de perigo.
> >>
> >> --
> >> Esta mensagem foi verificada pelo sistema de antivírus e
> >>  acredita-se estar livre de perigo.
> >>
> >>
> >> 
> =
> >> Instru�ões para entrar na lista, sair da lista e usar a lista em
> >> http://www.mat.puc-rio.br/~obmlistas/obm-l.html
> >> 
> =
> >
> >
> >
> > --
> > Esta mensagem foi verificada pelo sistema de antivírus e
> > acredita-se estar livre de perigo.
>
> --
> Esta mensagem foi verificada pelo sistema de antivírus e
>  acredita-se estar livre de perigo.
>
>
> =
> Instru�ões para entrar na lista, sair da lista e usar a lista em
> http://www.mat.puc-rio.br/~obmlistas/obm-l.html
> =
>

-- 
Esta mensagem foi verificada pelo sistema de antiv�rus e
 acredita-se estar livre de perigo.



[obm-l] Re: [obm-l] Re: [obm-l] Re: [obm-l] Re: [obm-l] Teoria dos números

2018-03-23 Por tôpico Claudio Buffara
Na verdade os meus questionamentos surgiram por causa do meu interesse em
ensino de matemática.

Por exemplo, produtos notáveis e fatorações são notoriamente mal ensinados,
pelo menos nos livros didáticos de 8o e 9o ano que eu examinei.
Nenhum menciona que:
a) as generalizações de (x+y)^2 = x^2 + 2xy + y^2  e  x^2 - y^2 =
(x-y)(x+y) para expoentes maiores levam ao teorema do binômio (erroneamente
chamado de binômio de Newton - nota histórica: Newton generalizou o teorema
para expoentes racionais) e à fórmula da soma dos termos de uma PG;
b) (x+y)^2 = x^2 + 2xy + y^2 é a base para a ideia de se completar
quadrados, a qual, por sua vez, não só resulta na fórmula para as raízes de
uma equação quadrática, mas também na elucidação das propriedades da função
quadrática;
c) o uso inteligente da expansão de (x+y)^3 leva à formula das raízes de
uma equação cúbica.

***

Há tempos, o Hermann, participante desta lista, postou uma dúvida sobre
produtos notáveis e pediu dicas de livros com exercícios sobre produtos
notáveis e fatoração.
Eu tenho duas sugestões, ambas em inglês:
- Algebra, de I.M.Gelfand e A.Shen - Birkhäuser (este faz as generalizações
que eu mencionei acima)
- A Problem Book in Algebra, de V.A. Krechmar - Mir Publishers (pros
entusiastas)
Ambos estão disponíveis na Amazon.

***

Anos atrás eu gostava de soluções "mágicas", obtidas por meio de alguma
sacada brilhante que eu jamais conseguiria ter.
Após me deparar com várias destas soluções, me ocorreu que elas talvez
tivessem um efeito perverso na motivação dos estudantes de matemática, pois
passavam a impressão de que é preciso ser um gênio para dominar a matéria.
Daí o meu interesse em saber como vocês obtiveram certas fatorações.
Entendo que trabalho braçal, experiência, alguma lógica e um pouco de
otimismo são, para a maioria de nós, as únicas formas de progredir na
resolução de um problema como o que deu origem a este thread.

Dito isso (e posso estar enganado) nem o Pedro José e nem mais ninguém
explicou de onde veio a conjectura (correta) de que:
z = -(x+y)/2 é solução de *(x + y)(y + z)(z + x)/2 + (x + y + z)3 =  – xyz*

[]s,
Claudio.



2018-03-23 6:20 GMT-03:00 Anderson Torres :

> Em 21 de março de 2018 09:47, Claudio Buffara
>  escreveu:
> > Como você passou de:
> > 4abc + (a+b+c)^3 + (-a+b+c)(a-b+c)(a+b-c) = 1
> >
> > Para:
> > 4(a+b+c)(ab+ac+bc) - 4abc = 1
>
> It's kind of magic. Eu simplesmente abri tudo com vontade e notei
> certas repetições
> que sempre aparecem em certas fatorações; ou melhor dizendo, estava
> pensando em
> escrever tudo em termos dos famigerados polinômios simétricos e cheguei
> nisso.
>
> Sempre que vejo algo como (a^2b+ab^2), já escrevo ab(a+b) e tento
> procurar um abc
> para isso resultar em ab(a+b+c).
>
> Mas não avancei daí. Penso que dá para fatorar ainda mais...
>
> >
> > ???
> >
> > []s,
> > Claudio.
> >
> >
> > 2018-03-20 23:14 GMT-03:00 Anderson Torres  >:
> >>
> >> Em 13 de março de 2018 20:19, Douglas Oliveira de Lima
> >>  escreveu:
> >> > Essa achei legal e estou postando.
> >> >
> >> > Resolva nos inteiros a seguinte equação:  (x + y)(y + z)(z + x)/2 +
> (x +
> >> > y +
> >> > z)3 = 1 – xyz .
> >> >
> >>
> >> Substituição mágica: x=-a+b+c, y=a-b+c, z=a+b-c. Com isso, x+y=2c,
> >> x+y+z=a+b+c e
> >>
> >> 4abc + (a+b+c)^3 + (-a+b+c)(a-b+c)(a+b-c) = 1
> >>
> >> Usando polinômios simétricos,
> >>
> >> 4(a+b+c)(ab+ac+bc) - 4abc = 1
> >>
> >> Agora estou confuso...
> >>
> >> > Abraço do
> >> > Douglas Oliveira
> >> >
> >> > --
> >> > Esta mensagem foi verificada pelo sistema de antivírus e
> >> > acredita-se estar livre de perigo.
> >>
> >> --
> >> Esta mensagem foi verificada pelo sistema de antivírus e
> >>  acredita-se estar livre de perigo.
> >>
> >>
> >> 
> =
> >> Instru�ões para entrar na lista, sair da lista e usar a lista em
> >> http://www.mat.puc-rio.br/~obmlistas/obm-l.html
> >> 
> =
> >
> >
> >
> > --
> > Esta mensagem foi verificada pelo sistema de antivírus e
> > acredita-se estar livre de perigo.
>
> --
> Esta mensagem foi verificada pelo sistema de antivírus e
>  acredita-se estar livre de perigo.
>
>
> =
> Instru�ões para entrar na lista, sair da lista e usar a lista em
> http://www.mat.puc-rio.br/~obmlistas/obm-l.html
> =
>

-- 
Esta mensagem foi verificada pelo sistema de antiv�rus e
 acredita-se estar livre de perigo.



[obm-l] Re: [obm-l] Re: [obm-l] Re: [obm-l] Teoria dos números

2018-03-23 Por tôpico Anderson Torres
Em 21 de março de 2018 09:47, Claudio Buffara
 escreveu:
> Como você passou de:
> 4abc + (a+b+c)^3 + (-a+b+c)(a-b+c)(a+b-c) = 1
>
> Para:
> 4(a+b+c)(ab+ac+bc) - 4abc = 1

It's kind of magic. Eu simplesmente abri tudo com vontade e notei
certas repetições
que sempre aparecem em certas fatorações; ou melhor dizendo, estava pensando em
escrever tudo em termos dos famigerados polinômios simétricos e cheguei nisso.

Sempre que vejo algo como (a^2b+ab^2), já escrevo ab(a+b) e tento
procurar um abc
para isso resultar em ab(a+b+c).

Mas não avancei daí. Penso que dá para fatorar ainda mais...

>
> ???
>
> []s,
> Claudio.
>
>
> 2018-03-20 23:14 GMT-03:00 Anderson Torres :
>>
>> Em 13 de março de 2018 20:19, Douglas Oliveira de Lima
>>  escreveu:
>> > Essa achei legal e estou postando.
>> >
>> > Resolva nos inteiros a seguinte equação:  (x + y)(y + z)(z + x)/2 + (x +
>> > y +
>> > z)3 = 1 – xyz .
>> >
>>
>> Substituição mágica: x=-a+b+c, y=a-b+c, z=a+b-c. Com isso, x+y=2c,
>> x+y+z=a+b+c e
>>
>> 4abc + (a+b+c)^3 + (-a+b+c)(a-b+c)(a+b-c) = 1
>>
>> Usando polinômios simétricos,
>>
>> 4(a+b+c)(ab+ac+bc) - 4abc = 1
>>
>> Agora estou confuso...
>>
>> > Abraço do
>> > Douglas Oliveira
>> >
>> > --
>> > Esta mensagem foi verificada pelo sistema de antivírus e
>> > acredita-se estar livre de perigo.
>>
>> --
>> Esta mensagem foi verificada pelo sistema de antivírus e
>>  acredita-se estar livre de perigo.
>>
>>
>> =
>> Instru�ões para entrar na lista, sair da lista e usar a lista em
>> http://www.mat.puc-rio.br/~obmlistas/obm-l.html
>> =
>
>
>
> --
> Esta mensagem foi verificada pelo sistema de antivírus e
> acredita-se estar livre de perigo.

-- 
Esta mensagem foi verificada pelo sistema de antiv�rus e
 acredita-se estar livre de perigo.


=
Instru��es para entrar na lista, sair da lista e usar a lista em
http://www.mat.puc-rio.br/~obmlistas/obm-l.html
=


[obm-l] Re: [obm-l] Re: [obm-l] Re: [obm-l] Re: [obm-l] Teoria dos números

2018-03-22 Por tôpico Pedro José
Boa noite!
Nem havia reparado que a transformação do Gugu, foi feita em composição com
a anterior que fora postada. Acabou sendo a mesma que postei.
Escolhi porque fazia sumir os termos com expoente 3.

Saudações,
PJMS

Em 22 de mar de 2018 22:59, "Pedro José"  escreveu:

> Boa noite!
> Vi duas proposições de substituições de variáveis, nas notas anteriores e
> ratifico os questionamentos do Cláudio.
> Aventurei uma substituição:
> a=x+y ; b=x+z; c = y + z.
> Aí, na munheca cancelam-se os termos com expoentes cúbicos. E separando os
> termos de
>  (a+b)*(a+c), no que sobra, chega- se facilmente a:
> 1/2*(a+b)*(a+c)*(b+c) = 1 ou
> (a+b)*(a+c)*(b+c) = 2.
> Para x, y e z inteiros só atendem soluções em a, b e c com dois ímpares e
> um par ou três pares.
> Então, só atendem a= 0, b=1 e c= 1 ou a=-2, b=1 e c= 1. E suas
> permutações. Que trazendo para as variáveis originais dá o mesmo resultado
> já encontrado.
> Mas minha dúvida é: para esses casos de polinômios simétricos há alguma
> técnica para escolha de variáveis e para fatorar, como é.g. , na mudança de
> variáveis das cônicas para retirada do termo xy, possibilitando a
> identificação da cônica?
> Saudações,
> PJMS
>
> Em 22 de mar de 2018 7:56 AM, "Claudio Buffara" 
> escreveu:
>
>> Tudo muito bom, mas o que ninguém explicou é como foram obtidas as
>> fatorações/transformações algébricas mágicas.
>> Insight?
>> Conhecimentos prévios?
>> Tentativa e erro e muito braço?
>>
>> []s,
>> Claudio.
>>
>>
>> 2018-03-21 18:54 GMT-03:00 :
>>
>>> Sim, e fazendo a=u/2, b=v/2 e c=w/2 temos (u+v+w)(uv+uw+vw)-uvw=2, ou
>>> seja, u^2v+uv^2+u^2w+uw^2+v^2w+vw^2+2uvw=2, mas
>>> u^2v+uv^2+u^2w+uw^2+v^2w+vw^2+2uvw=(u+v)(u+w)(v+w). Assim, podemos ter
>>> u+v=2, u+w=v+w=1, o que dá w=0, u=v=1; u+v=2, u+w=v+w=-1, o que dá w=-2,
>>> u=v=1; u+v=-2, u+w=1, v+w=-1, o que dá w=1, u=0, v=-2; u+v=-2, u+w=-1,
>>> v+w=1, o que dá w=1, u=-2, v=0 e as coisas simétricas. Daí saem as soluções
>>> (1,0,0), (2,-1,-1), (-3/2,-1/2,3/2), (-3/2,3/2,-1/2) (e as coisas
>>> simétricas), mas as duas últimas não são inteiras, de modo que só temos as
>>> soluções que já tínhamos achado...
>>>Abraços,
>>>  Gugu
>>>
>>> Quoting Bernardo Freitas Paulo da Costa :
>>>
>>> 2018-03-20 23:14 GMT-03:00 Anderson Torres :

> Em 13 de março de 2018 20:19, Douglas Oliveira de Lima
>  escreveu:
>
>> Essa achei legal e estou postando.
>>
>> Resolva nos inteiros a seguinte equação:  (x + y)(y + z)(z + x)/2 +
>> (x + y +
>> z)3 = 1 – xyz .
>>
>>
> Substituição mágica: x=-a+b+c, y=a-b+c, z=a+b-c. Com isso, x+y=2c,
> x+y+z=a+b+c e
>
> 4abc + (a+b+c)^3 + (-a+b+c)(a-b+c)(a+b-c) = 1
>
> Usando polinômios simétricos,
>
> 4(a+b+c)(ab+ac+bc) - 4abc = 1
>
> Agora estou confuso...
>

 Note que a,b,c não precisam mais ser inteiros, podem ser inteiros
 divididos por 2 (se não me engano)

 Abraços,
 --
 Bernardo Freitas Paulo da Costa

 --
 Esta mensagem foi verificada pelo sistema de antiv?rus e
  acredita-se estar livre de perigo.


 
 =
 Instru??es para entrar na lista, sair da lista e usar a lista em
 http://www.mat.puc-rio.br/~obmlistas/obm-l.html
 
 =

>>>
>>>
>>>
>>>
>>> --
>>> Esta mensagem foi verificada pelo sistema de antivírus e
>>> acredita-se estar livre de perigo.
>>>
>>> 
>>> =
>>> Instru�ões para entrar na lista, sair da lista e usar a lista em
>>> http://www.mat.puc-rio.br/~obmlistas/obm-l.html
>>> 
>>> =
>>>
>>
>>
>> --
>> Esta mensagem foi verificada pelo sistema de antivírus e
>> acredita-se estar livre de perigo.
>
>

-- 
Esta mensagem foi verificada pelo sistema de antiv�rus e
 acredita-se estar livre de perigo.



Re: [obm-l] Re: [obm-l] Re: [obm-l] Re: [obm-l] Teoria dos números

2018-03-22 Por tôpico gugu

   Oi Claudio,
   Eu não sei de onde veio a substituição mágica do Anderson Torres -  
só achei uma fatoração na expressão obtida a partir dela... Não sou  
especialmente fã desse tipo de problema.

   Abraços,
 Gugu

Quoting Claudio Buffara :


Tudo muito bom, mas o que ninguém explicou é como foram obtidas as
fatorações/transformações algébricas mágicas.
Insight?
Conhecimentos prévios?
Tentativa e erro e muito braço?

[]s,
Claudio.


2018-03-21 18:54 GMT-03:00 :


Sim, e fazendo a=u/2, b=v/2 e c=w/2 temos (u+v+w)(uv+uw+vw)-uvw=2, ou
seja, u^2v+uv^2+u^2w+uw^2+v^2w+vw^2+2uvw=2, mas
u^2v+uv^2+u^2w+uw^2+v^2w+vw^2+2uvw=(u+v)(u+w)(v+w). Assim, podemos ter
u+v=2, u+w=v+w=1, o que dá w=0, u=v=1; u+v=2, u+w=v+w=-1, o que dá w=-2,
u=v=1; u+v=-2, u+w=1, v+w=-1, o que dá w=1, u=0, v=-2; u+v=-2, u+w=-1,
v+w=1, o que dá w=1, u=-2, v=0 e as coisas simétricas. Daí saem as soluções
(1,0,0), (2,-1,-1), (-3/2,-1/2,3/2), (-3/2,3/2,-1/2) (e as coisas
simétricas), mas as duas últimas não são inteiras, de modo que só temos as
soluções que já tínhamos achado...
   Abraços,
 Gugu

Quoting Bernardo Freitas Paulo da Costa :

2018-03-20 23:14 GMT-03:00 Anderson Torres :



Em 13 de março de 2018 20:19, Douglas Oliveira de Lima
 escreveu:


Essa achei legal e estou postando.

Resolva nos inteiros a seguinte equação:  (x + y)(y + z)(z + x)/2 + (x
+ y +
z)3 = 1 – xyz .



Substituição mágica: x=-a+b+c, y=a-b+c, z=a+b-c. Com isso, x+y=2c,
x+y+z=a+b+c e

4abc + (a+b+c)^3 + (-a+b+c)(a-b+c)(a+b-c) = 1

Usando polinômios simétricos,

4(a+b+c)(ab+ac+bc) - 4abc = 1

Agora estou confuso...



Note que a,b,c não precisam mais ser inteiros, podem ser inteiros
divididos por 2 (se não me engano)

Abraços,
--
Bernardo Freitas Paulo da Costa

--
Esta mensagem foi verificada pelo sistema de antiv?rus e
 acredita-se estar livre de perigo.


=
Instru??es para entrar na lista, sair da lista e usar a lista em
http://www.mat.puc-rio.br/~obmlistas/obm-l.html
=






--
Esta mensagem foi verificada pelo sistema de antivírus e
acredita-se estar livre de perigo.

=
Instru�ões para entrar na lista, sair da lista e usar a lista em
http://www.mat.puc-rio.br/~obmlistas/obm-l.html
=



--
Esta mensagem foi verificada pelo sistema de antiv?rus e
 acredita-se estar livre de perigo.





--
Esta mensagem foi verificada pelo sistema de antiv�rus e
acredita-se estar livre de perigo.

=
Instru��es para entrar na lista, sair da lista e usar a lista em
http://www.mat.puc-rio.br/~obmlistas/obm-l.html
=


[obm-l] Re: [obm-l] Re: [obm-l] Re: [obm-l] Re: [obm-l] Teoria dos números

2018-03-22 Por tôpico Pedro José
Boa noite!
Vi duas proposições de substituições de variáveis, nas notas anteriores e
ratifico os questionamentos do Cláudio.
Aventurei uma substituição:
a=x+y ; b=x+z; c = y + z.
Aí, na munheca cancelam-se os termos com expoentes cúbicos. E separando os
termos de
 (a+b)*(a+c), no que sobra, chega- se facilmente a:
1/2*(a+b)*(a+c)*(b+c) = 1 ou
(a+b)*(a+c)*(b+c) = 2.
Para x, y e z inteiros só atendem soluções em a, b e c com dois ímpares e
um par ou três pares.
Então, só atendem a= 0, b=1 e c= 1 ou a=-2, b=1 e c= 1. E suas permutações.
Que trazendo para as variáveis originais dá o mesmo resultado já encontrado.
Mas minha dúvida é: para esses casos de polinômios simétricos há alguma
técnica para escolha de variáveis e para fatorar, como é.g. , na mudança de
variáveis das cônicas para retirada do termo xy, possibilitando a
identificação da cônica?
Saudações,
PJMS

Em 22 de mar de 2018 7:56 AM, "Claudio Buffara" 
escreveu:

> Tudo muito bom, mas o que ninguém explicou é como foram obtidas as
> fatorações/transformações algébricas mágicas.
> Insight?
> Conhecimentos prévios?
> Tentativa e erro e muito braço?
>
> []s,
> Claudio.
>
>
> 2018-03-21 18:54 GMT-03:00 :
>
>> Sim, e fazendo a=u/2, b=v/2 e c=w/2 temos (u+v+w)(uv+uw+vw)-uvw=2, ou
>> seja, u^2v+uv^2+u^2w+uw^2+v^2w+vw^2+2uvw=2, mas
>> u^2v+uv^2+u^2w+uw^2+v^2w+vw^2+2uvw=(u+v)(u+w)(v+w). Assim, podemos ter
>> u+v=2, u+w=v+w=1, o que dá w=0, u=v=1; u+v=2, u+w=v+w=-1, o que dá w=-2,
>> u=v=1; u+v=-2, u+w=1, v+w=-1, o que dá w=1, u=0, v=-2; u+v=-2, u+w=-1,
>> v+w=1, o que dá w=1, u=-2, v=0 e as coisas simétricas. Daí saem as soluções
>> (1,0,0), (2,-1,-1), (-3/2,-1/2,3/2), (-3/2,3/2,-1/2) (e as coisas
>> simétricas), mas as duas últimas não são inteiras, de modo que só temos as
>> soluções que já tínhamos achado...
>>Abraços,
>>  Gugu
>>
>> Quoting Bernardo Freitas Paulo da Costa :
>>
>> 2018-03-20 23:14 GMT-03:00 Anderson Torres 
>>> :
>>>
 Em 13 de março de 2018 20:19, Douglas Oliveira de Lima
  escreveu:

> Essa achei legal e estou postando.
>
> Resolva nos inteiros a seguinte equação:  (x + y)(y + z)(z + x)/2 + (x
> + y +
> z)3 = 1 – xyz .
>
>
 Substituição mágica: x=-a+b+c, y=a-b+c, z=a+b-c. Com isso, x+y=2c,
 x+y+z=a+b+c e

 4abc + (a+b+c)^3 + (-a+b+c)(a-b+c)(a+b-c) = 1

 Usando polinômios simétricos,

 4(a+b+c)(ab+ac+bc) - 4abc = 1

 Agora estou confuso...

>>>
>>> Note que a,b,c não precisam mais ser inteiros, podem ser inteiros
>>> divididos por 2 (se não me engano)
>>>
>>> Abraços,
>>> --
>>> Bernardo Freitas Paulo da Costa
>>>
>>> --
>>> Esta mensagem foi verificada pelo sistema de antiv?rus e
>>>  acredita-se estar livre de perigo.
>>>
>>>
>>> 
>>> =
>>> Instru??es para entrar na lista, sair da lista e usar a lista em
>>> http://www.mat.puc-rio.br/~obmlistas/obm-l.html
>>> 
>>> =
>>>
>>
>>
>>
>>
>> --
>> Esta mensagem foi verificada pelo sistema de antivírus e
>> acredita-se estar livre de perigo.
>>
>> =
>> Instru�ões para entrar na lista, sair da lista e usar a lista em
>> http://www.mat.puc-rio.br/~obmlistas/obm-l.html
>> =
>>
>
>
> --
> Esta mensagem foi verificada pelo sistema de antivírus e
> acredita-se estar livre de perigo.

-- 
Esta mensagem foi verificada pelo sistema de antiv�rus e
 acredita-se estar livre de perigo.



[obm-l] Re: [obm-l] Re: [obm-l] Re: [obm-l] Teoria dos números

2018-03-22 Por tôpico Claudio Buffara
Tudo muito bom, mas o que ninguém explicou é como foram obtidas as
fatorações/transformações algébricas mágicas.
Insight?
Conhecimentos prévios?
Tentativa e erro e muito braço?

[]s,
Claudio.


2018-03-21 18:54 GMT-03:00 :

> Sim, e fazendo a=u/2, b=v/2 e c=w/2 temos (u+v+w)(uv+uw+vw)-uvw=2, ou
> seja, u^2v+uv^2+u^2w+uw^2+v^2w+vw^2+2uvw=2, mas
> u^2v+uv^2+u^2w+uw^2+v^2w+vw^2+2uvw=(u+v)(u+w)(v+w). Assim, podemos ter
> u+v=2, u+w=v+w=1, o que dá w=0, u=v=1; u+v=2, u+w=v+w=-1, o que dá w=-2,
> u=v=1; u+v=-2, u+w=1, v+w=-1, o que dá w=1, u=0, v=-2; u+v=-2, u+w=-1,
> v+w=1, o que dá w=1, u=-2, v=0 e as coisas simétricas. Daí saem as soluções
> (1,0,0), (2,-1,-1), (-3/2,-1/2,3/2), (-3/2,3/2,-1/2) (e as coisas
> simétricas), mas as duas últimas não são inteiras, de modo que só temos as
> soluções que já tínhamos achado...
>Abraços,
>  Gugu
>
> Quoting Bernardo Freitas Paulo da Costa :
>
> 2018-03-20 23:14 GMT-03:00 Anderson Torres :
>>
>>> Em 13 de março de 2018 20:19, Douglas Oliveira de Lima
>>>  escreveu:
>>>
 Essa achei legal e estou postando.

 Resolva nos inteiros a seguinte equação:  (x + y)(y + z)(z + x)/2 + (x
 + y +
 z)3 = 1 – xyz .


>>> Substituição mágica: x=-a+b+c, y=a-b+c, z=a+b-c. Com isso, x+y=2c,
>>> x+y+z=a+b+c e
>>>
>>> 4abc + (a+b+c)^3 + (-a+b+c)(a-b+c)(a+b-c) = 1
>>>
>>> Usando polinômios simétricos,
>>>
>>> 4(a+b+c)(ab+ac+bc) - 4abc = 1
>>>
>>> Agora estou confuso...
>>>
>>
>> Note que a,b,c não precisam mais ser inteiros, podem ser inteiros
>> divididos por 2 (se não me engano)
>>
>> Abraços,
>> --
>> Bernardo Freitas Paulo da Costa
>>
>> --
>> Esta mensagem foi verificada pelo sistema de antiv?rus e
>>  acredita-se estar livre de perigo.
>>
>>
>> =
>> Instru??es para entrar na lista, sair da lista e usar a lista em
>> http://www.mat.puc-rio.br/~obmlistas/obm-l.html
>> =
>>
>
>
>
>
> --
> Esta mensagem foi verificada pelo sistema de antivírus e
> acredita-se estar livre de perigo.
>
> =
> Instru�ões para entrar na lista, sair da lista e usar a lista em
> http://www.mat.puc-rio.br/~obmlistas/obm-l.html
> =
>

-- 
Esta mensagem foi verificada pelo sistema de antiv�rus e
 acredita-se estar livre de perigo.



  1   2   3   >